Exercices de géométrie affine et euclidienne

publicité
Accueil
Géométrie affine
Convexité
Géométrie euclidienne
Execices de géométrie
Isométries planes
Isométries de l’espace
affine et euclidienne
Coniques
Géométrie analytique
Complexes
Homographies
Jean-Marc Decauwert
Aide
version du 22 décembre 2006
Précédente
Suivante
Plein écran
Quitter
[email protected]
Géométrie affine
Accueil
Géométrie affine
Convexité
Géométrie euclidienne
Isométries planes
Isométries de l’espace
Coniques
Géométrie analytique
Complexes
Homographies
Aide
Précédente
Suivante
Plein écran
Quitter
•
•
•
•
•
•
•
•
•
•
•
•
•
•
•
•
•
Quadrilatère
Composition de symétries centrales
Composition d’homothéties
Le trapèze
Polygone des milieux
Le tourniquet dans le triangle
Un problème de construction
Deux triangles
Un cas particulier du théorème de Desargues
Une droite et son image
Transformations cycliques
Affinités et transvections
Desargues dans l’espace
Projection centrale, rapport et birapport
Coordonnées barycentriques et déterminants
Aire algébrique d’un polygone
Action du groupe affine sur les triplets de droites
Convexité
Accueil
Géométrie affine
Convexité
Géométrie euclidienne
Isométries planes
Isométries de l’espace
Coniques
Géométrie analytique
Complexes
Homographies
Aide
Précédente
Suivante
Plein écran
Quitter
•
•
•
•
•
•
•
•
•
•
•
•
•
Demi-espaces
Régionnement du plan par un repère affine
Cônes convexes
Quadrilatères
Diagonales d’un polygone convexe
Milieux
Une propriété des triangles
Le théorème de Carathéodory
Projection sur un convexe fermé
Séparation de convexes
Hyperplans d’appui
Génération par les demi-espaces
Domaines de Voronoı̈
Géométrie euclidienne
Accueil
Géométrie affine
Convexité
Géométrie euclidienne
Isométries planes
Isométries de l’espace
Coniques
Géométrie analytique
Complexes
Homographies
Aide
Précédente
Suivante
Plein écran
Quitter
•
•
•
•
•
•
•
•
•
•
•
•
•
•
•
Distances aux points d’un repère affine
Fonction scalaire de Leibniz
Cercles d’Apollonius
Triangle orthique
Bissectrices et cercle circonscrit
Le pivot
Cercles tangents
Trois cercles
Le théorème des trois tangentes
Rayons des cercles inscrit et exinscrits
Un problème de maximisation
Le problème de Fermat
Trisection
Un problème de recouvrement
Disque de rayon minimal contenant un compact
Isométries planes
Accueil
Géométrie affine
Convexité
Géométrie euclidienne
Isométries planes
Isométries de l’espace
Coniques
Géométrie analytique
Complexes
Homographies
Aide
Précédente
Suivante
Plein écran
Quitter
•
•
•
•
•
•
•
•
•
•
•
•
•
Composition de réflexions (1)
Composition de réflexions (2)
Composition de réflexions (3)
Symétrie glissée
Composition de rotations
Composition de symétries glissées
Le tourniquet dans le cercle
Polygone régulier
Deux carrés (ou trois)
Billard polygonal
Plus court chemin
Le problème de Fagnano
Sous-groupes finis d’isométries
Isométries de l’espace
Accueil
Géométrie affine
Convexité
Géométrie euclidienne
Isométries planes
Isométries de l’espace
Coniques
Géométrie analytique
Complexes
Homographies
Aide
Précédente
Suivante
Plein écran
Quitter
•
•
•
•
•
•
•
Composée de trois réflexions
Caractérisation de l’axe d’un vissage
Tétraèdres équifaciaux
Isométries du tétraèdre régulier
Isométries du cube
Cube, tétraèdres et octaèdre
Isométries de l’hélice circulaire
Coniques
Accueil
Géométrie affine
Convexité
Géométrie euclidienne
Isométries planes
Isométries de l’espace
Coniques
Géométrie analytique
Complexes
Homographies
Aide
Précédente
Suivante
Plein écran
Quitter
•
•
•
•
•
Tangentes menées d’un point à la parabole
Un problème de lieu géométrique
Diamètres conjugués de l’ellipse
Ellipse de Steiner d’un triangle
Construction de l’hyperbole
Géométrie analytique
Accueil
Géométrie affine
Convexité
Géométrie euclidienne
Isométries planes
Isométries de l’espace
Coniques
Géométrie analytique
Complexes
Homographies
Aide
Précédente
Suivante
Plein écran
Quitter
•
•
•
•
•
•
•
•
•
•
•
Projection et affinités
Transformation affine du plan
Position relative de deux cercles
Perpendiculaire commune
Equation normale d’une droite, bissectrices
Réflexion
Isométrie de l’espace (1)
Isométrie de l’espace (2)
Isométries du cube et du tétraèdre
Equation d’une conique
Une construction de l’ellipse
Complexes
Accueil
Géométrie affine
Convexité
Géométrie euclidienne
Isométries planes
Isométries de l’espace
Coniques
Géométrie analytique
Complexes
Homographies
Aide
Précédente
Suivante
Plein écran
Quitter
•
•
•
•
•
•
•
•
Paramétrisation du cercle
Orthocentre
Deux carrés (ou trois)
Un triangle et son image
Configuration de Vecten
Triangles de Napoléon
Le théorème de Ptolémée
Projection orthogonale d’un cube
Homographies
Accueil
Géométrie affine
Convexité
Géométrie euclidienne
Isométries planes
Isométries de l’espace
Coniques
Géométrie analytique
Complexes
Homographies
Aide
Précédente
Suivante
Plein écran
Quitter
•
•
•
•
z − a Lignes de niveau de
z − b
Matrices et homographies
Points fixes d’une homographie
Invariant anallagmatique de deux cercles
Quadrilatère
Accueil
Géométrie affine
Convexité
Soit, dans le plan affine, ABCD un quadrilatère, I, J, K, L les milieux respectifs
des segments AB, BC, CD et DA. Montrer que IK et JL ont même milieu.
Géométrie euclidienne
Isométries planes
Isométries de l’espace
Coniques
Géométrie analytique
Complexes
Homographies
Aide
Précédente
Suivante
Plein écran
Quitter
Indication
Solution
Que peut-on dire de l’isobarycentre des quatre points A, B, C, D ?
Accueil
Géométrie affine
Convexité
Géométrie euclidienne
Isométries planes
Isométries de l’espace
Coniques
Géométrie analytique
Complexes
Homographies
Aide
Précédente
Suivante
Plein écran
Quitter
Retour
Solution
L’isobarycentre des quatre points A, B, C, D est, par associativité, l’isobarycentre
de I et K, i.e. le milieu du segment IK. C’est aussi, pour la même raison, celui de
JL.
Accueil
Géométrie affine
Convexité
Géométrie euclidienne
Isométries planes
Isométries de l’espace
Coniques
Géométrie analytique
Complexes
Homographies
Aide
Précédente
Suivante
Plein écran
Quitter
Autre solution
Table
Accueil
Géométrie affine
−
→ −−→ 1 −→
Le quadrilatère IJKL est un parallélogramme, puisque IJ = LK = AC (considérez
2
les triangles ABC et ACD). Ses diagonales IK et JL se coupent donc en leurs
milieux.
Convexité
Géométrie euclidienne
Isométries planes
Isométries de l’espace
Coniques
Géométrie analytique
Complexes
Homographies
Aide
Précédente
Suivante
Plein écran
Quitter
Table
Composition de symétries centrales
Accueil
Géométrie affine
Convexité
Géométrie euclidienne
Soit ABC un triangle, A0 , B 0 , C 0 les milieux respectifs de BC, CA et AB, sA0 ,
sB 0 , sC 0 les symétries centrales par rapport à ces points. Déterminer la nature
géométrique des transformations f = sB 0 ◦ sA0 et g = sC 0 ◦ sB 0 ◦ sA0 .
Isométries planes
Isométries de l’espace
Coniques
Géométrie analytique
Complexes
Homographies
Aide
Précédente
Suivante
Plein écran
Quitter
Indication
Solution
Déterminez les parties linéaires f~ et ~g de f et g, puis les images f (B) et g(B) du
point B par ces deux transformations.
Accueil
Géométrie affine
Convexité
Géométrie euclidienne
Isométries planes
Isométries de l’espace
Coniques
Géométrie analytique
Complexes
Homographies
Aide
Précédente
Suivante
Plein écran
Quitter
Retour
Solution
Accueil
Géométrie affine
Convexité
Géométrie euclidienne
La partie linéaire d’une symétrie centrale est l’homothétie vectorielle de rapport -1.
La composée de deux symétries centrales a pour partie linéaire l’identité, c’est donc
une translation. La composée de trois symétries centrales a pour partie linéaire
l’homothétie vectorielle de rapport -1, c’est donc une symétrie centrale. Comme
−→
f (B) = A et g(B) = B, f est la translation de vecteur BA et g la symétrie
centrale de centre B.
Isométries planes
Isométries de l’espace
Coniques
Géométrie analytique
Complexes
Homographies
Aide
Précédente
Suivante
Plein écran
Quitter
Table
Composition d’homotheties
Accueil
Géométrie affine
Convexité
Géométrie euclidienne
Soient, dans le plan affine E, h1 et h2 deux homothéties de centres respectifs O1
et O2 et de même rapport λ 6= 0. Déterminer la nature géométrique de la transformation f = h2 ◦ h−1
1 .
Isométries planes
Isométries de l’espace
Coniques
Géométrie analytique
Complexes
Homographies
Aide
Précédente
Suivante
Plein écran
Quitter
Solution
Accueil
Géométrie affine
Convexité
Géométrie euclidienne
La partie linéaire d’une homothétie affine de rapport λ est l’homothétie vectorielle
→
−
de rapport λ. La partie linéaire de f = h2 ◦h−1
1 est donc l’identité de E . Il en résulte
que f est une translation. L’image f (O1 ) = h2 (O1 ) de O1 par cette translation
−−−−−→
−−−−−→ −−−→ −−−−−→
−−−→
−−−→
vérifie O2 f (O1 ) = λO2 O1 , d’où O1 f (O1 ) = O1 O2 + O2 f (O1 = (1 − λ)O1 O2 : f est
−−−→
donc la translation de vecteur (1 − λ)O1 O2 .
Isométries planes
Isométries de l’espace
Coniques
Géométrie analytique
Complexes
Homographies
Aide
Précédente
Suivante
Plein écran
Quitter
Suite
Soit F une partie non vide de E, F1 = h1 (F ) son image par h1 , F2 = h2 (F )
son image par h2 . Montrer que F2 est l’image de F1 par une translation dont on
précisera le vecteur.
Accueil
Géométrie affine
Convexité
Géométrie euclidienne
Isométries planes
Isométries de l’espace
Coniques
Géométrie analytique
Complexes
Homographies
Aide
Précédente
Suivante
Plein écran
Quitter
Solution
−−−→
F2 est l’image de F1 par la translation f de vecteur (1 − λ)O1 O2 .
Accueil
Géométrie affine
Convexité
Géométrie euclidienne
Isométries planes
Isométries de l’espace
Coniques
Géométrie analytique
Complexes
Homographies
Aide
Précédente
Suivante
Plein écran
Quitter
Table
Le trapèze
Accueil
Géométrie affine
Convexité
Géométrie euclidienne
Isométries planes
Isométries de l’espace
Coniques
Soit ABCD un trapèze de bases AB et CD. On note K et L les milieux de AB
et CD et on suppose que les droites AD et BC se coupent en un point I et les
droites AC et BD en un point J. Montrer que les points I, J, K, L sont alignés
et que :
IK JL
= −1 .
IL JK
Géométrie analytique
Complexes
Homographies
Aide
Précédente
Suivante
Plein écran
Quitter
Indication
Solution
Considérez des homothéties de centres I et J qui transforment la droite AB en la
droite CD.
Accueil
Géométrie affine
Convexité
Géométrie euclidienne
Isométries planes
Isométries de l’espace
Coniques
Géométrie analytique
Complexes
Homographies
Aide
Précédente
Suivante
Plein écran
Quitter
Retour
Solution
Accueil
Géométrie affine
Convexité
Géométrie euclidienne
Isométries planes
Isométries de l’espace
Coniques
Géométrie analytique
Complexes
Il existe une unique homothétie h de centre I transformant D en A. Cette homothétie transforme la droite CD en une droite parallèle passant par A, i.e. en la
droite AB. Elle transforme donc C en B, et le milieu L de CD en le milieu K de
AB. Les points I, K et L sont donc alignés et le rapport de h est égal à IK
. De
IL
0
même l’unique homothétie h de centre J transformant A en C transforme B en
D et le milieu K de AB en le milieu L de CD. Les points J, K et L sont donc
JL
alignés et le rapport de h0 est égal à JK
. La composée h0 ◦h de ces deux homothéties
transforme D en C, C en D, et laisse fixe L. C’est donc la symétrie par rapport
à L, i.e. l’homothétie de centre L et de rapport −1. Mais son rapport est aussi le
JL
des rapports de ces deux homothéties.
produit IK
IL JK
Homographies
Aide
Précédente
Suivante
Plein écran
Table
Quitter
Polygone des milieux (1)
Accueil
Géométrie affine
Convexité
Géométrie euclidienne
Soit ABC un triangle. Montrer qu’il existe un triangle A0 B 0 C 0 et un seul tel que
A soit le milieu de B 0 C 0 , B le milieu de C 0 A0 , et C le milieu de A0 B 0 . Indiquer une
construction géométrique de ce triangle.
Isométries planes
Isométries de l’espace
Coniques
Géométrie analytique
Complexes
Homographies
Aide
Précédente
Suivante
Plein écran
Quitter
Solution
Suite
Polygone des milieux (2)
Accueil
Géométrie affine
Convexité
Géométrie euclidienne
Isométries planes
Soit A0 B 0 C 0 D0 un quadrilatère. Donner une condition nécessaire et suffisante pour
qu’il existe un quadrilatère ABCD tel que A0 soit le milieu de AB, B 0 le milieu de
BC, C 0 le milieu de CD, et D0 le milieu de DA. Ce quadrilatère, s’il existe, est-il
unique?
Isométries de l’espace
Coniques
Géométrie analytique
Complexes
Homographies
Aide
Précédente
Suivante
Plein écran
Quitter
Indication
Solution
Suite
Accueil
Géométrie affine
Convexité
Géométrie euclidienne
Isométries planes
Isométries de l’espace
−−→
Si ABCD existe, les égalités A0 B 0 =
−−0 →0 1 −→
D C = AC montrent que A0 B 0 C 0 D0
2
est un parallélogramme. Cette condition est donc nécessaire pour qu’il
existe une solution.
Coniques
Géométrie analytique
Complexes
Homographies
Aide
Si elle est vérifiée, soit A un point quelconque du plan, B son symétrique par
rapport à A0 , C le symétrique de B par rapport à B 0 , D le symétrique de C par
rapport à C 0 ; D0 est alors le milieu de DA, ce qui montre que ABCD est solution.
Il y a donc dans ce cas une infinité de solutions : l’un des points ABCD peut être
choisi arbitrairement, les autres sont alors uniquement déterminés.
Précédente
Suivante
Plein écran
Quitter
Retour
Suite
Polygone des milieux (3)
Accueil
Géométrie affine
Convexité
Géométrie euclidienne
Isométries planes
Isométries de l’espace
Etant donnés n points B1 , . . . ,Bn du plan affine E, peut-on toujours trouver n
points A1 , . . . ,An de E tels que Bi soit, pour tout i = 1, . . . ,n, le milieu de Ai Ai+1
(avec la convention An+1 = A1 )?
Donner une construction géométrique des points Ai à partir des points Bi lorsque
la solution existe.
Coniques
Géométrie analytique
Complexes
Homographies
Aide
Précédente
Suivante
Plein écran
Quitter
Indication
Solution
Le polygone des milieux est-il quelconque?
Accueil
Géométrie affine
Convexité
Géométrie euclidienne
Isométries planes
Isométries de l’espace
Coniques
Géométrie analytique
Complexes
Homographies
Aide
Précédente
Suivante
Plein écran
Quitter
Retour
Solution
Indication
Accueil
Géométrie affine
On pourra considérer la composée des symétries de centres B1 , B2 , . . . ,Bn .
Convexité
Géométrie euclidienne
Isométries planes
Isométries de l’espace
Coniques
Géométrie analytique
Complexes
Homographies
Aide
Précédente
Suivante
Plein écran
Quitter
Retour
Solution
Solution
Accueil
Géométrie affine
Convexité
Il suffit naturellement de considérer le triangle obtenu en traçant les parallèles aux
côtés menées par les sommets opposés.
Géométrie euclidienne
Isométries planes
Isométries de l’espace
Coniques
Géométrie analytique
Complexes
Homographies
Aide
Précédente
Suivante
Plein écran
Quitter
Retour
Suite
Solution
Accueil
Géométrie affine
Convexité
Géométrie euclidienne
Isométries planes
Isométries de l’espace
Coniques
Géométrie analytique
Complexes
Homographies
Si le problème admet une solution A1 , . . . , An , la composée f = sBn ◦ · · · ◦ sB1 des
symétries de centres B1 , B2 , . . . ,Bn laisse fixe le point A1 . Or cette composée est :
• une symétrie centrale si n est impair ;
−−−→
−−−−−→
• la translation de vecteur ~v = 2(B1 B2 + · · · + Bn−1 Bn ) si n est pair.
Si n est impair, le problème admet donc une solution et une seule : A1 est le centre
de f , A2 = sB1 (A1 ), . . . .
Si n est pair, soit ~v = ~0 : f est alors l’identité et le problème admet une infinité de
solutions (A1 quelconque, A2 = sB1 (A1 ), . . . ), soit ~v 6= ~0 : f n’a dans ce cas pas de
point fixe et le problème n’admet pas de solution.
Aide
Précédente
Comment construisez-vous la solution si n est impair?
Suivante
Plein écran
Quitter
Autre solution
Solution
Solution
Accueil
Géométrie affine
Convexité
Il suffit de partir d’un point quelconque M et de construire son image f (M ) : le
centre de f est le milieu de M f (M ), qui est donc le point A1 .
Géométrie euclidienne
Isométries planes
Isométries de l’espace
Coniques
Géométrie analytique
Complexes
Homographies
Aide
Précédente
Suivante
Plein écran
Quitter
Retour
Table
Autre solution
Accueil
Géométrie affine
Convexité
Géométrie euclidienne
Isométries planes
Isométries de l’espace
Coniques
Géométrie analytique
Complexes
Si a1 , . . . ,an sont les affixes des points Ak et b1 , . . . ,bn les affixes des points Bk , le
problème équivaut à résoudre le système:
a1 + a2
a2 + a3
...
an−1 + an
a1 + an
=
=
=
=
=
2b1
2b2
...
2bn−1
2bn
Homographies
Aide
Précédente
Suivante
Le déterminant de ce système se calcule en développant par rapport à la première
colonnne :
1 1 0
.
.
.
0 1 1 0
.
.
.
.
.
.
.
.
.
n+1
.
= 1 + (−1)
.
.
.
.
.
.
.
.
.
.
0
1
1
1 0
. . 0 1
Plein écran
Quitter
Suite
Accueil
Géométrie affine
Convexité
Si n est impair, le système est de Cramer : il admet donc une solution et une seule
quels que soient les points Bk .
Si n est pair, le déterminant est nul (ce qu’on pouvait voir directement en remarquant que la somme des lignes d’indice pair est égale à la somme des lignes d’indice
impair) et le sytème est de rang n − 1. Si la condition
Géométrie euclidienne
Isométries planes
Isométries de l’espace
Coniques
Géométrie analytique
Complexes
a1 + a3 + · · · + an−1 = a2 + a4 + · · · + an
est vérifiée, le système admet une infinité de solutions (on peut choisir arbitrairement a1 et les autres points sont alors uniquement déterminés) ; sinon le sytème
n’admet pas de solution.
Homographies
Aide
Précédente
Suivante
Plein écran
Quitter
Retour
Table
Le tourniquet dans le triangle
Accueil
Géométrie affine
Convexité
Géométrie euclidienne
Isométries planes
Isométries de l’espace
Par un point D du côté AB d’un triangle ABC on trace la parallèle à BC qui
coupe AC en E ; par E on trace la parallèle à AB qui coupe CB en F ; par F on
trace la parallèle à CA qui coupe BA en G ; par G on trace la parallèle à BC qui
coupe AC en H ; par H on trace la parallèle à AB qui coupe CB en I ; par I on
trace la parallèle à CA qui coupe BA en J. Montrer que J = D.
Coniques
Géométrie analytique
Complexes
Homographies
Aide
Précédente
Suivante
Plein écran
Quitter
Indication
Solution
Remarquez que l’application de la droite AB dans elle-même qui au point D associe
le point G est affine. Quelles sont les images des points A et B par cette application?
Accueil
Géométrie affine
Convexité
Géométrie euclidienne
Isométries planes
Isométries de l’espace
Coniques
Géométrie analytique
Complexes
Homographies
Aide
Précédente
Suivante
Plein écran
Quitter
Retour
Solution
Accueil
Géométrie affine
L’application f de la droite AB dans elle-même qui au point D associe le point G
est affine, puisque composée de trois projections. Elle échange les points A et B et
est donc involutive, puisque (A,B) est un repère affine de cette droite. L’image J
de D par f ◦ f est donc égale à D.
Convexité
Géométrie euclidienne
Isométries planes
Isométries de l’espace
Coniques
Géométrie analytique
Complexes
Homographies
Aide
Précédente
Suivante
Plein écran
Remarque : L’application f est donc la symétrie par rapport au milieu de AB.
En appliquant trois fois le théorème de Thalès, on pouvait d’ailleurs montrer que
DA
GB
=
, et en déduire que D et G étaient symétriques par rapport à ce milieu.
DB
GA
Table
Quitter
Un problème de construction
Accueil
Géométrie affine
Convexité
Soit, dans un plan affine E, ABCD un parallélogramme dont on suppose les sommets A et B fixés. Déterminer le lieu de D quand C décrit une droite ∆ de E.
Géométrie euclidienne
Isométries planes
Isométries de l’espace
Coniques
Géométrie analytique
Complexes
Homographies
Aide
Précédente
Suivante
Plein écran
Quitter
Solution
Suite
−→
Le point D se déduit de C par la translation de vecteur BA. Il décrit donc, quand
C décrit la droite ∆, la droite ∆1 image de ∆ par cette translation.
Accueil
Géométrie affine
Convexité
Géométrie euclidienne
Isométries planes
Isométries de l’espace
Coniques
Géométrie analytique
Complexes
Homographies
Aide
Précédente
Suivante
Plein écran
Quitter
Suite
Accueil
Géométrie affine
En déduire une construction d’un parallélogramme ABCD dont les sommets A et
B sont fixés, les sommets C et D devant appartenir respectivement à deux droites
∆ et ∆0 données de E (on discutera l’existence et l’unicité de la solution selon la
position de ces droites).
Convexité
Géométrie euclidienne
Isométries planes
Isométries de l’espace
Coniques
Géométrie analytique
Complexes
Homographies
Aide
Précédente
Suivante
Plein écran
Quitter
Solution
Le point D s’obtient comme intersection de ∆0 et de la droite ∆1 déduite de ∆ par
−→
la translation de vecteur BA.
Accueil
Géométrie affine
Convexité
Géométrie euclidienne
Isométries planes
Isométries de l’espace
Coniques
Géométrie analytique
Complexes
Homographies
Aide
Précédente
Suivante
Plein écran
Quitter
Table
Deux triangles (1)
Accueil
Géométrie affine
Convexité
Géométrie euclidienne
Soit, dans le plan affine, ABC un triangle non aplati, A1 le symétrique de B par
rapport à C, B1 le symétrique de C par rapport à A, C1 le symétrique de A par
rapport à B. Comparer les aires des triangles ABC et A1 B1 C1 .
Isométries planes
Isométries de l’espace
Coniques
Géométrie analytique
Complexes
Homographies
Aide
Précédente
Suivante
Plein écran
Quitter
Indication
Solution
−−−→ −−−→
−→ −−→
Comparez par exemple les déterminants det(A1 B1 ,A1 C1 ) et det(CA,CB).
Accueil
Géométrie affine
Convexité
Géométrie euclidienne
Isométries planes
Isométries de l’espace
Coniques
Géométrie analytique
Complexes
Homographies
Aide
Précédente
Suivante
Plein écran
Quitter
Retour
Solution
On a :
Accueil
Géométrie affine
Convexité
Géométrie euclidienne
Isométries planes
Isométries de l’espace
−−−→ −−−→
−−→ −−→ −−→ −−→
det(A1 B1 ,A1 C1 ) = det(A1 C + CB1 ,A1 B + BC1 )
−−→
−→ −−→ −→
= det(CB + 2CA,2CB − BA)
−−→
−→ −−→ −→
= det(CB + 2CA,3CB − CA)
−→ −−→
= 7 det(CA,CB)
Coniques
Géométrie analytique
Complexes
Homographies
Aide
Précédente
L’aire
du
triangle
A1 B1 C1 est donc 7 fois
celle du triangle ABC.
Suivante
Plein écran
Quitter
Remarque
Suite
On pouvait aussi remarquer que les aires de chacun des triangles A1 BC1 , B1 CA1
et C1 AB1 étaient égales au double de celle du triangle ABC.
Accueil
Géométrie affine
Convexité
Géométrie euclidienne
Isométries planes
Isométries de l’espace
Coniques
Géométrie analytique
Complexes
Homographies
Aide
Précédente
Suivante
Plein écran
Quitter
Retour
Suite
Deux triangles (2)
Accueil
Géométrie affine
Reconstruire le triangle ABC à partir du triangle A1 B1 C1 .
Convexité
Géométrie euclidienne
Isométries planes
Isométries de l’espace
Coniques
Géométrie analytique
Complexes
Homographies
Aide
Précédente
Suivante
Plein écran
Quitter
Indication
Solution
Accueil
Géométrie affine
Soient A2 , B2 , C2 les points d’intersection des droites BC, CA et AB avec les
droites B1 C1 , C1 A1 et A1 B1 . Ecrire les coordonnées barycentriques des points A1 ,
B1 et C1 dans le repère affine (A,B,C). En déduire les coordonnées barycentriques
des points A, B, C, puis celles des points A2 , B2 , C2 dans le repère affine (A1 ,B1 ,C1 ).
Convexité
Géométrie euclidienne
Isométries planes
Isométries de l’espace
Coniques
Géométrie analytique
Complexes
Homographies
Aide
Précédente
Suivante
Plein écran
Quitter
Retour
Solution
Soient A2 , B2 , C2 les points d’intersection des droites BC, CA et AB avec les
droites B1 C1 , C1 A1 et A1 B1 . On a :
Accueil
A1 = 2C − B
Géométrie affine
Convexité
B1 = 2A − C
Géométrie euclidienne
Isométries planes
C1 = 2B − A
Isométries de l’espace
Coniques
Géométrie analytique
Complexes
Homographies
Aide
Précédente
L’égalité
d’où :
2
A = A1 +
7
1
B = A1 +
7
4
C = A1 +
7
4
B1 +
7
2
B1 +
7
1
B1 +
7
1
C1
7
4
C1
7
2
C1
7
!
4
1
6 1
2
1
2
A1 + B1 + C1
A = A1 + B1 + C1 =
7
7
7
7 3
3
7
1
2
montre que C2 = A1 + B1 , puisque ce dernier point appartient aux deux droites
3
3
A1 B1 et AC1 . C2 est donc situé au tiers du segment B1 A1 à partir de B1 . La droite
AB s’en déduit, puisque les points C1 et C2 lui appartiennent. Les autres côtés du
triangle ABC s’obtiennent de la même manière.
Suivante
Plein écran
Quitter
Retour
Table
Un cas particulier du théorème de Desargues
Accueil
Géométrie affine
Convexité
Géométrie euclidienne
Montrer que deux triangles non aplatis du plan affine se déduisent l’un de l’autre
par une homothétie ou une translation si et seulement si leurs côtés sont deux à
deux parallèles.
Isométries planes
Isométries de l’espace
Coniques
Géométrie analytique
Complexes
Homographies
Aide
Précédente
Suivante
Plein écran
Quitter
Indication
Solution
−−→
−→
Si A0 B 0 = λAB, il existe une et une seule homothétie ou translation f qui transforme A en A0 et B en B 0 (démonstration). Que peut-on dire de f (C)?
Accueil
Géométrie affine
Convexité
Géométrie euclidienne
Isométries planes
Isométries de l’espace
Coniques
Géométrie analytique
Complexes
Homographies
Aide
Précédente
Suivante
Plein écran
Quitter
Retour
Solution
Accueil
Géométrie affine
Convexité
Géométrie euclidienne
−−→ −→
−−→ −−→
Si A0 B 0 = AB, le quadrilatère ABB 0 A0 est un parallélogramme et AA0 = BB 0 : la
−−→
translation de vecteur AA0 transforme donc A en A0 et B en B 0 .
−−→
−→
Si A0 B 0 = λAB, avec λ 6= 1, 0, il existe une homothétie et une seule de rapport
λ qui transforme A en A0 et B en B 0 . Son centre O est déterminé par la relation
−−→0
−→
−→ −−→
−→
−→ −−→
OA = λOA, qui s’écrit encore OA + AA0 = λOA, ou (1 − λ)AO = AA0 .
Isométries planes
Isométries de l’espace
Coniques
Géométrie analytique
Complexes
Homographies
Aide
Précédente
Suivante
Plein écran
Quitter
Retour
Accueil
Géométrie affine
Convexité
Géométrie euclidienne
Une homothétie ou une translation conserve le parallélisme : la condition est donc
nécessaire.
Réciproquement, les droites AB et A0 B 0 étant parallèles, il existe une homothétie
ou une translation f qui transforme A en A0 et B en B 0 . L’image f (C) de C par f
appartient à la parallèle à AC passant par A0 , i.e. à la droite A0 C 0 et à la parallèle
à BC passant par B 0 , i.e. à la droite B 0 C 0 : c’est donc le point C 0 .
Isométries planes
Isométries de l’espace
Coniques
Géométrie analytique
Complexes
Homographies
Aide
Précédente
Suivante
Plein écran
Quitter
Application
Accueil
Géométrie affine
Soient D1 et D2 deux droites sécantes du plan affine E et M un point de E
n’appartenant à aucune de ces droites. On suppose que le point O d’intersection
de D1 et D2 est situé hors du cadre de la figure. Donner une construction de la
droite OM .
Convexité
Géométrie euclidienne
Isométries planes
Isométries de l’espace
Coniques
Géométrie analytique
Complexes
Homographies
Aide
Précédente
Suivante
Plein écran
Quitter
Solution
Accueil
Géométrie affine
Soient P1 un point de D1 et P2 un point de D2 tels que la droite P1 P2 ne passe pas
par M . Une parallèle à P1 P2 coupe D1 en P10 et D2 en P20 . Les parallèles à P1 M
(resp. P2 M ) menées par P10 (resp. P20 ) se coupent en M 0 . La droite M M 0 passe par
O.
Convexité
Géométrie euclidienne
Isométries planes
Isométries de l’espace
Coniques
Géométrie analytique
Complexes
Homographies
Aide
Précédente
Suivante
Plein écran
Quitter
Table
Une droite et son image
Accueil
Géométrie affine
Convexité
Géométrie euclidienne
Soit f une transformation affine d’un espace affine E et D une droite de E. Montrer
que l’ensemble des milieux des segments M f (M ) pour M ∈ D est une droite ou
un point.
Isométries planes
Isométries de l’espace
Coniques
Géométrie analytique
Complexes
Homographies
Aide
Précédente
Suivante
Plein écran
Quitter
Indication
Solution
Soient A et B deux points distincts de D. Exprimez M (resp. f (M )) comme barycentre de A et B (resp. f (A) et f (B)).
Accueil
Géométrie affine
Convexité
Géométrie euclidienne
Isométries planes
Isométries de l’espace
Coniques
Géométrie analytique
Complexes
Homographies
Aide
Précédente
Suivante
Plein écran
Quitter
Retour
Solution
Accueil
Géométrie affine
Convexité
Géométrie euclidienne
Isométries planes
Isométries de l’espace
Coniques
Soient A et B deux points distincts de D. Tout point M de E s’écrit de manière
unique comme barycentre αA + (1 − α)B (α ∈ R) de A et B. La transformation
f étant affine, elle conserve les barycentres : f (M ) = αf (A) + (1 − α)f (B). Le
α
1−α
milieu g(M ) de M f (M ) est le barycentre du système pondéré (A, ), (B,
),
2
2
α
1−α
(f (A), ), (f (B),
), ou encore du système (g(A), α), (g(B), 1 − α), où g(A)
2
2
est le milieu de Af (A) et g(B) le milieu de Bf (B). Si g(A) = g(B), on a g(M ) =
g(A) = g(B) pour tout point M de D. Sinon le point g(M ) = αg(A) + (1 − α)g(B)
décrit la droite g(A)g(B) quand α décrit R.
Géométrie analytique
Complexes
Homographies
Aide
Précédente
Suivante
Plein écran
Quitter
Autre solution
Table
Isométries planes
On peut aussi remarquer que l’application g qui à M associe le milieu g(M ) de
→
f~ + id−
E . En effet si M et N sont
M f (M ) est affine, d’application linéaire associée
2
deux points de E, on vérifie facilement que
−−→ −−−−−−−→ −−→
−−→
−−−−−−−→ M N + f (M )f (N ) M N + f~(M N )
g(M )g(N ) =
=
.
2
2
Isométries de l’espace
Il en résulte que l’image g(D) de la droite D par g est une droite ou un point.
Accueil
Géométrie affine
Convexité
Géométrie euclidienne
Coniques
Géométrie analytique
Complexes
Homographies
Aide
Précédente
Suivante
Plein écran
Quitter
Table
Transformations cycliques
Accueil
Géométrie affine
Convexité
Géométrie euclidienne
Isométries planes
Isométries de l’espace
Soient A, B, C, D quatre points du plan affine P tels que trois d’entre eux ne soient
jamais alignés. Donner une condition nécessaire et suffisante sur le quadrilatère
ABCD pour qu’il existe une transformation affine f de P vérifiant f (A) = B,
f (B) = C, f (C) = D, f (D) = A. Montrer qu’une telle transformation, si elle
existe, est un élément d’ordre 4 du groupe affine de P .
Coniques
Géométrie analytique
Complexes
Homographies
Aide
Précédente
Suivante
Plein écran
Quitter
Solution
Les points A, B, C constituent un repère affine de P . Soit D = αA + βB + γC,
avec α + β + γ = 1, l’écriture de D dans ce repère. Si une telle transformation f
existe, elle vérifie :
Accueil
Géométrie affine
Convexité
Géométrie euclidienne
Isométries planes
f (D) = αf (A) + βf (B) + γf (C)
= αB + βC + γ(αA + βB + γC)
= αγA + (α + γβ)B + (β + γ 2 )C .
Isométries de l’espace
Coniques
Géométrie analytique
Complexes
Homographies
Aide
Précédente
Suivante
L’égalité f (D) = A se traduit alors par le système αγ = 1, α + γβ = 0, β + γ 2 = 0.
On en déduit β = −γ 2 , α = γ 3 , γ 4 = 1, d’où γ = ±1. Si γ = −1, on a α = β = γ =
−1, ce qui est impossible, puisque α + β + γ = 1. On a donc α = γ = 1, β = −1,
ce qui signifie que ABCD est un parallélogramme.
Réciproquement, si ABCD est un parallélogramme, l’unique transformation affine
de P vérifiant f (A) = B, f (B) = C, f (C) = D vérifie aussi f (D) = f (A−B+C) =
B − C + D = A.
Les deux transformations affines f 4 et idP coı̈ncident sur le repère affine A, B, C ;
elles sont donc égales. Par contre f 2 6= idP (f 2 est la symétrie centrale par rapport
au centre du parallélogramme) : f est donc d’ordre 4 dans GA(P ).
Plein écran
Quitter
Retour
Table
Affinités et transvections (1)
Accueil
Géométrie affine
Convexité
Soit (A0 , . . . ,An ) un repère affine d’un espace affine E de dimension n et A00 un
point de E n’appartenant pas à l’hyperplan affine H engendré par A1 , . . . , An .
Géométrie euclidienne
Isométries planes
Isométries de l’espace
Montrer qu’il existe une transformation affine f de E et une seule qui laisse fixe
tout point de H et transforme A0 en A00 .
Coniques
Géométrie analytique
Complexes
Homographies
Aide
Précédente
Suivante
Plein écran
Quitter
Solution
Suite
On sait qu’il existe une et une seule transformation affine f transformant le repère
affine (A0 ,A1 , . . . ,An ) en la famille (A00 ,A1 , . . . ,An ) : f (A0 ) = A00 , f (Ai ) = Ai pour
tout i = 1, . . . ,n. Cette transformation affine laisse invariant tout point de H.
Accueil
Géométrie affine
Convexité
Géométrie euclidienne
Isométries planes
Isométries de l’espace
Coniques
Géométrie analytique
Complexes
Homographies
Aide
Précédente
Suivante
Plein écran
Quitter
Retour
Suite
Affinités et transvections (2)
Accueil
Géométrie affine
Convexité
On suppose que A00 n’appartient pas à l’hyperplan affine H 0 parallèle à H passant
par A0 . Montrer que f est une affinité de base H dont on précisera la direction.
Géométrie euclidienne
Isométries planes
Isométries de l’espace
Coniques
Géométrie analytique
Complexes
Homographies
Aide
Précédente
Suivante
Plein écran
Quitter
Solution
Suite
Accueil
Géométrie affine
Soit A le point d’intersection de la droite A0 A00 avec H. L’affinité de base H, de diAA00
rection (A0 A00 ) et de rapport
et f sont deux applications affines qui coı̈ncident
AA0
sur le repère affine (A0 , . . . ,An ). Elles sont donc égales.
Convexité
Géométrie euclidienne
Isométries planes
Isométries de l’espace
Coniques
Géométrie analytique
Complexes
Homographies
Aide
Précédente
Suivante
Plein écran
Quitter
Retour
Suite
Affinités et transvections (3)
Accueil
Géométrie affine
Convexité
Géométrie euclidienne
On suppose maintenant que A00 appartient à l’hyperplan affine H 0 parallèle à H
passant par A0 . Montrer qu’il existe une fonction affine ϕ sur E nulle sur H telle
−−−−−→
−−−→
que M f (M ) = ϕ(M )A0 A00 pour tout point M de E.
Isométries planes
Isométries de l’espace
Coniques
Géométrie analytique
Complexes
Homographies
Aide
Précédente
Suivante
Plein écran
Quitter
Solution
Suite
Tout point M du plan s’écrit M = α0 A0 + α1 A1 + · · · + αn An avec
n
X
αi = 1. Son
i=0
Accueil
image f (M ) par l’application affine f s’écrit
Géométrie affine
Convexité
Géométrie euclidienne
Isométries planes
Isométries de l’espace
Coniques
Géométrie analytique
f (M ) = α0 f (A0 ) + α1 f (A1 ) + · · · + αn f (An ) = α0 A00 + α1 A1 + · · · + αn An .
−−−−−→
−−−→
−−−→
On en déduit M f (M ) = α0 A0 A00 = ϕ(M )A0 A00 , où ϕ est la forme affine qui associe
à tout point de E sa coordonnée suivant A0 dans le repère affine (A0 ,A1 , . . . ,An ),
i.e. l’unique application affine de E dans R vérifiant ϕ(A0 ) = 1, ϕ(Ai ) = 0 pour
tout i = 1, . . . ,n.
Complexes
Homographies
Aide
Précédente
Suivante
Plein écran
Quitter
Retour
Suite
Affinités et transvections (4)
Accueil
Géométrie affine
Convexité
On suppose que E est un plan. Donner une construction géométrique de l’image
M 0 = f (M ) d’un point M de E par f .
Géométrie euclidienne
Isométries planes
Isométries de l’espace
Coniques
Géométrie analytique
Complexes
Homographies
Aide
Précédente
Suivante
Plein écran
Quitter
Solution
Table
Accueil
Géométrie affine
Convexité
Géométrie euclidienne
Isométries planes
Si M n’appartient pas à H 0 , l’image par f de la droite A0 M est la droite A00 m,
où m est l’intersection de la droite A0 M avec H, puisque m est fixe par f . Le
point M 0 = f (M ) est donc l’intersection de la droite A00 m avec la parallèle à H
−−−−−→
−−−→
menée par M , puisque M f (M ) est proportionnel à A0 A00 . Si M appartient à H 0 ,
on commence par construire l’image par f d’un point n’appartenant pas à H 0 et
on fait une construction analogue à partir de ce point et de son image (on peut
−−−−−→ −−−→
aussi remarquer que dans ce cas M f (M ) = A0 A00 ).
Isométries de l’espace
Coniques
Géométrie analytique
Complexes
Homographies
Aide
Précédente
Suivante
Plein écran
Table
Quitter
Le théorème de Desargues
Accueil
Géométrie affine
Convexité
Géométrie euclidienne
Isométries planes
Soit, dans l’espace affine de dimension 3, OABC un tétraèdre et Π un plan coupant
les trois arêtes OA, OB et OC en A0 , B 0 et C 0 . On suppose les droites BC et B 0 C 0
(resp. CA et C 0 A0 , AB et A0 B 0 ) sécantes en des points α, β et γ. Montrer que les
trois points α, β et γ sont alignés.
Isométries de l’espace
Coniques
Géométrie analytique
Complexes
Homographies
Aide
Précédente
Suivante
Plein écran
Quitter
Indication
Solution
Considérez l’intersection des plans ABC et Π.
Accueil
Géométrie affine
Convexité
Géométrie euclidienne
Isométries planes
Isométries de l’espace
Coniques
Géométrie analytique
Complexes
Homographies
Aide
Précédente
Suivante
Plein écran
Quitter
Retour
Solution
Les plans ABC et Π se coupent suivant une droite ∆ et les points α, β et γ appartiennent tous trois à ∆.
Accueil
Géométrie affine
Convexité
Géométrie euclidienne
Isométries planes
Isométries de l’espace
Coniques
Géométrie analytique
Complexes
Homographies
Aide
Précédente
Suivante
Plein écran
Quitter
Remarque
Accueil
Géométrie affine
Convexité
On en déduit, en considérant la figure plane ci-dessous comme la projection sur
le plan ABC d’une figure de l’espace, que si deux triangles ABC et A0 B 0 C 0 d’un
même plan sont tels que les droites AA0 , BB 0 et CC 0 soient concourantes, alors les
points d’intersection des côtés BC et B 0 C 0 , CA et C 0 A0 , AB et A0 B 0 (s’ils existent)
sont alignés.
Géométrie euclidienne
Isométries planes
Isométries de l’espace
Coniques
Géométrie analytique
Complexes
Homographies
Aide
Précédente
Suivante
Plein écran
Quitter
Cas particulier
Table
Si les côtés BC et B 0 C 0 sont parallèles, et les côtés CA et C 0 A0 , AB et A0 B 0 sécants
en des points β et γ, alors la droite βγ est parallèle aux droites BC et B 0 C 0 .
Accueil
Géométrie affine
Convexité
Géométrie euclidienne
Isométries planes
Isométries de l’espace
Coniques
Géométrie analytique
Complexes
Homographies
Aide
Précédente
Suivante
Plein écran
Quitter
Si les côtés BC et B 0 C 0 , ainsi que les côtés CA et C 0 A0 , sont parallèles, alors les
côtés AB et A0 B 0 le sont aussi (pourquoi?). On retrouve alors le cas particulier du
théorème de Desargues considéré précédemment.
Table
Projection centrale, rapport et birapport
Accueil
Géométrie affine
Convexité
Géométrie euclidienne
Soit ∆ et ∆0 deux droites sécantes en O. Deux droites D et D0 coupent ∆ et ∆0 en
A, B, A0 , B 0 . On suppose les milieux I et I 0 des segments AB et A0 B 0 alignés avec
O. Montrer que les droites D et D0 sont parallèles.
Isométries planes
Isométries de l’espace
Coniques
Géométrie analytique
Complexes
Homographies
Aide
Précédente
Suivante
Plein écran
Quitter
Indication
Solution
Soit D00 l’image de D par l’homothétie de centre O qui transforme I en I 0 , A00 et B 00
ses points d’intersection avec ∆ et ∆0 . Que peut-on dire du quadrilatère A00 B 0 B 00 A0 ?
Accueil
Géométrie affine
Convexité
Géométrie euclidienne
Isométries planes
Isométries de l’espace
Coniques
Géométrie analytique
Complexes
Homographies
Aide
Précédente
Suivante
Plein écran
Quitter
Retour
Solution
Accueil
Géométrie affine
Convexité
Soit D00 l’image de D par l’homothétie de centre O qui transforme I en I 0 , A00 et B 00
ses points d’intersection avec ∆ et ∆0 . Les diagonales du quadrilatère A00 B 0 B 00 A0
se coupent en leurs milieux. Si ce quadrilatère n’était pas aplati, ce serait un parallélogramme et les droites ∆ et ∆0 seraient parallèles, ce qui n’est pas. On a donc
D00 = D0 , ce qui montre que D et D0 sont parallèles.
Géométrie euclidienne
Isométries planes
Isométries de l’espace
Coniques
Géométrie analytique
Complexes
Homographies
Aide
Précédente
Suivante
Plein écran
Quitter
Suite
Accueil
Géométrie affine
Convexité
Géométrie euclidienne
Isométries planes
Isométries de l’espace
Cet exercice montre en particulier que la projection centrale de centre O qui à un
point M du plan associe le point d’intersection de la droite OM avec une droite
fixée D0 n’est pas affine (elle ne conserve pas les milieux). On peut d’ailleurs remarquer que cette application n’est pas définie sur un espace affine, puisque l’image
d’un point de la parallèle à D0 passant par O n’est pas définie. La restriction de
cette application à une droite D ne passant pas par O est bien définie et est une
application affine dans le seul cas où D et D0 sont parallèles (c’est alors la restriction à D de l’unique homothétie de centre O qui transforme D en D0 ).
Coniques
Géométrie analytique
Complexes
La suite de l’exercice va montrer que la projection centrale conserve cependant
le birapport de quatre points.
Homographies
Aide
Précédente
Suivante
Plein écran
Quitter
Retour
Suite
Projection centrale et birapport
Accueil
Géométrie affine
Convexité
Géométrie euclidienne
Isométries planes
Soit A, B, C, D quatre points distincts d’une droite ∆ du plan affine P et O un
point de P n’appartenant pas à ∆. Montrer que
−→ −→
det(OA,OC)
AC
=
−→ −−→ .
AD
det(OA,OD)
Isométries de l’espace
Coniques
Géométrie analytique
Complexes
Homographies
Aide
Précédente
Suivante
Plein écran
Quitter
Solution
Suite
Accueil
Géométrie affine
Convexité
Géométrie euclidienne
→
−
−−→
Soit ~v un vecteur directeur de ∆, de sorte que M N = M N~v pour tout couple
(M,N ) de points de ∆. L’égalité cherchée résulte immédiatement des deux égalités :
−→ −→
−→ −→ −→
−→ −→
−→
det(OA,OC) = det(OA,OA + AC) = det(OA,AC) = AC det(OA,~v )
−→ −−→
−→ −→ −−→
−→ −−→
−→
det(OA,OD) = det(OA,OA + AD) = det(OA,AD) = AD det(OA,~v ) .
Isométries planes
Isométries de l’espace
Coniques
Géométrie analytique
Complexes
Homographies
Aide
Précédente
Suivante
Plein écran
Quitter
Retour
Suite
En déduire que si une autre droite ∆0 de P ne passant pas par O coupe les quatre
droites OA, OB, OC et OD en des points A0 , B 0 , C 0 et D0 , on a :
Accueil
Géométrie affine
Convexité
Géométrie euclidienne
AC BD
A0 C 0 B 0 D 0
·
= 0 0· 0 0
AD BC
AD BC
(conservation du birapport par les projections centrales).
Isométries planes
Isométries de l’espace
Coniques
Géométrie analytique
Complexes
Homographies
Aide
Précédente
Suivante
Plein écran
Quitter
Indication
Solution
−−→
−→ −−→
−−→
Il existe des réels α, β, γ, δ non nuls tels que OA0 = αOA, OB 0 = β OB,
−−→0
−→ −−→
−−→
OC = γ OC, OD0 = δ OD.
Accueil
Géométrie affine
Convexité
Géométrie euclidienne
Isométries planes
Isométries de l’espace
Coniques
Géométrie analytique
Complexes
Homographies
Aide
Précédente
Suivante
Plein écran
Quitter
Retour
Solution
Accueil
Géométrie affine
Convexité
Géométrie euclidienne
Isométries planes
Isométries de l’espace
Coniques
Géométrie analytique
Complexes
Homographies
Aide
−−→
−→
Les points O, A et A0 étant alignés, il existe un réel α non nul tel que OA0 = αOA.
−−→
−−→ −−→
−→ −−→
−−→
De même, il existe β, γ, δ non nuls tels que OB 0 = β OB, OC 0 = γ OC, OD0 = δ OD.
On en déduit :
−−→ −−→
−−→ −−→
A0 C 0 B 0 D 0
det(OA0 ,OC 0 ) det(OB 0 ,OD0 )
·
=
−−→ −−→ ·
−−→ −−→
A0 D 0 B 0 C 0
det(OA0 ,OD0 ) det(OB 0 ,OC 0 )
−→ −→
−−→ −−→
det(αOA,γ OC) det(β OB,δ OD)
=
−→ −−→ ·
−−→ −→
det(αOA,δ OD) det(β OB,γ OC)
−→ −→
−−→ −−→
det(OA,OC) det(OB,OD)
=
−→ −−→ ·
−−→ −→
det(OA,OD) det(OB,OC)
AC BD
·
.
=
AD BC
On en déduit qu’une projection centrale conserve le birapport de quatre points.
Précédente
Suivante
Plein écran
Quitter
Remarque
Table
Accueil
Géométrie affine
Convexité
Géométrie euclidienne
Isométries planes
Isométries de l’espace
Coniques
Géométrie analytique
AC
s’interprète alors
Supposons le plan muni d’un produit scalaire. Le quotient
AD
comme le rapport des aires des triangles OAC et OAD, puisque ces deux triangles
×
OC sin AOC
ont même hauteur. Il est donc encore égal au rapport
. La valeur
×
OD sin AOD
Ø
×
AC BD
sin BOD
sin AOC
absolue
et ne dépend
·
du birapport est donc égale à
×
×
AD BC
sin AOD
sin BOC
donc que des droites OA, OB, OC et OD, et non de la sécante ∆. Les égalités
établies précédemment ne font que refléter ces dernières sans avoir à se placer dans
le cadre euclidien.
Complexes
Homographies
Aide
Précédente
Suivante
Plein écran
Table
Quitter
Coordonnées barycentriques et déterminants
Accueil
Géométrie affine
Convexité
Géométrie euclidienne
Isométries planes
Soit ABC un triangle non aplati du plan affine E. Montrer que les aires algébriques
−−→ −−→
−−→ −−→
−−→ −−→
∆(M B,M C), ∆(M C,M A), ∆(M A,M B) des triangles M BC, M CA et M AB
constituent un système de coordonnées barycentriques du point M dans le repère
affine (A,B,C).
Isométries de l’espace
Coniques
Géométrie analytique
Complexes
Homographies
Aide
Précédente
Suivante
Plein écran
Quitter
Solution
Solution
Accueil
Géométrie affine
Convexité
Géométrie euclidienne
Isométries planes
Isométries de l’espace
Il suffit de montrer que :
−−→ −−→
−→ −→
det(M B,M C) = α det(AB,AC)
−−→ −−→
−→ −→
det(M C,M A) = β det(AB,AC)
−−→ −−→
−→ −→
det(M A,M B) = α det(AB,AC)
Coniques
Géométrie analytique
Complexes
Homographies
si (α,β,γ) sont les coordonnées barycentriques réduites de M dans le repère ABC,
−−→
−→
−→ −−→
ce qui se vérifie immédiatement en remarquant que AM = β AB + γ AC, BM =
−→
−−→ −−→
−→
−−→
αBA + γ BC, CM = αCA + β CB, et en développant les déterminants.
Aide
Précédente
Suivante
Plein écran
Quitter
Table
Aire algébrique d’un polygone : cas du triangle
Accueil
Géométrie affine
Convexité
−
→
Soit E un plan affine et (~i,~j) une base de E . On note ∆(~u,~v ) le déterminant de deux vecteurs ~u et
→
−
~v de E dans cette base.
Géométrie euclidienne
Isométries planes
Isométries de l’espace
Soit ABC un triangle de E. Montrer que la somme
−−→ −−→
−−→ −−→
−−→ −−→
∆(M A,M B) + ∆(M B,M C) + ∆(M C,M A)
Coniques
Géométrie analytique
Complexes
ne dépend pas du point M de E et l’exprimer en fonction de l’aire algébrique
−→ −→
∆(AB,AC) du triangle ABC.
Homographies
Aide
On remarquera que le signe de l’aire algébrique d’un triangle dépend de l’orientation de ce triangle, c’est-à-dire de l’ordre dans lequel sont écrits les sommets.
Précédente
Suivante
Plein écran
Quitter
Solution
Suite
Solution
Accueil
Pour tout couple M,N de points de E :
Géométrie affine
Convexité
Géométrie euclidienne
Isométries planes
Isométries de l’espace
Coniques
Géométrie analytique
Complexes
Homographies
−−→ −−→
−−→ −−→
−−→ −−→
∆(M A,M B) + ∆(M B,M C) + ∆(M C,M A)
−−→ −−→ −−→ −−→
−−→ −−→ −−→ −−→
−−→ −−→ −−→ −−→
= ∆(M N + N A,M N + N B) + ∆(M N + N B,M N + N C) + ∆(M N + N C,M N + N A)
−−→ −−→
−−→ −−→
−−→ −−→
= ∆(N A,N B) + ∆(N B,N C) + ∆(N C,N A)
−−→ −−→ −−→
+ ∆(M N ,N B − N A)
−−→ −−→ −−→
+ ∆(M N ,N C − N B)
−−→ −−→ −−→
+ ∆(M N ,N A − N C)
−−→ −−→
−−→ −−→
−−→ −−→
= ∆(N A,N B) + ∆(N B,N C) + ∆(N C,N A) .
En particulier, pour N = A, on obtient :
Aide
−−→ −−→
−−→ −−→
−−→ −−→
−→ −→
∆(M A,M B) + ∆(M B,M C) + ∆(M C,M A) = ∆(AB,AC) .
Précédente
Cette relation exprime simplement que l’aire algébrique du triangle ABC est la somme des aires
Suivante
algébriques des triangles M AB, M BC et M CA. Ces aires constituent également un système de
coordonnées barycentriques du point M dans le repère affine (A,B,C).
Plein écran
Quitter
Retour
Suite
Aire algébrique d’un polygone
Accueil
Géométrie affine
Convexité
Soit n un entier ≥ 2 et A1 , . . . An n points de E. Montrer que la somme
−−−→ −−−→
−−−→ −−−→
−−−−→ −−−→
−−−→ −−−→
∆(M A1 ,M A2 ) + ∆(M A2 ,M A3 ) + · · · + ∆(M An−1 ,M An ) + ∆(M An ,M A1 )
Géométrie euclidienne
Isométries planes
ne dépend pas du point M de E.
Isométries de l’espace
Coniques
On appellera cette somme aire algébrique du polygone A1 . . . An .
Géométrie analytique
On peut aisément vérifier que si le polygone est convexe, et si M est intérieur au polygone, ces
Complexes
aires sont toutes de même signe et donc que l’aire géométrique du polygone (i.e. la valeur absolue
Homographies
de l’aire algébrique) est la somme des aires géométriques des triangles M Ai Ai+1 et représente donc
l’aire du polygone au sens usuel.
Aide
Précédente
Suivante
Plein écran
Quitter
Solution
Table
Solution
Accueil
Géométrie affine
Convexité
Géométrie euclidienne
Isométries planes
Isométries de l’espace
Coniques
Géométrie analytique
Pour tout couple M,N de points de E :
−−→ −−−−→
−−→ −−→ −−→ −−−−→
∆(M Ai ,M Ai+1 ) = ∆(M N + N Ai ,M N + N Ai+1 )
−−→ −−−−→
−−→ −−−−→ −−→
= ∆(N Ai ,N Ai+1 ) + ∆(M N ,N Ai+1 − N Ai )
−−→ −−−−→
−−→ −−−−→
= ∆(N Ai ,N Ai+1 ) + ∆(M N ,Ai Ai+1 ) ,
d’où, en faisant la somme et en posant An+1 = A1 :
Complexes
n
X
Homographies
i=1
−−→ −−−−→
∆(M Ai ,M Ai+1 ) =
=
Aide
=
Précédente
=
Suivante
n
X
i=1
n
X
i=1
n
X
i=1
n
X
n
X
−−→ −−−−→
−−→ −−−−→
∆(N Ai ,N Ai+1 ) +
∆(M N ,Ai Ai+1 )
i=1
n −
−−−→
−−→ −−−−→
−−→ X
∆(N Ai ,N Ai+1 ) + ∆(M N , Ai Ai+1 )
i=1
−−→ −−−−→
−−→ −
→
∆(N Ai ,N Ai+1 ) + ∆(M N , 0 )
−−→ −−−−→
∆(N Ai ,N Ai+1 ) .
i=1
Plein écran
Quitter
Retour
Table
Aire d’un polygone convexe
Accueil
Géométrie affine
Convexité
Géométrie euclidienne
Isométries planes
Isométries de l’espace
Le polygone A1 . . . An est convexe si pour tout entier i = 1, . . . ,n, il est situé tout
entier d’un même côté de la droite Ai Ai+1 . Si le point M est intérieur au polygone,
ses coordonnées barycentriques réduites (αi ,βi ,γi ) dans le repère (Ai−1 ,Ai ,Ai+1 )
vérifient αi > 0, βi < 0, γi > 0. Il en résulte que les aires algébriques des triangles
orientés M Ai−1 Ai et M Ai Ai+1 sont de même signe.
Coniques
Géométrie analytique
Complexes
Homographies
Aide
Précédente
Suivante
Plein écran
Quitter
Retour
Action du groupe affine sur les triplets de droites (1)
Accueil
Géométrie affine
Convexité
Géométrie euclidienne
Isométries planes
Soient ∆1 , ∆2 , ∆3 (resp. ∆01 , ∆02 , ∆03 ) trois droites d’un plan affine P en position générale (i.e. deux à deux sécantes et d’intersections distinctes). Montrer qu’il
existe une transformation affine f de P dans P et une seule qui vérifie f (∆i ) = ∆0i
pour i = 1,2,3.
Isométries de l’espace
Coniques
Géométrie analytique
Complexes
Homographies
Aide
Précédente
Suivante
Plein écran
Quitter
Indication
Solution
Que peut-on dire de l’image des points A, B et C ?
Accueil
Géométrie affine
Convexité
Géométrie euclidienne
Isométries planes
Isométries de l’espace
Coniques
Géométrie analytique
Complexes
Homographies
Aide
Précédente
Suivante
Plein écran
Quitter
Retour
Solution
Accueil
Géométrie affine
Convexité
Soient A, B et C les points d’intersection de ces droites deux à deux. Une transformation affine f transforme ∆1 en ∆01 , ∆2 en ∆02 et ∆3 en ∆03 si et seulement si
f (A) = A0 , f (B) = B 0 , f (C) = C 0 . Or (A,B,C) est un repère affine du plan : il
existe donc une application affine f et une seule qui vérifie ces conditions et elle
est bijective.
Géométrie euclidienne
Isométries planes
Isométries de l’espace
Coniques
Géométrie analytique
Complexes
Homographies
Aide
Précédente
Suivante
Plein écran
Quitter
Suite
Action du groupe affine sur les triplets de droites (2)
Accueil
Géométrie affine
Convexité
Géométrie euclidienne
On suppose maintenant ∆1 , ∆2 et ∆3 , concourantes en un point A. Existe-t-il toujours une transformation affine f qui vérifie f (∆i ) = ∆0i pour i = 1,2,3 ? Si oui,
est-elle unique?
Isométries planes
Isométries de l’espace
Coniques
Géométrie analytique
Complexes
Homographies
Aide
Précédente
Suivante
Plein écran
Quitter
Indication
Solution
Observez la figure...
Accueil
Géométrie affine
Convexité
Géométrie euclidienne
Isométries planes
Isométries de l’espace
Coniques
Géométrie analytique
Complexes
Homographies
Aide
Précédente
Suivante
Plein écran
Quitter
Retour
Solution
Accueil
Géométrie affine
Convexité
Géométrie euclidienne
Isométries planes
Isométries de l’espace
Coniques
Soient B et C (resp. B 0 et C 0 ) les points d’intersection d’une parallèle à ∆1 avec
∆2 et ∆3 (resp. ∆02 et ∆03 ). Il existe une transformation affine f et une seule qui
transforme le repère affine (A,B,C) en le repère affine (A0 ,B 0 ,C 0 ). Cette application
transforme les droites ∆2 et ∆3 en les droites ∆02 et ∆03 . Comme toute transformation affine conserve le parallélisme, elle transforme la droite ∆1 en une droite
parallèle à la droite B 0 C 0 passant par A0 , c’est-à-dire en la droite ∆01 .
Mais une transformation qui vérifie f (∆i ) = ∆0i pour i = 1,2,3 n’est pas unique :
toute transformation affine h ◦ f , où h est une homothétie quelconque de centre
A0 , possède aussi cette propriété.
Géométrie analytique
Complexes
Homographies
Aide
Précédente
Suivante
Plein écran
Quitter
Suite
Action du groupe affine sur les triplets de droites (3)
Accueil
Géométrie affine
Convexité
Géométrie euclidienne
Isométries planes
Soient ∆1 , ∆2 , ∆3 (resp. ∆01 , ∆02 , ∆03 ) trois droites de l’espace affine S de dimension
3. On suppose que deux quelconques de ces droites ne sont jamais coplanaires.
Montrer qu’il existe une transformation affine f de S dans S et une seule qui
vérifie f (∆i ) = ∆0i pour i = 1,2,3.
Isométries de l’espace
Coniques
Géométrie analytique
Complexes
Homographies
Aide
Précédente
Suivante
Plein écran
Quitter
Indication
Solution
Montrez qu’il existe un parallélépipède ABCDEF GH et un seul qui s’appuie sur
ces trois droites (voir figure).
Accueil
Géométrie affine
Convexité
Géométrie euclidienne
Isométries planes
Isométries de l’espace
Coniques
Géométrie analytique
Complexes
Homographies
Aide
Précédente
Suivante
Plein écran
Quitter
Retour
Solution
Accueil
Géométrie affine
Convexité
Géométrie euclidienne
Isométries planes
Isométries de l’espace
Coniques
Montrons qu’il existe un parallélépipède ABCDEF GH et un seul qui s’appuie sur
∆1 , ∆2 , ∆3 (voir figure). En effet le point A est obtenu comme intersection de
∆1 et du plan passant par ∆3 dont la direction contient celle de ∆2 . Il est donc
uniquement déterminé. De même pour les autres points.
Soit A0 B 0 C 0 D0 E 0 F 0 G0 H 0 un parallélépipède s’appuyant de même sur ∆01 , ∆02 , ∆03 .
Une transformation affine f de S transforme ∆i en ∆0i pour i = 1,2,3 si et seulement
si elle transforme ABCDEF GH en A0 B 0 C 0 D0 E 0 F 0 G0 H 0 . Il faut et il suffit pour cela
qu’elle transforme le repère affine (A,B,E,D) de S en le repère affine (A0 ,B 0 ,E 0 ,D0 ).
Or il existe une et une seule transformation affine qui vérifie cette propriété.
Géométrie analytique
Complexes
Homographies
Aide
Précédente
Suivante
Plein écran
Quitter
Retour
Table
Demi-espaces
Accueil
Géométrie affine
Convexité
Géométrie euclidienne
Soit E un espace affine de dimension n et H un hyperplan affine de E. Montrer
que la relation R définie sur E \ H par
M R N si et seulement si [M N ] ∩ H = ∅
Isométries planes
Isométries de l’espace
est une relation d’équivalence qui sépare E \ H en exactement deux classes.
Coniques
Géométrie analytique
Complexes
Homographies
Aide
Précédente
Suivante
Plein écran
Quitter
Indication
Solution
→
−
Considérez un repère cartésien (O,~e1 , . . . ,~en ), avec O ∈ H et ~e1 , . . . ,~en−1 ∈ H .
Accueil
Géométrie affine
Convexité
Géométrie euclidienne
Isométries planes
Isométries de l’espace
Coniques
Géométrie analytique
Complexes
Homographies
Aide
Précédente
Suivante
Plein écran
Quitter
Solution
Accueil
Géométrie affine
Convexité
Géométrie euclidienne
Isométries planes
Isométries de l’espace
Coniques
Géométrie analytique
Complexes
→
−
Soit (O,~e1 , . . . ,~en ) un repère cartésien de E, avec O ∈ H et ~e1 , . . . ,~en−1 ∈ H ,
(M )
(N )
)
)
x1 , . . . ,x(M
(resp. x1 , . . . ,x(N
n
n ) les coordonnées de M (resp. N ) dans ce repère.
)
)
Les points M et N sont en relation si et seulement si x(M
et x(N
sont de même
n
n
signe. En effet tout point du segment [M N ] s’écrit αM + (1 − α)N avec α ∈ [0,1] et
(N )
)
= 0, autrement dit si
ce point appartient à H si et seulement si αx(M
n + (1 − α)xn
(M )
(N )
0 appartient au segment de R d’extrémités xn et xn . On vérifie sans peine que
)
)
la relation “ x(M
et x(N
sont de même signe ” induit une relation d’équivalence
n
n
sur E \ H.
On dit que deux points équivalents sont du même côté de H et les deux classes
d’équivalence sont appelées demi-espaces (ouverts) délimités par H.
Homographies
Aide
Précédente
Suivante
Plein écran
Quitter
Table
Régionnement du plan par un repère affine
Accueil
Géométrie affine
Convexité
Géométrie euclidienne
Soit A, B, C un repère affine du plan. Caractériser en termes de coordonnées
barycentriques réduites dans ce repère les sept régions du plan délimitées par les
droites AB, BC et CA.
Isométries planes
Isométries de l’espace
Coniques
Géométrie analytique
Complexes
Homographies
Aide
Précédente
Suivante
Plein écran
Quitter
Solution
Accueil
Géométrie affine
Convexité
Géométrie euclidienne
Soit (α, β, γ) les coordonnées barycentriques réduites d’un point M = αA+βB+γC
(α+β +γ = 1) dans le repère affine A, B, C. La droite BC a pour équation α = 0 et
partage le plan en deux demi-plans : celui qui contient A est caractérisé par α > 0
et l’autre par α < 0. Les droites AB, BC et CA divisent ainsi le plan en 7 régions
ouvertes caractérisées par les 7 triplets de signes possibles pour (α, β, γ) (ces trois
nombres ne peuvent être simultanément négatifs, puisque leur somme vaut 1).
Isométries planes
Isométries de l’espace
Coniques
Géométrie analytique
Complexes
Homographies
Aide
Précédente
Suivante
Plein écran
Quitter
Suite
En combien de régions les plans portant les faces d’un tétraèdre non aplati partagentils l’espace?
Accueil
Géométrie affine
Convexité
Géométrie euclidienne
Isométries planes
Isométries de l’espace
Coniques
Géométrie analytique
Complexes
Homographies
Aide
Précédente
Suivante
Plein écran
Quitter
Indication
Solution
Les sommets du tétraèdre constituent un repère affine de l’espace. Chacune des
régions délimitées par les plans portant les faces est caractérisée par le quadruplet
des signes des coordonnées barycentriques réduites d’un point dans ce repère.
Accueil
Géométrie affine
Convexité
Géométrie euclidienne
Isométries planes
Isométries de l’espace
Coniques
Géométrie analytique
Complexes
Homographies
Aide
Précédente
Suivante
Plein écran
Quitter
Solution
Accueil
Géométrie affine
Convexité
Géométrie euclidienne
Isométries planes
Isométries de l’espace
Les sommets du tétraèdre constituent un repère affine de l’espace. Chacune des
régions délimitées par les plans portant les faces est caractérisée par le quadruplet
des signes des coordonnées barycentriques réduites d’un point dans ce repère. Il y
a donc 15 régions, correspondant aux 15 quadruplets de signes possibles (24 − 1,
puisqu’ici encore les coordonnées barycentriques réduites ne peuvent être simultanément négatives). (On peut distinguer ainsi 4 types de régions selon le nombre
de signes + et de signes - parmi les coordonnées barycentriques : faites une figure
et distinguez ces régions).
Coniques
Géométrie analytique
Complexes
Homographies
Aide
Précédente
Suivante
Plein écran
Quitter
Table
Cônes convexes
Accueil
Géométrie affine
Convexité
Géométrie euclidienne
Soit C un convexe d’un espace affine E et O un point de E. Montrer que la réunion
Γ des demi-droites fermées d’origine O passant par M , pour M décrivant C, est
un convexe.
Isométries planes
Isométries de l’espace
Coniques
Géométrie analytique
Complexes
Homographies
Aide
Précédente
Suivante
Plein écran
Quitter
Solution
Accueil
Géométrie affine
Convexité
Géométrie euclidienne
Isométries planes
Isométries de l’espace
Coniques
Géométrie analytique
Soient M et N deux points de Γ et P un point du segment [M N ]. Il existe deux
−−→
−−→
réels λ ≥ 0 et µ ≥ 0 et deux points M1 et N1 de C tels que OM = λOM1 et
−−→
−−→
ON = µON1 , ainsi qu’un réel α ∈ [0,1] tel que P = αM + (1 − α)N , d’où :
−→
−−→
−−→
−−→
−−→
OP = αOM + (1 − α)ON = αλOM1 + (1 − α)µON1 .
Si αλ = (1 − α)µ = 0, on a P = O. Sinon
le réel ν = αλ + (1 − α)µ
est strictement
„
Ž
−→
αλ −−→ (1 − α)µ −−→
positif. On peut alors écrire : OP = ν
OM1 +
ON , soit en posant
ν
ν
−→
−−→
αλ
α1 =
et P1 = α1 M1 + (1 − α1 )N1 , OP = ν OP1 . Le point P1 appartient à C,
ν
puisque α1 appartient à [0,1] et C est convexe : il en résulte que P appartient à Γ.
Complexes
Homographies
Aide
Précédente
Suivante
Plein écran
Table
Quitter
Quadrilatères
Accueil
Géométrie affine
Convexité
Géométrie euclidienne
Isométries planes
Soient A, B, C, D quatre points trois à trois non alignés du plan affine P . Montrer
qu’il existe quatre réels non nuls α, β, γ, δ, de somme nulle, tels que le vecteur
−→
−−→
−→
−−→
αOA + β OB + γ OC + δ OD soit nul pour tout point O de P , et que ces nombres
sont uniques à multiplication près par un même scalaire non nul.
Isométries de l’espace
Coniques
Géométrie analytique
Complexes
Homographies
Aide
Précédente
Suivante
Plein écran
Quitter
Solution
Suite
Accueil
Géométrie affine
Convexité
Géométrie euclidienne
Isométries planes
Isométries de l’espace
Coniques
−→
−−→
−→
−−→
Si α + β + γ + δ = 0, le vecteur αOA + β OB + γ OC + δ OD ne dépend pas du point
−→
−→
−−→
O. Il est en particulier égal à β AB + γ AC + δ AD. Les points A, B, C n’étant pas
−→ −→
→
−
alignés, les vecteurs AB et AC forment une base du plan vectoriel P . Il existe donc
−−→
−→
−→
deux réels λ et µ tels que AD = λAB + µAC. Ces réels vérifient λ 6= 0 (sinon A, C
et D seraient alignés), µ 6= 0 (sinon A, B et D seraient alignés), λ + µ 6= 1 (sinon
B, C et D seraient alignés). Le quadruplet (1 − λ − µ, λ, µ, − 1) convient donc et
il est unique à multiplication près par un réel non nul, puisque la décomposition
−−→
−→ −→
du vecteur AD dans la base (AB,AC) est unique.
Géométrie analytique
Complexes
Homographies
Aide
Précédente
Suivante
Plein écran
Quitter
Suite
Quadrilatères (suite)
Accueil
Géométrie affine
Convexité
En déduire, en examinant les signes de ces nombres, que tout quadrilatère est de
l’un des trois types suivants :
Géométrie euclidienne
Isométries planes
A
B
D
Isométries de l’espace
D
Coniques
A
Géométrie analytique
Complexes
Homographies
Aide
D
A
C
C
B
C
B
Précédente
Suivante
Plein écran
Quitter
Solution
Accueil
Géométrie affine
Convexité
Géométrie euclidienne
Isométries planes
Les nombres α, β, γ et δ étant de somme nulle, ils ne peuvent être tous de même
signe. À permutation circulaire et à multiplication par -1 près, il reste donc trois
possibilités pour leurs signes: (+, + , + , −), (+, − , + , −), (+, + , − , −).
Le premier cas correspond à la première figure (un point à l’intérieur du triangle
formé par les trois autres : quadrilatère ni convexe ni croisé), le second à la seconde
figure (les diagonales AC et BD se coupent : quadrilatère convexe), et le dernier à
la dernière figure (les côtés AB et CD se coupent : quadrilatère croisé).
Isométries de l’espace
A
B
Coniques
D
Géométrie analytique
D
Complexes
A
Homographies
D
Aide
A
Précédente
C
C
B
C
B
Suivante
Plein écran
Quitter
Table
Diagonales d’un polygone convexe
Accueil
Géométrie affine
Convexité
Géométrie euclidienne
Déterminer le nombre de points d’intersection des diagonales d’un polygone convexe
A1 A2 . . . An intérieurs à ce polygone. On supposera que trois diagonales ne sont jamais concourantes.
Isométries planes
Isométries de l’espace
Coniques
Géométrie analytique
Complexes
Homographies
Aide
Précédente
Suivante
Plein écran
Quitter
Indication
Solution
Combien de sommets faut-il pour déterminer un tel point d’intersection?
Accueil
Géométrie affine
Convexité
Géométrie euclidienne
Isométries planes
Isométries de l’espace
Coniques
Géométrie analytique
Complexes
Homographies
Aide
Précédente
Suivante
Plein écran
Quitter
Retour
Solution
Accueil
Géométrie affine
Convexité
Géométrie euclidienne
Quatre sommets distincts déterminent un unique quadrilatère convexe et donc
un unique point d’intersection intérieur au polygone. Réciproquement tout point
d’intersection détermine uniquement 4 sommets distincts (les extrémités des diagonales dont il constitue l’intersection). Il y a donc„autant
de points que de parties
Ž
n
à 4 éléments dans un ensemble à n éléments, i.e.
.
4
Isométries planes
Isométries de l’espace
Coniques
Géométrie analytique
Complexes
Homographies
Aide
Précédente
Suivante
Plein écran
Quitter
Table
Ensemble des milieux
Accueil
Géométrie affine
Convexité
Géométrie euclidienne
Soient C et C 0 deux convexes non vides d’un espace affine E. Montrer que l’ensemble
Γ des milieux des segments M M 0 , où M parcourt C et M 0 parcourt C 0 , est un
convexe.
Isométries planes
Isométries de l’espace
Coniques
Géométrie analytique
Complexes
Homographies
Aide
Précédente
Suivante
Plein écran
Quitter
Solution
Suite
Accueil
Géométrie affine
Convexité
Il suffit de montrer que si M1 et M2 (resp. M10 et M20 ) sont deux points de C (resp.
C 0 ), le point
!
!
1
1 0
1 0
1
α M1 + M1 + (1 − α) M2 + M2
2
2
2
2
appartient à Γ pour tout α ∈ [0,1]. Mais ce point s’écrit aussi :
Géométrie euclidienne
Isométries planes
Isométries de l’espace
Coniques
Géométrie analytique
1
1
(αM1 + (1 − α)M2 ) + (αM10 + (1 − α)M20 )
2
2
et αM1 + (1 − α)M2 (resp. αM10 + (1 − α)M20 ) appartient à C (resp. C 0 ) puisque ces
ensembles sont convexes.
Complexes
Homographies
Aide
Précédente
Suivante
Plein écran
Quitter
Retour
Suite
Milieux (suite)
Accueil
Géométrie affine
Décrire Γ quand C et C 0 sont deux segments AB et CD.
Convexité
Géométrie euclidienne
Isométries planes
Isométries de l’espace
Coniques
Géométrie analytique
Complexes
Homographies
Aide
Précédente
Suivante
Plein écran
Quitter
Retour
Solution
Accueil
Géométrie affine
Convexité
Géométrie euclidienne
Isométries planes
Isométries de l’espace
Coniques
Géométrie analytique
Complexes
Homographies
Aide
Précédente
Suivante
Les milieux E, F , G, H des segments
AC, BC, BD et BC forment un pa−−→ −→
rallélogramme, puisque HG = EF =
1 −→
AB. Ces quatre points, ainsi donc
2
que leur enveloppe convexe Γ0 (le parallélogramme plein) sont donc inclus dans Γ. Pour montrer que tout
1
1
point K = M + M 0 , avec M =
2
2
αA+(1−α)B et M 0 = γC +(1−γ)D
(0 ≤ α,γ ≤ 1) appartient à Γ0 , il suffit
d’écrire :
!
!
1 0
1
1 0
1
K = α A + M + (1 − α) B + M
2
2
2
2
!
!!
1
1
1
1
= α γ A + C + (1 − γ) A + D
2
2
2
2
!
!!
1
1
1
1
+(1 − α) γ B + C + (1 − γ) B + D
2
2
2
2
= αγE + α(1 − γ)H + (1 − α)γF + (1 − α)(1 − γ)G .
Plein écran
Quitter
Retour
Table
Une propriété des triangles
Accueil
Géométrie affine
Convexité
Géométrie euclidienne
Isométries planes
Isométries de l’espace
Coniques
Géométrie analytique
Soit, dans le plan affine E, T un triangle et T 0 un triangle image de T par une
translation ou une homothétie de rapport positif. Montrer que l’intersection de T
et de T 0 est :
• soit vide ;
• soit réduite à un point ;
• soit un triangle image de T par une translation ou une homothétie de rapport
positif.
Complexes
Homographies
Aide
Précédente
Suivante
Plein écran
Quitter
Indication
Solution
−→ −→
Considérez un repère cartésien (A,AB,AC), où A, B, C sont les sommets du triangle et écrivez des inégalités caractérisant les points de T et de T 0 .
Accueil
Géométrie affine
Convexité
Géométrie euclidienne
Isométries planes
Isométries de l’espace
Coniques
Géométrie analytique
Complexes
Homographies
Aide
Précédente
Suivante
Plein écran
Quitter
Solution
Accueil
Géométrie affine
Convexité
Géométrie euclidienne
Isométries planes
Isométries de l’espace
Coniques
Géométrie analytique
Complexes
Soient A, B, C les sommets du triangle. Un point M de coordonnées (x,y) dans
−→ −→
le repère cartésien (A,AB,AC) appartient à T si et seulement si x ≥ 0, y ≥ 0,
x + y ≤ 1. Il existe deux réels réels a et b et un réel c > 0 tels que le triangle T 0
est caractérisé par les inégalités x ≥ a, y ≥ b, (x − a) + (y − b) ≤ c. L’intersection
T ∩ T 0 est alors caractérisée par les inégalités x ≥ max(a,0), y ≥ max(b,0), x + y ≤
min(a + b + c,1). Elle est donc :
• vide si max(a,0) + max(b,0) > min(a + b + c,1) ;
• réduite à un point si max(a,0) + max(b,0) = min(a + b + c,1) ;
• un triangle image de T par une translation ou une homothétie de rapport
positif si max(a,0) + max(b,0) < min(a + b + c,1).
Homographies
Aide
Précédente
Suivante
Plein écran
Quitter
Table
Le théorème de Carathéodory (1)
Accueil
Géométrie affine
Convexité
Géométrie euclidienne
Soient A0 , A1 , · · · ,Am m + 1 points affinement dépendants d’un espace affine E.
Montrer qu’il existe m + 1 réels α0 , α1 , · · · ,αm de somme nulle, non tous nuls, tels
m
−−→
P
que le vecteur
αi OAi soit nul pour tout point O de E.
i=0
Isométries planes
Isométries de l’espace
Coniques
Géométrie analytique
Complexes
Homographies
Aide
Précédente
Suivante
Plein écran
Quitter
Solution
Suite
Accueil
Géométrie affine
Convexité
Géométrie euclidienne
Isométries planes
Dire que les points A0 ,A1 , · · · ,Am sont affinement dépendants signifie que le système
−−−→
−−−→
(A0 A1 , . . . ,A0 Am ) est lié. Il existe donc m réels α1 , · · · ,αm non tous nuls tels que
m
m
m
−−−→
P
P
P
le vecteur
αi A0 Ai soit nul. Posons α0 = − αi . Comme
αi = 0, le vecteur
i=1
i=1
i=0
m
−−→
P
αi OAi ne dépend pas du point O de E ; en prenant O = A0 , on voit qu’il est
i=0
m
−−−→
P
égal à
αi A0 Ai , qui est nul.
i=1
Isométries de l’espace
Coniques
Géométrie analytique
Complexes
Homographies
Aide
Précédente
Suivante
Plein écran
Quitter
Suite
Le théorème de Carathéodory (2)
Accueil
Géométrie affine
Convexité
Géométrie euclidienne
En déduire que tout barycentre à coefficients tous positifs de m+1 points A0 , A1 , · · · ,Am
affinement dépendants d’un espace affine E peut s’écrire comme barycentre à coefficients tous positifs de m de ces points.
Isométries planes
Isométries de l’espace
Coniques
Géométrie analytique
Complexes
Homographies
Aide
Précédente
Suivante
Plein écran
Quitter
Solution
Suite
Soit M un barycentre à coefficients tous positifs des m + 1 points A0 , A1 , · · · ,Am :
Accueil
m
X
Géométrie affine
i=0
Convexité
Géométrie euclidienne
−−→ −
→
λi M Ai = 0 ,
Coniques
λi > 0.
i=0
Ces points étant affinement dépendants, il existe m + 1 réels α0 , α1 , · · · , αm de
m
−−→
P
somme nulle, non tous nuls, tels que le vecteur
αi M Ai soit nul, d’où :
i=0
Isométries planes
Isométries de l’espace
λi ≥ 0 pour tout i = 1, . . . , m,
m
X
m
X
−−→ −
→
(λi − tαi )M Ai = 0 pour tout réel t.
i=0
Géométrie analytique
Complexes
Homographies
Aide
Précédente
Soit I l’ensemble des indices i tels que αi soit strictement positif (cet ensemble n’est
λi
pas vide, puisque les αi sont de somme nulle et ne sont pas tous nuls) et t = min .
i∈I αi
P
P
m
m
On a alors λi − tαi ≥ 0 pour tout i = 0, . . . ,m, i=0 (λi − tαi ) = i=0 λi > 0 et
λj
λj − tαj = 0 si t =
. Le point M s’écrit donc comme barycentre à coefficients
αj
tous positifs des points Ai , i 6= j.
Suivante
Plein écran
Quitter
Suite
Le théorème de Carathéodory (3)
Accueil
Géométrie affine
Convexité
Géométrie euclidienne
En déduire le théorème de Carathéodory : tout point de l’enveloppe convexe d’une
partie d’un espace affine de dimension n est barycentre à coefficients tous positifs
de n + 1 points de cette partie.
Isométries planes
Isométries de l’espace
Coniques
Géométrie analytique
Complexes
Homographies
Aide
Précédente
Suivante
Plein écran
Quitter
Solution
Accueil
Géométrie affine
Convexité
Géométrie euclidienne
Isométries planes
On sait que l’enveloppe convexe d’une partie A de E est l’ensemble des barycentres
à coefficients tous positifs de points de A. Tout point M de cette enveloppe convexe
s’écrit donc comme barycentre à coefficients tous positifs de m points de A, pour
un certain entier m. Comme m points d’un espace affine de dimension n sont
affinement dépendants si m > n + 1, un raisonnement par récurrence immédiat
permet de montrer que M s’écrit comme barycentre à coefficients tous positifs
d’au plus n + 1 de ces points.
Isométries de l’espace
Coniques
Géométrie analytique
Complexes
Homographies
Aide
Précédente
Suivante
Plein écran
Quitter
Application
Application
Accueil
Géométrie affine
Convexité
En déduire que l’enveloppe convexe d’un compact non vide d’un espace affine de
dimension finie est compacte.
Géométrie euclidienne
Isométries planes
Isométries de l’espace
Coniques
Géométrie analytique
Complexes
Homographies
Aide
Précédente
Suivante
Plein écran
Quitter
Solution
Soit K un compact non vide de E. Notons
Accueil
n
Σn = {(λ0 , . . . , λn ) ∈ [0,1] |
et ϕ l’application de K n+1 × Σn dans E définie par
Géométrie euclidienne
Isométries planes
Isométries de l’espace
Coniques
Géométrie analytique
Complexes
Homographies
λi = 1}
i=0
Géométrie affine
Convexité
m
X
ϕ(A0 , . . . ,An ,λ0 , . . . ,λn ) =
n
X
λ i Ai .
i=0
Le théorème de Carathéodory implique que l’enveloppe convexe de K est l’image
de ϕ. Mais ϕ est continue et K n+1 × Σn est compact. L’image d’un compact par
une application continue étant compacte, il en résulte que l’enveloppe convexe de
K est compacte.
Aide
Précédente
Remarque : L’enveloppe convexe d’un fermé n’est par contre pas nécessairement
fermée. Déterminez, pour vous en convaincre, l’enveloppe convexe de la réunion
d’une droite D et d’un point O n’appartenant pas à cette droite.
Suivante
Plein écran
Quitter
Solution
L’enveloppe convexe est la bande du plan comprise entre la droite D et la parallèle
à D passant par O, cette dernière droite exclue à l’exception du point O. Cette
enveloppe convexe n’est donc pas fermée, alors que la réunion de D et de O l’est.
Accueil
Géométrie affine
Convexité
Géométrie euclidienne
Isométries planes
Isométries de l’espace
Coniques
Géométrie analytique
Complexes
0
Homographies
D
Aide
Précédente
Suivante
Plein écran
Quitter
Table
Projection sur un convexe fermé
Accueil
Géométrie affine
Convexité
Géométrie euclidienne
Soit C un convexe fermé non vide d’un espace affine euclidien E. Montrer que pour
tout point M de E il existe un unique point P de C réalisant la distance de M à
C, i.e. vérifiant M P = d(M,C) = inf{M Q | Q ∈ C}.
Isométries planes
Isométries de l’espace
Coniques
Géométrie analytique
Complexes
Homographies
Aide
Précédente
Suivante
Plein écran
Quitter
Solution
Accueil
Géométrie affine
Convexité
Existence :
Soit N un point de C et K l’intersection de C et de la boule fermée de centre M
et de rayon M N . On a clairement inf{M Q | Q ∈ C} = inf{M Q | Q ∈ K}. Mais
K est compact et la fonction Q 7→ M Q est continue. Cette fonction atteint donc
sa borne inférieure sur K.
Géométrie euclidienne
Isométries planes
Isométries de l’espace
Coniques
Géométrie analytique
Unicité :
Supposons qu’il existe deux points distincts P1 et P2 de C vérifiant M P1 = M P2 =
inf{M Q | Q ∈ C}. Le triangle M P1 P2 serait isocèle en M et le milieu I de P1 P2
appartiendrait à C, puisque C est convexe, et vérifierait
Complexes
Homographies
Aide
M I 2 = M P12 − IP12 < M P12 ,
ce qui contredit l’hypothèse. Le point P de C vérifiant M P = d(M,C) = inf{M Q |
Q ∈ C} est donc unique. On l’appelle projeté de M sur le convexe C.
Précédente
Suivante
Plein écran
Quitter
Suite
−−→ −→
Montrer que le projeté P de M sur C est l’unique point de C vérifiant P M · P Q ≤ 0
pour tout point Q de C.
Accueil
Géométrie affine
Convexité
Géométrie euclidienne
Isométries planes
Isométries de l’espace
Coniques
Géométrie analytique
Complexes
Homographies
Aide
Précédente
Suivante
Plein écran
Quitter
Indication
Solution
Accueil
Géométrie affine
Convexité
Géométrie euclidienne
Soit Q un point de C et R un point du segment [P Q]. Il existe donc ε ∈ [0,1] tel que
−→
−→
P R = εP Q. En utilisant l’inégalité M P 2 ≤ M R2 et en faisant varier ε, montrez
−−→ −→
que P M · P Q ≤ 0. Pour démontrer l’unicité du point P vérifiant cette relation,
−−→ −−→
supposez que P1 et P2 la vérifient et exprimez P1 P22 en fonction de P1 M · P1 P2 et
−−→ −−→
P2 M · P2 P1 . Que peut-on dire du signe de ces produits scalaires?
Isométries planes
Isométries de l’espace
Coniques
Géométrie analytique
Complexes
Homographies
Aide
Précédente
Suivante
Plein écran
Quitter
Solution
Accueil
Géométrie affine
Convexité
Géométrie euclidienne
Isométries planes
Soit Q un point de C et R un point du segment [P Q]. Il existe donc ε ∈ [0,1] tel
−→
−→
que P R = εP Q. Comme R appartient à C, on a :
−−→ −→
−−→ −→
M P 2 ≤ M R2 = (M P + P R)2 = M P 2 + 2εM P · P Q + ε2 P Q2 ,
−−→ −→
−−→ −→
d’où 0 ≤ 2εM P · P Q + ε2 P Q2 et, pour ε > 0, 0 ≤ 2M P · P Q + εP Q2 . En faisant
−−→ −→
−−→ −→
tendre ε vers 0, on obtient 0 ≤ M P · P Q, i.e. P M · P Q ≤ 0.
Isométries de l’espace
Coniques
Géométrie analytique
Complexes
Homographies
−−→ −−→
−−→ −−→
Soient P1 et P2 deux points de C vérifiant P1 M · P1 Q ≤ 0 (resp. P2 M · P2 Q ≤ 0)
pour tout point Q de C. En prenant Q = P2 (resp. Q = P1 ) dans cette inégalité,
−−→ −−→
−−→ −−→
on obtient P1 M · P1 P2 ≤ 0 et P2 M · P2 P1 ≤ 0, d’où, en faisant la somme
−−→ −−→ −−→
P1 P22 = (P1 M + M P2 ) · P1 P2 ≤ 0
Aide
i.e. P1 = P2 .
Précédente
Suivante
Plein écran
Quitter
Suite
Montrer que la projection p sur un convexe fermé C réduit les distances :
p(M )p(N ) ≤ M N pour tout couple (M,N ) de points de E.
Accueil
Géométrie affine
Convexité
Géométrie euclidienne
Donner une condition nécessaire et suffisante sur le convexe C pour que p soit
affine.
Isométries planes
Isométries de l’espace
Coniques
Géométrie analytique
Complexes
Homographies
Aide
Précédente
Suivante
Plein écran
Quitter
Solution
Accueil
Géométrie affine
Convexité
Géométrie euclidienne
Isométries planes
Isométries de l’espace
Coniques
Géométrie analytique
Soient M1 , M2 deux points de E et P1 , P2 leurs projetés sur C. Les inégalités
−−−→ −−→
−−→ −−−→
M1 P1 · P1 P2 ≥ 0 et P1 P2 · P2 M2 ≥ 0 (appliquez la propriété précédente avec Q = P2
et Q = P1 ) montrent que :
−−−→ −−→ −−−→
M1 M22 = (M1 P1 + P1 P2 + P2 M2 )2
−−−→ −−−→
≥ M1 P12 + P1 P22 + P2 M22 + 2M1 P1 · P2 M2
−−−→ −−−→
≥ (M1 P1 + P2 M2 )2 + P1 P22
≥ P1 P22 .
La projection p diminue donc les distances.
Complexes
Homographies
Aide
Précédente
Elle est affine si et seulement si C est un sous-espace affine de E : la condition est
nécessaire, puisque l’image p(E) de la projection sur C est égale à C et que l’image
d’une application affine est un sous-espace affine ; elle est suffisante, puisque si C
est un sous-espace affine de E, la projection sur C n’est autre que la projection
orthogonale sur ce sous-espace.
Suivante
Plein écran
Quitter
Table
Séparation de convexes
Accueil
Géométrie affine
Convexité
Géométrie euclidienne
Isométries planes
Soient C1 et C2 deux convexes compacts disjoints d’un espace affine euclidien E.
Montrer qu’il existe un hyperplan H de E qui sépare strictement C1 de C2 , i.e. tel
que C1 soit inclus dans l’un des deux demi-espaces ouverts délimités par H et C2
dans l’autre.
Isométries de l’espace
Coniques
Géométrie analytique
Complexes
Homographies
Aide
Précédente
Suivante
Plein écran
Quitter
Indication
Solution
Soient P1 ∈ C1 et P2 ∈ C2 des points réalisant la distance de C1 à C2 :
P1 P2 = min{Q1 Q2 | Q1 ∈ C1 , Q2 ∈ C2 } .
Accueil
Géométrie affine
Convexité
Géométrie euclidienne
(On expliquera pourquoi de tels points existent.)
Montrer que l’hyperplan médiateur de P1 P2 convient.
Isométries planes
Isométries de l’espace
Coniques
Géométrie analytique
Complexes
Homographies
Aide
Précédente
Suivante
Plein écran
Quitter
Solution
La fonction (Q1 , Q2 ) 7→ Q1 Q2 de C1 × C2 dans [0, + ∞[ est continue. Elle atteint
donc son minimum sur le compact C1 × C2 . Soient P1 ∈ C1 et P2 ∈ C2 des points
réalisant ce minimum :
Accueil
Géométrie affine
Convexité
Géométrie euclidienne
Isométries planes
Isométries de l’espace
P1 P2 = min{Q1 Q2 | Q1 ∈ C1 , Q2 ∈ C2 }
et H1 (resp. H2 ) l’hyperplan orthogonal à P1 P2 passant par P1 (resp. P2 ). P1 (resp.
P2 ) est alors le projeté de P2 (resp. de P1 ) sur C1 (resp. C2 ) et C1 (resp. C2 ) est situé
tout entier du côté de H1 (resp. H2 ) qui ne contient pas P2 (resp. P1 ). L’hyperplan
médiateur H de P1 P2 est parallèle à P1 et P2 et sépare strictement C1 et C2 .
Coniques
Géométrie analytique
Complexes
Homographies
Aide
Précédente
Suivante
Plein écran
Quitter
Table
Hyperplans d’appui
Accueil
Géométrie affine
Convexité
Géométrie euclidienne
Isométries planes
Isométries de l’espace
Montrer que pour tout point M de la frontière d’un convexe fermé C d’un espace
affine euclidien E il existe un hyperplan affine H de E vérifiant :
• M appartient à H ;
• C est situé tout entier d’un même côté de H.
Un tel hyperplan est appelé hyperplan d’appui de C en M .
Coniques
Géométrie analytique
Complexes
Homographies
Aide
Précédente
Suivante
Plein écran
Quitter
Indication
Solution
Accueil
Géométrie affine
Convexité
Considérez une suite (Mn ) de points de E \ C convergeant vers M et leurs projetés
−−−→
Pn Mn
,
Pn sur C. Montrez que si ~u est une valeur d’adhérence de la suite ~un =
Pn Mn
l’hyperplan passant par M de vecteur normal ~u est un hyperplan d’appui de C en
M.
Géométrie euclidienne
Isométries planes
Isométries de l’espace
Coniques
Géométrie analytique
Complexes
Homographies
Aide
Précédente
Suivante
Plein écran
Quitter
Solution
Accueil
Géométrie affine
Convexité
Géométrie euclidienne
Isométries planes
Isométries de l’espace
Soit (Mn ) une suite de points de E \ C convergeant vers M (une telle suite existe
puisque C est fermé et que M appartient à la frontière de C). Soit Pn le projeté de
−−−→
Pn M n
. La suite Pn converge vers M , puisque M Pn ≤ M Mn (la
Mn sur C et ~un =
Pn M n
−−−→ −−→
projection p sur C diminue les distances et p(M ) = M ) et Pn Mn · Pn Q ≤ 0 pour
tout point Q de C, ce qui équivaut à :
−−→
~un · Pn Q ≤ 0 pour tout point Q de C (∗)
Coniques
Géométrie analytique
Complexes
Homographies
Soit ~u une valeur d’adhérence de la suite (~un ) (cette suite est bornée, puisque tous
les vecteurs ~un sont unitaires). En passant à la limite dans la relation (∗), on obtient
−−→
~u · M Q ≤ 0 pour tout point Q de C, ce qui signifie que C se trouve tout entier
d’un même côté de l’hyperplan normal à ~u passant par M .
Aide
Précédente
Suivante
Plein écran
Quitter
Remarque
Table
Remarque : Si dim(E) = 2 et si la frontière de C est une courbe lisse, l’hyperplan
d’appui en M est unique : c’est la tangente en M à la frontière de C. Dans le cas
général, il peut exister une infinité d’hyperplans d’appui en M .
Accueil
Géométrie affine
Convexité
Géométrie euclidienne
Isométries planes
Isométries de l’espace
Coniques
Géométrie analytique
Complexes
Homographies
Aide
Précédente
Suivante
Plein écran
Quitter
Table
Génération par les demi-espaces
Accueil
Géométrie affine
Convexité
Montrer que tout convexe fermé d’un espace affine euclidien est l’intersection des
demi-espaces fermés qui le contiennent.
Géométrie euclidienne
Isométries planes
Isométries de l’espace
Coniques
Géométrie analytique
Complexes
Homographies
Aide
Précédente
Suivante
Plein écran
Quitter
Solution
Accueil
Géométrie affine
Convexité
Géométrie euclidienne
Isométries planes
Isométries de l’espace
Soit C un convexe fermé d’un espace affine euclidien E. L’intersection C 0 des demiespaces fermés contenant C contient évidemment C. Montrons que le complémentaire
de C 0 dans E contient le complémentaire de C : l’assertion en résultera. Soit
−−→ −→
M ∈ E \ C et P son projeté sur C. L’inégalité P M · P Q ≤ 0 pour tout point
−−→
Q de C montre que C est inclus dans le demi-espace fermé de E orthogonal à P M
passant par P qui ne contient pas M . Ce demi-espace contient C 0 et M ne lui
appartient pas : M appartient donc au complémentaire de C 0 .
Coniques
Géométrie analytique
Complexes
Homographies
Aide
Précédente
Suivante
Plein écran
Quitter
Remarque
Table
Accueil
Géométrie affine
Soit C = E, aucun demi-espace de E ne contient C et C 0 = E. La seconde partie
de la démonstration montre que si C 6= E, il existe au moins un demi-espace fermé
de E contenant C. Tout convexe fermé de E distinct de E est donc inclus dans un
demi-espace.
Convexité
Géométrie euclidienne
Isométries planes
Isométries de l’espace
Coniques
On peut également remarquer que la propriété démontrée est purement affine :
la démonstration donnée ici utilise la structure euclidienne de E, mais tout espace
affine de dimension finie peut être muni d’une structure euclidienne et le résultat
ne dépend pas du choix de cette structure.
Géométrie analytique
Complexes
Homographies
Aide
Précédente
Suivante
Plein écran
Quitter
Table
Domaines de Voronoï
Accueil
Géométrie affine
Convexité
Soient A1 , . . . , An n points distincts du plan affine euclidien E. Montrer que les
domaines Vi (i = 1, . . . ,n) définis par :
Géométrie euclidienne
Vi = { M ∈ E | M Ai ≤ M Aj pour tout j = 1, . . . ,n }
Isométries planes
Isométries de l’espace
sont convexes.
Coniques
Géométrie analytique
Complexes
Homographies
Aide
Précédente
Suivante
Plein écran
Quitter
Indication
Solution
Que peut-on dire de l’ensemble des points M vérifiant M Ai ≤ M Aj ?
Accueil
Géométrie affine
Convexité
Géométrie euclidienne
Isométries planes
Isométries de l’espace
Coniques
Géométrie analytique
Complexes
Homographies
Aide
Précédente
Suivante
Plein écran
Quitter
Solution
Pour tout i et tout j, l’ensemble des points M vérifiant M Ai ≤ M Aj est le demiplan fermé délimité par la médiatrice de Ai Aj contenant Ai . Un demi-plan est
convexe : Vi est donc convexe comme intersection de convexes.
Accueil
Géométrie affine
Convexité
Géométrie euclidienne
Isométries planes
Isométries de l’espace
Coniques
Géométrie analytique
Complexes
Homographies
Aide
Précédente
Suivante
Plein écran
Quitter
Table
Distances aux points d’un repère affine
Accueil
Géométrie affine
Convexité
Géométrie euclidienne
Soit E un espace affine euclidien de dimension n et A0 , A1 , . . . ,An un repère affine
de E. Montrer qu’un point M de E est entièrement déterminé par ses distances
aux points de ce repère : M Ai = N Ai pour tout i = 0, 1, . . . ,n implique M = N .
Isométries planes
Isométries de l’espace
Coniques
Géométrie analytique
Complexes
Homographies
Aide
Précédente
Suivante
Plein écran
Quitter
Solution
Accueil
Géométrie affine
Supposons qu’il existe deux points distincts M et N de E tels que M Ai = N Ai
pour tout i = 0, 1, . . . ,n. Les n + 1 points A0 , . . . ,An appartiendraient tous à l’hyperplan médiateur de M N , ce qui contredirait le fait que A0 , A1 , . . . ,An est un
repère affine de E.
Convexité
Géométrie euclidienne
Isométries planes
Isométries de l’espace
Remarque : l’application M 7→ (M A0 , . . . ,M An ) est donc injective, mais elle n’est
pas surjective : on ne peut se donner arbitrairement les distances M Ai .
Coniques
Géométrie analytique
Complexes
Homographies
Aide
Précédente
Suivante
Plein écran
Quitter
Table
Fonction scalaire de Leibniz
Accueil
Géométrie affine
Soit (Ai ,λi )i=1,...,n un système de points pondérés d’un espace affine euclidien E de
Convexité
poids total
Géométrie euclidienne
n
X
Isométries planes
ϕ(M ) =
Isométries de l’espace
Coniques
Géométrie analytique
Complexes
Homographies
λi non nul. On définit une fonction ϕ de E dans R par
i=1
n
X
λi M A2i .
i=1
Montrer que
„
ϕ(M ) =
n
X
Ž
λi M G2 + ϕ(G)
i=1
où G est le barycentre du système pondéré (Ai ,λi )i=1,...,n .
Aide
Précédente
Suivante
Plein écran
Quitter
Solution
Soit G le barycentre du système pondéré (Ai ,λi )i=1,...,n . On a :
Accueil
ϕ(M ) =
Géométrie affine
Convexité
=
Géométrie euclidienne
Isométries planes
=
Isométries de l’espace
i=1
n
X
i=1
n
X
λi M A2i

−−→ −−→‹2
λi M G + GAi
n
n
−−→ X
−−→ X
λi M G2 + 2M G ·
λi GAi +
λi GA2i
i=1
n
X
„
Coniques
=
Géométrie analytique
Ž
i=1
i=1
λi M G2 + ϕ(G)
i=1
Complexes
Homographies
n
X
puisque
n
X
−−→ −
→
λi GAi = 0 par définition du barycentre.
i=1
Aide
Précédente
Suivante
Plein écran
Quitter
Suite
En déduire les lignes de niveau de la fonction ϕ :
Accueil
Lk = {M ∈ E |
λi M A2i = k}
i=1
Géométrie affine
Convexité
n
X
pour tout réel k.
Géométrie euclidienne
Isométries planes
Isométries de l’espace
Coniques
Géométrie analytique
Complexes
Homographies
Aide
Précédente
Suivante
Plein écran
Quitter
Solution
L’égalité obtenue précédemment montre que ϕ(M ) = k si et seulement si
Accueil
M G2 =
k − ϕ(G)
Géométrie affine
Isométries planes
Isométries de l’espace
Coniques
Homographies
Aide
λi
Il en résulte que Lk est :
k − ϕ(G)
• vide si X
< 0;
n
λi
i=1
Géométrie analytique
Complexes
.
i=1
Convexité
Géométrie euclidienne
n
X
• réduit au point G si
k − ϕ(G)
n
X
= 0;
λi
i=1
• une sphère de centre G si
k − ϕ(G)
n
X
> 0.
λi
i=1
Précédente
Suivante
Application : les cercles d’Apollonius.
Plein écran
Quitter
Table
Cercles d’Apollonius
Accueil
Géométrie affine
Convexité
Géométrie euclidienne
Isométries planes
Isométries de l’espace
Soient A et B deux points distincts du plan affine euclidien E et k un réel strictement positif différent de 1. Déterminer l’ensemble Ck des points M de E dont le
rapport des distances à A et B est égal à k :
Ck = {M ∈ E |
MA
= k} .
MB
Coniques
Géométrie analytique
Complexes
Homographies
Aide
Précédente
Suivante
Plein écran
Quitter
Indication
Solution
Première méthode : commencez par montrer qu’il existe exactement deux points K
−−→ −−→
et L de la droite AB appartenant à Ck , puis évaluez le produit scalaire M K · M L
pour M ∈ Ck .
Accueil
Géométrie affine
Convexité
Géométrie euclidienne
Seconde méthode : utilisez le résultat de l’exercice précédent.
Isométries planes
Isométries de l’espace
Coniques
Géométrie analytique
Complexes
Homographies
Aide
Précédente
Suivante
Plein écran
Quitter
Retour
Solution
Accueil
Géométrie affine
Convexité
Géométrie euclidienne
Isométries planes
Isométries de l’espace
−−→
−−→
MA
Si M appartient à la droite AB, l’égalité
= k équivaut à M A = k M B
MB
−−→
−−→
ou M A = −k M B. On obtient ainsi exactement deux points K et L, qui sont
les barycentres des systèmes pondérés ((A,1)),(B,k)) et ((A,1),(B, − k)). On a
−−→
−−→
−−→
−−→
−−→
M A − kM B
M A + k M B −−→
, ML =
, d’où
alors, pour tout point M de E, M K =
1+k
1−k
−−→ −−→ M A2 − k 2 M B 2
MK · ML =
. Il en résulte que M appartient à Ck si et seulement
1 − k2
−−→ −−→
si M K · M L = 0, i.e. si et seulement si M appartient au cercle de diamètre KL.
Coniques
Géométrie analytique
Complexes
Homographies
Aide
Précédente
Suivante
Plein écran
Autre méthode
Quitter
Suite
Accueil
Géométrie affine
Convexité
Géométrie euclidienne
Isométries planes
Isométries de l’espace
MA
La relation
= k s’écrit aussi ϕ(M ) = 0, où ϕ(M ) = M A2 − k 2 M B 2 . En
MB
appliquant le résultat de l’exercice précédent, on voit que Ck est un cercle de
centre G, où G est le barycentre du système pondéré ((A,1),(B, − k 2 )). Les points
K et L appartiennent à Ck et G appartient à la droite AB : Ck est donc le cercle
de diamètre KL. On peut vérifier directement que G est le milieu de KL :
!
!
1
−k 2
1
1
k
1
1
−k
A
+
B
=
A
+
B
+
A
+
B .
1 − k2
1 − k2
2 1+k
1+k
2 1−k
1−k
Coniques
Géométrie analytique
Complexes
Homographies
Aide
Précédente
Suivante
Plein écran
Quitter
Suite
Montrer que, pour tout réel k > 0, k 6= 1, le cercle Ck est orthogonal au cercle de
diamètre AB. En déduire qu’il est orthogonal à tout cercle passant par A et B.
Accueil
Géométrie affine
Convexité
Géométrie euclidienne
Isométries planes
Isométries de l’espace
Coniques
Géométrie analytique
Complexes
Homographies
Aide
Précédente
Suivante
Plein écran
Quitter
Solution
Soit I le milieu de AB. La puissance de I par rapport au cercle Ck est égale à :
−→ −
→
IK · IL =
Accueil
Géométrie affine
Convexité
Géométrie euclidienne
Isométries planes
Isométries de l’espace
Coniques
!
!
→
→
1 −
k −→
1 −
−k −→
IA +
IB ·
IA +
IB
1+k
1+k
1−k
1−k
IA2 − k 2 IB 2
=
1 − k2
= IA2 = IB 2
qui est aussi le rayon du cercle de diamètre AB. Ce cercle est donc orthogonal à
Ck .
Géométrie analytique
Complexes
Homographies
Aide
Précédente
Suivante
Plein écran
Quitter
Suite
Accueil
Géométrie affine
Convexité
Il en résulte que la puissance du centre G de Ck par rapport au cercle de diamètre
AB est égale au rayon de Ck . Mais cette puissance est aussi la puissance de G par
−→ −−→
rapport à tout cercle passant par A et B, puisqu’elle est égale à GA · GB. Tous les
cercles Ck sont donc orthogonaux à tous les cercles passant par A et B (ces deux
familles de cercles constituent des faisceaux orthogonaux).
Géométrie euclidienne
Isométries planes
Isométries de l’espace
Coniques
Géométrie analytique
Complexes
Homographies
Aide
Précédente
Suivante
Plein écran
Quitter
Table
Le triangle orthique
Accueil
Géométrie affine
Convexité
Soit ABC un triangle du plan affine euclidien et A0 , B 0 , C 0 les pieds des hauteurs.
Montrer que les côtés du triangle ABC sont des bissectrices du triangle A0 B 0 C 0 .
Géométrie euclidienne
Isométries planes
Isométries de l’espace
Coniques
Géométrie analytique
Complexes
Homographies
Aide
Précédente
Suivante
Plein écran
Quitter
Indication
Solution
Montrer que la somme des angles de droites (A0 A,A0 B 0 ) + (A0 A,A0 C 0 ) est nulle (que
peut-on dire des cercles de diamètres AB et AC ?).
Accueil
Géométrie affine
Convexité
Géométrie euclidienne
Isométries planes
Isométries de l’espace
Coniques
Géométrie analytique
Complexes
Homographies
Aide
Précédente
Suivante
Plein écran
Quitter
Retour
Solution
Les points A, A0 , B, B 0 (resp. A, A0 , C, C 0 ) sont cocycliques, puisqu’ils appartiennent au cercle de diamètre AB (resp. AC). On a donc :
Accueil
Géométrie affine
Convexité
Géométrie euclidienne
Isométries planes
Isométries de l’espace
(A0 A,A0 B 0 ) + (A0 A,A0 C 0 ) = (BA,BB 0 ) + (CA,CC 0 )
= (BA,AC) + (AC,BB 0 ) + (CA,AB) + (AB,CC 0 )
= 0
(égalités d’angles de droites), ce qui montre que AA0 est une bissectrice de l’angle
en A0 du triangle A0 B 0 C 0 .
Coniques
Géométrie analytique
Complexes
Homographies
Aide
Précédente
Suivante
Plein écran
Quitter
Remarque
Table
Remarque :
Accueil
Géométrie affine
Convexité
Géométrie euclidienne
Isométries planes
Si le triangle ABC a tous ses angles aigus, les hauteurs AA0 , BB 0 et CC 0 sont
les bissectrices intérieures du triangle A0 B 0 C 0 ; si l’un des angles du triangle ABC
est obtus, deux des hauteurs de ABC sont des bissectrices extérieures de A0 B 0 C 0 ,
l’autre une bissectrice intérieure.
Isométries de l’espace
Coniques
Géométrie analytique
Complexes
Pour une application de cet exercice, on pourra étudier le problème de Fagnano
(trajectoire de billard dans le triangle).
Homographies
Aide
Précédente
Suivante
Plein écran
Quitter
Table
Bissectrices et cercle circonscrit
Accueil
Géométrie affine
Convexité
Géométrie euclidienne
Les bissectrices intérieure et extérieure en A d’un triangle ABC non isocèle en A
recoupent le cercle Γ circonscrit à ce triangle respectivement en I et J. Montrer
que les points I et J appartiennent à la médiatrice de BC.
Isométries planes
Isométries de l’espace
Coniques
Géométrie analytique
Complexes
Homographies
Aide
Précédente
Suivante
Plein écran
Quitter
Indication
Solution
−−→ −→
−→ −→
−−→ −→
−→ −→
Comparer les angles de vecteurs (OB,OI) et (OI,OC) (resp. (OB,OJ) et (OJ,OC))
où O est le centre du cercle circonscrit Γ.
Accueil
Géométrie affine
Convexité
Géométrie euclidienne
Isométries planes
Isométries de l’espace
Coniques
Géométrie analytique
Complexes
Homographies
Aide
Précédente
Suivante
Plein écran
Quitter
Retour
Solution
Accueil
Géométrie affine
Convexité
Géométrie euclidienne
Les angles orientés de droites (AB,AI) et (AI,AC) (resp. (AB,AJ) et (AJ,AC))
−−→ −→
−→ −→
sont égaux. L’égalité (OB,OI) = 2(AB,AI) = 2(AI,AC) = (OI,OC) d’angles
−−→ −→
orientés de vecteurs en résulte par le théorème de l’angle inscrit. De même (OB,OJ) =
−→ −→
(OJ,OC). Les points I et J appartiennent donc à la bissectrice intérieure en O du
triangle isocèle OBC, qui est aussi la médiatrice de BC.
Isométries planes
Isométries de l’espace
Coniques
Géométrie analytique
Complexes
Homographies
Aide
Précédente
Suivante
Plein écran
Quitter
Table
Le pivot
Accueil
Géométrie affine
Convexité
Géométrie euclidienne
Soit ABC un triangle et A0 , B 0 , C 0 trois points situés respectivement sur les côtés
BC, CA et AB de ce triangle et différents des sommets. Montrer que les cercles
circonscrits aux trois triangles AB 0 C 0 , BC 0 A0 et CA0 B 0 ont un point commun.
Isométries planes
Isométries de l’espace
Coniques
Géométrie analytique
Complexes
Homographies
Aide
Précédente
Suivante
Plein écran
Quitter
Indication
Solution
Soit I le point d’intersection des cercles circonscrits aux triangles AB 0 C 0 et BC 0 A0 .
Comparer les angles (IA0 ,IB 0 ) et (CA0 ,CB 0 ).
Accueil
Géométrie affine
Convexité
Géométrie euclidienne
Isométries planes
Isométries de l’espace
Coniques
Géométrie analytique
Complexes
Homographies
Aide
Précédente
Suivante
Plein écran
Quitter
Retour
Solution
Si les cercles circonscrits aux triangles AB 0 C 0 et BC 0 A0 se coupent en un second
point I, les égalités :
Accueil
Géométrie affine
Convexité
Géométrie euclidienne
Isométries planes
Isométries de l’espace
Coniques
Géométrie analytique
Complexes
Homographies
(IA0 ,IB 0 ) =
=
=
=
=
(IA0 ,IC 0 ) + (IC 0 ,IB 0 )
(BA0 ,BC 0 ) + (AC 0 ,AB 0 )
(BC,BA) + (AB,AC)
(CB,CA)
(CA0 ,CB 0 )
montrent que les points I, A0 , B 0 et C sont cocycliques.
Si ces deux cercles sont tangents en C 0 , les angles (B 0 C 0 ,B 0 C) et (A0 C 0 ,A0 C) sont
droits et le cercle de diamètre CC 0 , qui est le cercle circonscrit à CA0 B 0 , passe par
C 0.
Aide
Précédente
Suivante
Plein écran
Quitter
Table
Cercles tangents
Accueil
Géométrie affine
Convexité
Géométrie euclidienne
Isométries planes
Soient, dans le plan affine euclidien, C1 , C2 , C3 trois cercles de centres respectifs
O1 , O2 , O3 tangents extérieurement deux à deux. Montrer que les trois tangentes
communes à deux de ces cercles en leur point de contact sont concourantes. Que
représente leur point de concours pour le triangle O1 O2 O3 ?
Isométries de l’espace
Coniques
Géométrie analytique
Complexes
Homographies
Aide
Précédente
Suivante
Plein écran
Quitter
Solution
Accueil
Géométrie affine
Convexité
Géométrie euclidienne
Soient T1 , T2 , T3 les points de contact de ces cercles. Le point d’intersection I de
deux des tangentes considérées est équidistant de T1 , T2 , T3 , donc des côtés du
triangle O1 O2 O3 , puisque ces côtés sont perpendiculaires aux tangentes. Il est donc
centre d’un cercle tangent aux trois côtés du triangle O1 O2 O3 . Ce dernier cercle est
le cercle inscrit dans le triangle O1 O2 O3 , puisque les points T1 , T2 , T3 appartiennent
aux segments O2 O3 , O3 O1 , O1 O2 . La troisième tangente passe aussi par ce point.
Isométries planes
Isométries de l’espace
Coniques
Géométrie analytique
Complexes
Homographies
Aide
Précédente
Suivante
Plein écran
Quitter
Suite
Réciproquement, montrer que pour tout triangle O1 O2 O3 il existe exactement un
triplet de cercles C1 , C2 , C3 de centres O1 , O2 , O3 tangents extérieurement deux à
deux. Indiquer une construction de ces cercles.
Accueil
Géométrie affine
Convexité
Géométrie euclidienne
Isométries planes
Isométries de l’espace
Coniques
Géométrie analytique
Complexes
Homographies
Aide
Précédente
Suivante
Plein écran
Quitter
Solution
Accueil
Géométrie affine
Convexité
Géométrie euclidienne
Le point de concours I des trois tangentes est le centre du cercle inscrit dans le
triangle O1 O2 O3 et les points de contact T1 , T2 , T3 de ces cercles deux à deux
sont les points de contact de ce cercle inscrit avec les côtés du triangle, ce qui
détermine entièrement les cercles C1 , C2 , C3 . Pour les construire, il suffit donc de
construire le centre I du cercle inscrit (intersection des bissectrices intérieures),
puis ses projections T1 , T2 , T3 sur les côtés.
Isométries planes
Isométries de l’espace
Coniques
Géométrie analytique
Complexes
Homographies
Aide
Précédente
Suivante
Plein écran
Quitter
Table
Trois cercles
Accueil
Géométrie affine
Convexité
Géométrie euclidienne
Soient, dans le plan affine euclidien, C1 , C2 , C3 trois cercles tangents deux à deux.
Montrer que les trois tangentes communes à deux de ces cercles sont concourantes
ou parallèles.
Isométries planes
Isométries de l’espace
Coniques
Géométrie analytique
Complexes
Homographies
Aide
Précédente
Suivante
Plein écran
Quitter
Indication
Solution
Soit O le point d’intersection de deux de ces tangentes (si elles ne sont pas toutes
parallèles). Que peut-on dire de la puissance de O par rapport à ces cercles?
Accueil
Géométrie affine
Convexité
Géométrie euclidienne
Isométries planes
Isométries de l’espace
Coniques
Géométrie analytique
Complexes
Homographies
Aide
Précédente
Suivante
Plein écran
Quitter
Retour
Solution
Accueil
Géométrie affine
Si ces trois tangentes ne sont pas parallèles, le point O d’intersection de deux de
ces tangentes a même puissance par rapport aux trois cercles (on dit que O est le
centre radical des trois cercles). Il appartient donc à la troisième tangente commune
à deux de ces cercles.
Convexité
Géométrie euclidienne
Isométries planes
Isométries de l’espace
Coniques
Géométrie analytique
Complexes
Homographies
Aide
Précédente
Suivante
Plein écran
Quitter
Suite
En déduire que si ces trois tangentes ne sont pas parallèles, il existe un cercle
orthogonal aux trois cercles C1 , C2 , C3 .
Accueil
Géométrie affine
Convexité
Géométrie euclidienne
Isométries planes
Isométries de l’espace
Coniques
Géométrie analytique
Complexes
Homographies
Aide
Précédente
Suivante
Plein écran
Quitter
Solution
Le cercle de centre O passant par un des points de contact de ces tangentes passe
par les deux autres, puisque la puissance de O par rapport à chacun de ces cercles
est OT12 = OT22 = OT32 . Ce cercle est donc orthogonal aux trois cercles.
Accueil
Géométrie affine
Convexité
Géométrie euclidienne
Isométries planes
Isométries de l’espace
Coniques
Géométrie analytique
Complexes
Homographies
Aide
Précédente
Suivante
Plein écran
Quitter
Cas particulier
Table
Les trois tangentes communes à deux de ces cercles sont parallèles si et seulement
si leurs centres sont alignés. La droite des centres est dans ce cas orthogonale aux
trois cercles.
Accueil
Géométrie affine
Convexité
Géométrie euclidienne
Isométries planes
Isométries de l’espace
Coniques
Géométrie analytique
Complexes
Homographies
Aide
Précédente
Suivante
Plein écran
Quitter
Table
Le théorème des trois tangentes
Accueil
Géométrie affine
Convexité
Géométrie euclidienne
Soit ABC un triangle du plan affine euclidien. Le cercle exinscrit dans l’angle en
A touche les côtés BC, CA et AB en P , Q et R. Montrer que la somme AR + AQ
est égale au périmètre AB + BC + CA du triangle ABC.
Isométries planes
Isométries de l’espace
Coniques
Géométrie analytique
Complexes
Homographies
Aide
Précédente
Suivante
Plein écran
Quitter
Solution
Il suffit de remarquer que AR = AB +BR, AQ = AC +CQ, BR = BP , CQ = CP
(puisque la réflexion d’axe BIA (resp. CIA ) échange BR et BP (resp. CQ et CP )),
BC = BP + CP (puisque P appartient au segment BC).
Accueil
Géométrie affine
Convexité
Géométrie euclidienne
Isométries planes
Isométries de l’espace
Coniques
Géométrie analytique
Complexes
Homographies
Aide
Précédente
Suivante
Plein écran
Quitter
Retour
Suite
Accueil
Géométrie affine
En déduire que si l’on mène d’un point A extérieur à un cercle Γ deux tangentes
AR et AQ à ce cercle et si la tangente à Γ en un point P situé du même côté que
A de la droite RQ coupe ces deux tangentes en B et C, le périmètre du triangle
ABC est constant (i.e. ne dépend pas de la position de P sur l’arc RQ).
Convexité
Géométrie euclidienne
Isométries planes
Isométries de l’espace
Coniques
Géométrie analytique
Complexes
Homographies
Aide
Précédente
Suivante
Plein écran
Quitter
Solution
Il suffit de remarquer que Γ est le cercle exinscrit dans l’angle en A du triangle ABC et d’appliquer le résultat précédent : le périmètre du triangle ABC
est AB + BC + CA = AR + AQ = 2AR = 2AQ, puisque AR = AQ.
Accueil
Géométrie affine
Convexité
Géométrie euclidienne
Isométries planes
Isométries de l’espace
Coniques
Géométrie analytique
Complexes
Homographies
Aide
Précédente
Suivante
Plein écran
Quitter
Table
Rayons des cercles inscrit et exinscrits
Accueil
Géométrie affine
Convexité
Géométrie euclidienne
Soit ABC un triangle, a = BC, b = CA, c = AB les longueurs de ses côtés,
2p = a + b + c son périmètre. Exprimer l’aire S de ce triangle en fonction de p et
du rayon r du cercle inscrit.
Isométries planes
Isométries de l’espace
Coniques
Géométrie analytique
Complexes
Homographies
Aide
Précédente
Suivante
Plein écran
Quitter
Solution
Accueil
Géométrie affine
Soit I le centre du cercle inscrit. L’aire S du triangle ABC est égale à la somme
des aires des triangles BIC, CIA et AIB. Ces trois triangles ont pour hauteur r
r
et pour bases a, b, c. La somme de leurs aires est donc S = (a + b + c) = rp.
2
Convexité
Géométrie euclidienne
Isométries planes
Isométries de l’espace
Coniques
Géométrie analytique
Complexes
Homographies
Aide
Précédente
Suivante
Plein écran
Quitter
Suite
Donner de même des expressions de S faisant intervenir les rayons rA , rB , rC des
cercles exinscrits.
Accueil
Géométrie affine
Convexité
Géométrie euclidienne
Isométries planes
Isométries de l’espace
Coniques
Géométrie analytique
Complexes
Homographies
Aide
Précédente
Suivante
Plein écran
Quitter
Solution
Accueil
Géométrie affine
Convexité
Géométrie euclidienne
Soit IA le centre du cercle exinscrit dans l’angle en A, P , Q, R les points de ce
cercle avec les côtés BC, CA et AB. En écrivant que la somme des aires des
triangles AIA R et AIA Q est égale à la somme de l’aire S du triangle ABC et des
aires des triangles BIA R, BIA P , CIA P et CIA Q et en remarquant que BP = BR,
rA
CP = CQ, on obtient S = (b + c − a) = rA (p − a).
2
Isométries planes
Isométries de l’espace
Coniques
Géométrie analytique
Complexes
Homographies
Aide
Précédente
Suivante
Plein écran
Quitter
Suite
En déduire la relation :
Accueil
1
1
1
1
=
+
+
.
r
rA rB rC
Géométrie affine
Convexité
Géométrie euclidienne
Isométries planes
Isométries de l’espace
Coniques
Géométrie analytique
Complexes
Homographies
Aide
Précédente
Suivante
Plein écran
Quitter
Solution
Des relations :
S = rp = rA (p − a) = rB (p − b) = rA (p − c)
Accueil
Géométrie affine
on tire :
Convexité
Géométrie euclidienne
Isométries planes
Isométries de l’espace
d’où :
1
p
= ,
r
S
p−a
1
=
,
rA
S
p−b
1
=
,
rB
S
p−c
1
=
,
rC
S
1
1
3p − a − b − c
p
1
1
+
+
=
= = .
rA rB rC
S
S
r
Coniques
Géométrie analytique
Complexes
Homographies
Aide
Précédente
Suivante
Plein écran
Quitter
Table
Un problème de maximisation
Accueil
Géométrie affine
Convexité
Géométrie euclidienne
Isométries planes
Soient C et C 0 deux cercles sécants de centres O et O0 , A un de leurs points
d’intersection. Une droite D passant par A recoupe C et C 0 en deux points M
et M 0 situés de part et d’autre de A. Déterminer la position de la droite D qui
maximise la distance M M 0 et expliciter la valeur maximale de cette distance.
Isométries de l’espace
Coniques
Géométrie analytique
Complexes
Homographies
Aide
Précédente
Suivante
Plein écran
Quitter
Indication
Solution
Soient I et I 0 les milieux des segments AM et AM 0 . Montrez que II 0 ≤ OO0 . Dans
quel cas a-t-on égalité?
Accueil
Géométrie affine
Convexité
Géométrie euclidienne
Isométries planes
Isométries de l’espace
Coniques
Géométrie analytique
Complexes
Homographies
Aide
Précédente
Suivante
Plein écran
Quitter
Retour
Solution
Accueil
Géométrie affine
Convexité
Géométrie euclidienne
Les droites OI et O0 I 0 sont perpendiculaires à D (ce sont les médiatrices des segments AM et AM 0 ). Les points I et I 0 sont donc les projetés orthogonaux des points
O et O0 sur la droite D. Une projection orthogonale diminuant les distances, il en
résulte que II 0 ≤ OO0 , avec égalité si et seulement si D est parallèle à OO0 . La
distance M M 0 = 2II 0 est donc toujours inférieure ou égale à 2OO0 et lui est égale
si et seulement si D est parallèle à OO0 .
Isométries planes
Isométries de l’espace
Coniques
Géométrie analytique
Complexes
Homographies
Aide
Précédente
Suivante
Plein écran
Quitter
Table
Le problème de Fermat
Accueil
Géométrie affine
Convexité
Géométrie euclidienne
Soit P QR un triangle équilatéral. Montrer que la fonction qui à un point M du plan
associe la somme de ses distances aux trois côtés du triangle P QR est constante à
l’intérieur de ce triangle.
Isométries planes
Isométries de l’espace
Coniques
Géométrie analytique
Complexes
Homographies
Aide
Précédente
Suivante
Plein écran
Quitter
Solution
Suite
Accueil
Géométrie affine
Convexité
Soit M un point intérieur au triangle P QR, A, B et C ses projections orthogonales
sur les côtés QR, RP et P Q. L’aire du triangle P QR est la somme des aires
des triangles M P Q, M QR et M RP . Ces triangles ont tous trois pour base le
côté du triangle équilatéral. La somme de leurs hauteurs, qui est aussi la somme
M A+M B+M C des distances de M aux trois côtés du triangle, est donc constante.
Géométrie euclidienne
Isométries planes
Isométries de l’espace
Coniques
Géométrie analytique
Complexes
Homographies
Aide
Précédente
Suivante
Plein écran
Quitter
Retour
Suite
Accueil
Géométrie affine
Convexité
Géométrie euclidienne
Soit M un point intérieur au triangle P QR, A, B et C ses projections orthogonales
sur les côtés QR, RP et P Q. Montrer que M est l’unique point du plan minimisant
la somme N A + N B + N C des distances d’un point N aux trois points A, B et C.
Isométries planes
Isométries de l’espace
Coniques
Géométrie analytique
Complexes
Homographies
Aide
Précédente
Suivante
Plein écran
Quitter
Solution
Accueil
Géométrie affine
Convexité
Géométrie euclidienne
Isométries planes
Isométries de l’espace
Coniques
La solution du problème de minimisation est à rechercher dans le triangle ABC.
En effet si un point N est extérieur à ce triangle, il est situé dans un demi-plan
délimité par un des côtés et ne contenant pas le sommet opposé ; sa projection
orthogonale N 0 sur ce côté vérifie alors N 0 A + N 0 B + N 0 C < N A + N B + N C. Or
tout point M 0 du triangle P QR vérifie M 0 A0 + M 0 B 0 + M 0 C 0 = M A + M B + M C,
où A0 , B 0 , C 0 sont les projections orthogonales de M 0 sur les droites QR, RP et
P Q. Les inégalités M 0 A0 ≤ M 0 A, M 0 B 0 ≤ M 0 B, M 0 C 0 ≤ M 0 C montrent alors que
M A + M B + M C ≤ M 0 A + M 0 B + M 0 C, l’égalité ne pouvant être atteinte que si
M 0 est situé sur les droites M A, M B et M C, i.e. égal à M .
Géométrie analytique
Complexes
Homographies
Aide
Précédente
Suivante
Plein écran
Remarque
Quitter
Accueil
Géométrie affine
Convexité
Géométrie euclidienne
On peut montrer que si ABC est un triangle dont tous les angles sont inférieurs à
2π/3, il existe un unique point M intérieur à ABC à partir duquel on voit les trois
côtés AB, BC et CA sous le même angle, i.e. sous un angle de 2π/3. Le point M est
alors l’unique point du plan minimisant la somme des distances M A + M B + M C.
On peut en effet reconstituer le triangle équilatéral P QR à partir du triangle ABC
en menant par A, B et C les perpendiculaires à M A, M B et M C.
Isométries planes
Isométries de l’espace
Coniques
Géométrie analytique
Complexes
Homographies
Aide
Précédente
Suivante
Plein écran
Quitter
Question : comment construit-on ce point M ?
Solution
Accueil
Géométrie affine
Il suffit de prendre l’intersection des arcs capables lieux des points M tels que
−−→ −−→
−−→ −−→ −−→ −−→
les angles (M B,M C) (resp. (M C,M A), (M A,M B)) valent 2π/3. Les cercles portant ces arcs sont les cercles circonscrits aux triangles équilatéraux construits à
l’extérieur du triangle ABC sur les côtés de ce triangle.
Convexité
Géométrie euclidienne
Isométries planes
Isométries de l’espace
Coniques
Géométrie analytique
Complexes
Homographies
Aide
Précédente
Suivante
Plein écran
Table
Quitter
Arcs interceptés par deux sécantes
Accueil
Géométrie affine
Convexité
Géométrie euclidienne
Isométries planes
Soit Γ un cercle du plan affine euclidien P , O son centre et M un point de P
n’appartenant pas à Γ. Deux sécantes issues de M coupent le cercle respectivement
en A et B, et C et D. Démontrer l’égalité :
−−→ −−→
−→ −→
2(M A,M C) = (OB,OD) − (OC,OA) .
Isométries de l’espace
Coniques
Géométrie analytique
Complexes
Homographies
Aide
Précédente
Suivante
Plein écran
Quitter
Indication
Solution
Suite
Indication :
Accueil
Géométrie affine
Décomposez (M A,M C) en (M A,AD) + (AD,M C).
Convexité
Géométrie euclidienne
Isométries planes
Isométries de l’espace
Coniques
Géométrie analytique
Complexes
Homographies
Aide
Précédente
Suivante
Plein écran
Quitter
Retour
Solution
Suite
On a, par la relation de Chasles et le théorème de l’angle inscrit :
Accueil
Géométrie affine
Convexité
Géométrie euclidienne
Isométries planes
Isométries de l’espace
2(M A,M C) = 2(M A,AD) + 2(AD,M C)
= 2(AB,AD) + 2(DA,DC)
−−→ −−→
−→ −→
= (OB,OD) + (OA,OC)
−−→ −−→
−→ −→
= (OB,OD) − (OC,OA)
(égalités d’angles orientés de vecteurs).
Coniques
Géométrie analytique
Complexes
Homographies
Aide
Précédente
Suivante
Plein écran
Quitter
Remarque
Suite
Remarque :
Accueil
Géométrie affine
Convexité
Géométrie euclidienne
Isométries planes
Cette égalité d’angles orientés se traduit différemment en termes d’angles géométriques
suivant la position du point par rapport au cercle : si M est extérieur à Γ, l’angle
Ø
×
Ø
géométrique AM
C est la demi-différence des angles au centre AOC
et BOD,
si M
Ø
est intérieur à Γ, l’angle géométrique AM B en est la demi-somme.
Isométries de l’espace
Coniques
Géométrie analytique
Complexes
Homographies
Aide
Précédente
Suivante
Plein écran
Quitter
Retour
Suite
Accueil
Géométrie affine
Soient A et B deux points d’un cercle de centre O et de rayon R. On construit
sur le diamètre OA de ce cercle un point C extérieur au cercle tel que la droite
CB recoupe le cercle en un point M vérifiant M C = R. Montrer que la mesure de
×
×
l’angle géométrique ACB
est le tiers de la mesure de l’angle géométrique AOB.
Convexité
Géométrie euclidienne
Isométries planes
Isométries de l’espace
Coniques
Géométrie analytique
Complexes
Homographies
Aide
Précédente
Suivante
Plein écran
Quitter
Solution
Remarque
La première question de l’exercice nous apprend que :
−→ −−→
−−→ −→
2(CA,CB) = (OA,OB) − (OM ,OC) .
Accueil
Géométrie affine
Géométrie euclidienne
Le triangle CM O étant isocèle en M , il en résulte
−−→ −→
−→ −−→
(OM ,OC) = (CA,CB)
Isométries planes
d’où :
Convexité
Isométries de l’espace
−→ −−→
−→ −−→
3(CA,CB) = (OA,OB) .
Coniques
Géométrie analytique
Complexes
Homographies
Aide
Précédente
Suivante
Plein écran
Quitter
Remarque
Retour
Remarque :
Accueil
Géométrie affine
Convexité
Géométrie euclidienne
Isométries planes
Isométries de l’espace
Coniques
Géométrie analytique
L’exercice permet donc de diviser en trois un angle donné. Il ne prétend pas pour
autant résoudre le problème de la trisection de l’angle à la règle et au compas
(problème dont on sait qu’il est en toute généralité impossible). En effet la construction précédente peut se réaliser si l’on dispose d’un compas et d’une règle sur laquelle on peut reporter les distances (en l’occurence le rayon du cercle). Mais ce
report est interdit dans les problèmes classiques de construction à la règle et au
compas, où la règle ne peut servir qu’à tracer des droites.
Complexes
Homographies
Aide
Précédente
Suivante
Plein écran
Quitter
Retour
Table
Un problème de recouvrement
Accueil
Géométrie affine
Convexité
Géométrie euclidienne
Isométries planes
Isométries de l’espace
n disques de même rayon sont disposés sur une table rectangulaire sans se chevaucher. On suppose qu’il est impossible de rajouter sur la table un disque de
même rayon qui ne chevauche aucun des précédents. Montrer qu’on peut recouvrir complètement la table avec 4n disques de même rayon que les précédents (les
chevauchements étant cette fois autorisés).
Coniques
Géométrie analytique
Complexes
Homographies
Aide
Précédente
Suivante
Plein écran
Quitter
Indication
Solution
Accueil
Géométrie affine
Soit A, B, C, D les coins de la table, r le rayon commun des disques, O1 , . . . , On
leurs centres. Montrez d’abord que les n disques de centres O1 , . . . , On et de rayon
2r recouvrent complètement la table (en se chevauchant). Passez ensuite à des
disques de rayon r en utilisant des homothéties.
Convexité
Géométrie euclidienne
Isométries planes
Isométries de l’espace
Coniques
Géométrie analytique
Complexes
Homographies
Aide
Précédente
Suivante
Plein écran
Quitter
Retour
Solution
Accueil
Géométrie affine
Convexité
Géométrie euclidienne
Isométries planes
Isométries de l’espace
Soit A, B, C, D les coins de la table, r le rayon commun des disques, O1 , . . . , On
leurs centres. Les n disques de centres O1 , . . . , On et de rayon 2r recouvrent la table
(en se chevauchant) : en effet si un point M de la table vérifiait M Oi > 2r pour
tout i = 1, . . . ,n, le disque de centre M et de rayon r serait disjoint des disques de
centres O1 , . . . , On et de rayon r, ce qui est contraire à l’hypothèse. Les n disques
images de ces disques par l’homothétie de centre A (resp. B, C, D) et de rapport
1/2 recouvrent l’image du rectangle ABCD par cette même homothétie et ont
pour rayon r. Les 4n disques ainsi obtenus conviennent.
Coniques
Géométrie analytique
Complexes
Homographies
Aide
Précédente
Suivante
Plein écran
Quitter
Table
Disque de rayon minimal contenant un compact (1)
Accueil
Géométrie affine
Convexité
Géométrie euclidienne
Isométries planes
Soient D1 et D2 deux disques fermés de même rayon ρ du plan affine euclidien.
Montrer qu’il existe un disque fermé de rayon r < ρ contenant l’intersection de D1
et D2 . Préciser la valeur minimale de r en fonction de ρ et de la distance d des
centres de D1 et D2 .
Isométries de l’espace
Coniques
Géométrie analytique
Complexes
Homographies
Aide
Précédente
Suivante
Plein écran
Quitter
Solution
Suite
Si d > 2ρ, l’intersection est vide et r = 0.
Accueil
Géométrie affine
Convexité
Sinon l’intersection
D1 ∩ D2 est incluse dans le disque de centre le milieu de O1 O2
Ì
d2
2
et de rayon ρ − .
4
Géométrie euclidienne
Isométries planes
Isométries de l’espace
Coniques
Géométrie analytique
Complexes
Homographies
Aide
Précédente
Suivante
Plein écran
Quitter
Retour
Suite
Disque de rayon minimal contenant un compact (2)
Accueil
Géométrie affine
Convexité
Soit K un compact non vide du plan affine euclidien. On se propose de montrer
qu’il existe un unique disque fermé de rayon minimal contenant K.
Géométrie euclidienne
Isométries planes
Isométries de l’espace
Coniques
Géométrie analytique
Existence
Soit ρ la borne inférieure de l’ensemble des réels r tels qu’il existe un disque fermé
de rayon r contenant K. Montrer qu’il existe un disque fermé de rayon ρ contenant
K.
Complexes
Homographies
Aide
Précédente
Suivante
Plein écran
Quitter
Solution
Suite
Accueil
Géométrie affine
Convexité
Géométrie euclidienne
Isométries planes
Isométries de l’espace
Coniques
Géométrie analytique
Complexes
Homographies
K étant compact, il existe un disque fermé de centre O et de rayon R contenant K.
L’ensemble des réels r ≥ 0 tels qu’il existe un disque fermé de rayon r contenant
K n’est donc pas vide et il est minoré par 0. Soit ρ sa borne inférieure. Si ρ = R,
c’est terminé. Sinon, il existe une suite rn de réels ≤ R de limite ρ et une suite
On de points du plan telles que, pour tout n, K soit inclus dans le disque fermé
de centre On et de rayon rn . Les points On appartiennent tous au disque fermé
de centre O et de rayon 2R, puisque OOn ≤ OM + M On ≤ R + rn ≤ 2R pour
tout point M de K. Ce disque étant compact, on peut extraire de la suite On une
sous-suite convergente Onp . Soit Ω sa limite. On vérifie facilement que K est inclus
dans le disque fermé de rayon ρ et de centre Ω : en effet tout point M de K vérifie
M Onp ≤ rnp , d’où, en passant à la limite, M Ω ≤ ρ.
Aide
Précédente
Suivante
Plein écran
Quitter
Suite
Disque de rayon minimal contenant un compact (3)
Accueil
Géométrie affine
Convexité
Soit K un compact non vide du plan affine euclidien. On se propose de montrer
qu’il existe un unique disque fermé de rayon minimal contenant K.
Géométrie euclidienne
Isométries planes
Isométries de l’espace
Coniques
Géométrie analytique
Unicité
La borne inférieure ρ de l’ensemble des réels r tels qu’il existe un disque fermé de
rayon r contenant K est donc atteinte. Montrer qu’il n’y a qu’un seul disque fermé
de rayon ρ qui contient K.
Complexes
Homographies
Aide
Précédente
Suivante
Plein écran
Quitter
Solution
Accueil
Géométrie affine
S’il existait deux disques fermés distincts de même rayon ρ et de centres respectifs
O1 et O2 contenant K, il existerait, d’après la première question, un disque de
rayon r < ρ contenant K, ce qui contredirait la définition de ρ. Le disque fermé de
rayon ρ contenant K est donc unique.
Convexité
Géométrie euclidienne
Isométries planes
Isométries de l’espace
Coniques
Géométrie analytique
Complexes
Homographies
Aide
Précédente
Suivante
Plein écran
Quitter
Application
Application
Accueil
Géométrie affine
Convexité
En déduire que si G est le groupe des isométries conservant un compact K, il existe
un point Ω fixe par tout élément de G.
Géométrie euclidienne
Isométries planes
Isométries de l’espace
Coniques
Géométrie analytique
Complexes
Homographies
Aide
Précédente
Suivante
Plein écran
Quitter
Solution
Le centre Ω de l’unique disque fermé de rayon minimal ρ contenant K est fixe par
tout élément f de G, puisque G = f (G) est inclus dans le disque de centre f (Ω)
et de rayon ρ.
Accueil
Géométrie affine
Convexité
Géométrie euclidienne
Isométries planes
Isométries de l’espace
Remarque : Si K est fini, l’isobarycentre de K est aussi fixe par tout élément de G,
et on n’a pas besoin de la construction précédente. De même, si on sait que G est
fini, l’isobarycentre de la famille de points g(M ), où M est n’importe quel point
du plan et g décrit G, est fixe par tout élément f de G.
Coniques
Géométrie analytique
Complexes
Homographies
Aide
Précédente
Suivante
Plein écran
Quitter
Table
Composée de réflexions (1)
Accueil
Géométrie affine
Convexité
Géométrie euclidienne
Isométries planes
Isométries de l’espace
Soit, dans le plan affine euclidien, D1 , D2 et D3 trois droites concourantes en un
point O. On note si , pour i = 1,2,3, la réflexion d’axe Di .
a) Déterminer la nature géométrique de la transformation f = s3 ◦ s2 ◦ s1 , puis de
f ◦ f.
b) Comparer les transformations f et g = s1 ◦ s2 ◦ s3 .
Coniques
Géométrie analytique
Complexes
Homographies
Aide
Précédente
Suivante
Plein écran
Quitter
Indication
Solution
f est-elle un déplacement? un antidéplacement? admet-elle des points fixes?
Accueil
Géométrie affine
Convexité
Géométrie euclidienne
Isométries planes
Isométries de l’espace
Coniques
Géométrie analytique
Complexes
Homographies
Aide
Précédente
Suivante
Plein écran
Quitter
Retour
Solution
Solution :
Accueil
Géométrie affine
Convexité
Géométrie euclidienne
Isométries planes
Isométries de l’espace
Coniques
Géométrie analytique
Complexes
Homographies
Aide
Précédente
Suivante
Plein écran
Quitter
a) La transformation f est la composée de trois antidéplacements et laisse fixe le
point O; c’est donc un antidéplacement qui admet O comme point fixe, i.e. une
réflexion d’axe passant par O. Toute réflexion étant involutive, on en déduit que
f ◦ f est l’application identique.
b) L’application réciproque de f est donc f ; mais c’est aussi (s1 )−1 ◦(s2 )−1 ◦(s3 )−1 =
s1 ◦ s2 ◦ s3 = g. On a donc f = g.
Table
Composée de réflexions (2)
Accueil
Géométrie affine
Convexité
Géométrie euclidienne
Soit, dans le plan affine euclidien, ABC un triangle, I le centre de son cercle inscrit.
Déterminer la nature géométrique de la composée f = sBI ◦ sAI ◦ sCI des réflexions
d’axes les bissectrices intérieures du triangle.
Isométries planes
Isométries de l’espace
Coniques
Géométrie analytique
Complexes
Homographies
Aide
Précédente
Suivante
Plein écran
Quitter
Indication
Solution
f est-elle un déplacement? un antidéplacement? admet-elle des points fixes? Déterminez
l’image de la droite BC par f .
Accueil
Géométrie affine
Convexité
Géométrie euclidienne
Isométries planes
Isométries de l’espace
Coniques
Géométrie analytique
Complexes
Homographies
Aide
Précédente
Suivante
Plein écran
Quitter
Retour
Solution
Accueil
Géométrie affine
Convexité
Géométrie euclidienne
Isométries planes
f est la composée de trois antidéplacements : c’est donc un antidéplacement. Le
point I est fixe par chacune des trois réflexions ; il est donc fixe par f . Il en résulte
que f est une réflexion d’axe passant par I. La droite BC est globalement invariante
par f . Or les seules droites globalement invariantes par une réflexion sont l’axe et
les droites perpendiculaires à l’axe. La droite BC ne passe pas par I : l’axe de f
est donc la perpendiculaire à BC passant par I, i.e. la droite Iα, où α est le point
de contact du cercle inscrit avec BC.
Isométries de l’espace
Coniques
Géométrie analytique
Complexes
Homographies
Aide
Précédente
Suivante
Plein écran
Quitter
Table
Composée de réflexions (3)
Accueil
Géométrie affine
Convexité
Géométrie euclidienne
Soit, dans le plan affine euclidien, ABC un triangle. Montrer que la composée
f = sAB ◦ sCA ◦ sBC des réflexions d’axes les côtés du triangle est une symétrie
glissée.
Isométries planes
Isométries de l’espace
Coniques
Géométrie analytique
Complexes
Homographies
Aide
Précédente
Suivante
Plein écran
Quitter
Indication
Solution
Constatez que f est un antidéplacement et montrez que cet antidéplacement ne
peut être une réflexion.
Accueil
Géométrie affine
Convexité
Géométrie euclidienne
Isométries planes
Isométries de l’espace
Coniques
Géométrie analytique
Complexes
Homographies
Aide
Précédente
Suivante
Plein écran
Quitter
Retour
Solution
Accueil
Géométrie affine
Convexité
Géométrie euclidienne
f est la composée de trois antidéplacements : c’est donc un antidéplacement.
Si f était une réflexion s∆ d’axe ∆, la relation sAB ◦ sCA ◦ sBC = s∆ impliquerait
sAB ◦sCA = s∆ ◦sBC . Mais sAB ◦sCA est la rotation de centre A et d’angle 2(AC,AB).
La composée s∆ ◦ sBC ne peut être une rotation de centre A et d’angle non nul que
si les droites ∆ et BC sont sécantes en A, ce qui n’est pas le cas.
f est donc une symétrie glissée.
Isométries planes
Isométries de l’espace
Coniques
Géométrie analytique
Complexes
Homographies
Aide
Précédente
Suivante
Plein écran
Quitter
Suite
Montrer que l’axe de f est la droite P R passant par les pieds des hauteurs en A
et en C du triangle ABC.
Accueil
Géométrie affine
Convexité
Géométrie euclidienne
Isométries planes
Isométries de l’espace
Coniques
Géométrie analytique
Complexes
Homographies
Aide
Précédente
Suivante
Plein écran
Quitter
Indication
Solution
Que peut-on dire de l’image de la droite P R par f ? Souvenez-vous que les côtés du
triangle ABC sont des bissectrices des angles du triangle P QR (démonstration).
Accueil
Géométrie affine
Convexité
Géométrie euclidienne
Isométries planes
Isométries de l’espace
Coniques
Géométrie analytique
Complexes
Homographies
Aide
Précédente
Suivante
Plein écran
Quitter
Solution
Accueil
Géométrie affine
L’image de la droite P R par sBC est la droite P Q ; l’image de la droite QP par sCA
est la droite QR ; l’image de la droite RQ par sAB est la droite RP . Il en résulte
que la droite RP est globalement invariante par f . Or la seule droite globalement
invariante par une symétrie glissée est son axe (démonstration).
Convexité
Géométrie euclidienne
Isométries planes
Isométries de l’espace
Coniques
Géométrie analytique
Complexes
Homographies
Aide
Précédente
Suivante
Plein écran
Quitter
Table
Droites invariantes par une symétrie glissée
Accueil
Géométrie affine
Convexité
Montrer que la seule droite globalement invariante par une symétrie glissée est son
axe.
Géométrie euclidienne
Isométries planes
Isométries de l’espace
Coniques
Géométrie analytique
Complexes
Homographies
Aide
Précédente
Suivante
Plein écran
Quitter
Indication
Solution
Que peut-on dire de la direction d’une droite globalement invariante par une transformation affine?
Accueil
Géométrie affine
Convexité
Géométrie euclidienne
Isométries planes
Isométries de l’espace
Coniques
Géométrie analytique
Complexes
Homographies
Aide
Précédente
Suivante
Plein écran
Quitter
Solution
Accueil
Géométrie affine
Convexité
Géométrie euclidienne
Isométries planes
Isométries de l’espace
Coniques
Géométrie analytique
Soit D une droite globalement invariante par une transformation affine f : f (D) =
→
−
→
−
D et ~v un vecteur directeur de D. La relation f~( D ) = D montre que f~(~v ) est proportionnel à ~v , i.e. que ~v est un vecteur propre de f~. Si f = s∆ ◦ t~u = t~u ◦ s∆ est une
→
−
symétrie glissée d’axe ∆, f~ est la réflexion vectorielle d’axe ∆. Les sous-espaces
→
−
propres de f~ sont donc ∆, associé à la valeur propre +1, et la droite vectorielle
→
−
orthogonale ∆ ⊥ , associé à la valeur propre -1. Toute droite D globalement invariante par f est donc soit parallèle à ∆, soit perpendiculaire à ∆. Mais l’image par
f d’une droite perpendiculaire à ∆ est la droite parallèle déduite par la translation
t~u et l’image d’une droite parallèle à ∆ est la droite symétrique par rapport à ∆.
La seule droite globalement invariante par f est donc l’axe ∆ de f .
Complexes
Homographies
Aide
Précédente
Suivante
Plein écran
Table
Quitter
Composée de rotations
Accueil
Géométrie affine
Convexité
Géométrie euclidienne
Soient r1 et r2 deux rotations du plan affine euclidien, de centres O1 et O2 distincts
et d’angles α1 et α2 . On suppose α1 + α2 6= 0. Donner une construction du centre
de la rotation composée r2 ◦ r1 .
Isométries planes
Isométries de l’espace
Coniques
Géométrie analytique
Complexes
Homographies
Aide
Précédente
Suivante
Plein écran
Quitter
Indication
Solution
Décomposez r1 et r2 en produit de réflexions : r1 = s2 ◦ s1 , r2 = s4 ◦ s3 . Peut-on
s’arranger pour avoir s2 = s3 ?
Accueil
Géométrie affine
Convexité
Géométrie euclidienne
Isométries planes
Isométries de l’espace
Coniques
Géométrie analytique
Complexes
Homographies
Aide
Précédente
Suivante
Plein écran
Quitter
Retour
Solution
Accueil
Géométrie affine
Convexité
Géométrie euclidienne
Isométries planes
Isométries de l’espace
Coniques
Toute rotation peut se décomposer en produit de deux réflexions, l’axe de l’une de
ces réflexions pouvant être choisi arbitrairement parmi toutes les droites passant
par le centre de la rotation et l’autre étant alors déterminé de manière unique. On
peut donc décomposer r1 et r2 sous la forme r1 = s2 ◦ s1 , r2 = s4 ◦ s3 avec s2 = s3
en prenant pour axe ∆2 = ∆3 de cette réflexion la droite O1 O2 . Les axes ∆1 et ∆4
de s1 et s4 sont alors déterminés par les relations (∆1 ,∆2 ) = α1 /2, (∆3 ,∆4 ) = α2 /2
(égalités d’angles de droites). Le point d’intersection O de ∆1 et ∆4 (ces droites
sont sécantes, puisque α1 + α2 6= 0) est fixe par s1 et s4 , donc par r2 ◦ r1 = s4 ◦ s1 :
c’est le centre de la rotation composée.
Géométrie analytique
Complexes
Homographies
Aide
Précédente
Suivante
Plein écran
Quitter
Cas particulier
Accueil
Géométrie affine
Convexité
Si α1 + α2 = 0, les droites ∆1 et ∆4 sont parallèles. Le composé r2 ◦ r1 = s4 ◦ s1
des deux rotations est alors produit de deux réflexions d’axes parallèles : c’est une
translation de vecteur 2~u, où ~u est le vecteur A1 A4 , où A1 est n’importe quel point
de ∆1 et A4 son projeté orthogonal sur ∆4 (ce vecteur ne dépend pas du choix de
A1 ).
Géométrie euclidienne
Isométries planes
Isométries de l’espace
Coniques
Géométrie analytique
Complexes
Homographies
Aide
Précédente
Suivante
Plein écran
Quitter
Table
Composée de symétries glissées
Accueil
Géométrie affine
Convexité
Géométrie euclidienne
Déterminer la nature géométrique de la composée de deux symétries glissées d’axes
perpendiculaires (on précisera les éléments caractéristiques de cette transformation).
Isométries planes
Isométries de l’espace
Coniques
Géométrie analytique
Complexes
Homographies
Aide
Précédente
Suivante
Plein écran
Quitter
Indication
Solution
Etudiez d’abord la partie linéaire de cette transformation.
Accueil
Géométrie affine
Convexité
Géométrie euclidienne
Isométries planes
Isométries de l’espace
Coniques
Géométrie analytique
Complexes
Homographies
Aide
Précédente
Suivante
Plein écran
Quitter
Solution
Accueil
Géométrie affine
Convexité
Géométrie euclidienne
Isométries planes
Soient fi = si ◦ t~ui = t~ui ◦ si (i = 1,2) les deux symétries glissées, où si est une
→
−
réflexion d’axe Di et t~ui une translation de vecteur ~ui ∈ D i . La partie linéaire ~g de
→ puisque les droites D1 et D2 sont perpendiculaires.
g = f2 ◦f1 est ~g = f~2 ◦ f~1 = −id−
E
Il en résulte que g est une symétrie centrale.
Pour déterminer son centre, il suffit de connaı̂tre l’image d’un point, par exemple
−−→
l’image O0 du point d’intersection O des droites D1 et D2 . Le vecteur OO0 est égal
−→ ~u2 − ~u1
à ~u2 − ~u1 . On en déduit que le centre Ω de g est déterminé par OΩ =
.
2
Isométries de l’espace
Coniques
Géométrie analytique
Complexes
Homographies
Aide
Précédente
Suivante
Plein écran
Table
Quitter
Le tourniquet dans le cercle
Accueil
Géométrie affine
Convexité
Géométrie euclidienne
Isométries planes
Soient A1 , A2 , A3 , A4 quatre points distincts d’un même cercle C de centre O.
La parallèle à A1 A2 menée par A4 recoupe C en un point A5 , la parallèle à A2 A3
menée par A5 recoupe C en un point A6 , la parallèle à A3 A4 menée par A6 recoupe
C en un point A7 . Montrer que A7 = A1 .
Isométries de l’espace
Coniques
Géométrie analytique
Complexes
Homographies
Aide
Précédente
Suivante
Plein écran
Quitter
Indication
Solution
Soient ∆1 , ∆2 , ∆3 les médiatrices de A1 A2 , A2 A3 , A3 A4 . Comment passe-t-on de
A1 à A2 , de A2 à A3 , . . . ?
Accueil
Géométrie affine
Convexité
Géométrie euclidienne
Isométries planes
Isométries de l’espace
Coniques
Géométrie analytique
Complexes
Homographies
Aide
Précédente
Suivante
Plein écran
Quitter
Retour
Solution
Accueil
Géométrie affine
Convexité
La médiatrice ∆1 (resp. ∆2 , ∆3 ) de A1 A2 (resp. A2 A3 , A3 A4 ) est aussi celle de
A4 A5 (resp. A5 A6 , A6 A7 ). On a donc, en notant si la réflexion d’axe ∆i (i = 1,2,3) :
A7 = s3 ◦ s2 ◦ s1 ◦ s3 ◦ s2 ◦ s1 (A1 ) = f 2 (A1 ), en notant f = s3 ◦ s2 ◦ s1 . Mais f est
un antidéplacement laissant fixe le point O, donc une réflexion d’axe passant par
O, d’où f 2 = id et A7 = A1 .
Géométrie euclidienne
Isométries planes
Isométries de l’espace
Coniques
Géométrie analytique
Complexes
Homographies
Aide
Précédente
Suivante
Plein écran
Quitter
Table
Polygone régulier
Accueil
Géométrie affine
Convexité
Géométrie euclidienne
Isométries planes
Soient n ≥ 3 un entier et A1 , A2 , . . . , An n points distincts d’un même cercle C de
centre O. On suppose A1 A2 = A2 A3 = · · · = An−1 An = An A1 . Montrer qu’il existe
une rotation r de centre O telle que A2 = r(A1 ), A3 = r(A2 ), . . . , An = r(An−1 ),
A1 = r(An ).
Isométries de l’espace
Coniques
Géométrie analytique
Complexes
Homographies
Aide
Précédente
Suivante
Plein écran
Solution
Quitter
Accueil
Géométrie affine
Convexité
Géométrie euclidienne
Isométries planes
Pour tout k = 1, . . . ,n, les points Ak−1 et Ak+1 (où An+1 = A1 et A−1 = An )
sont distincts et appartiennent au cercle C et à un même cercle de centre Ak . Ils
−−→ −−−−→
sont donc symétriques par rapport à la droite OAk et les angles (OAk ,OAk−1 ) et
−−→ −−−−→
−−→ −−−−→
(OAk ,OAk+1 ) sont opposés. Il en résulte que tous les angles (OAk ,OAk+1 ) sont
−−→ −−→
égaux. La rotation r de centre O et d’angle (OA1 ,OA2 ) transforme Ak en Ak+1
pour tout k.
Isométries de l’espace
Coniques
Géométrie analytique
Complexes
Homographies
Aide
Précédente
Suivante
Plein écran
Quitter
Suite
Accueil
Géométrie affine
Soient n ≥ 3 un entier impair et A1 , A2 , . . . , An n points distincts d’un même
Û
cercle C de centre O. On suppose les angles Ak−1
Ak Ak+1 (k = 1, . . . ,n) égaux.
Montrer qu’il existe une rotation r de centre O telle que A2 = r(A1 ), A3 = r(A2 ),
. . . , An = r(An−1 ), A1 = r(An ).
Convexité
Géométrie euclidienne
Isométries planes
Isométries de l’espace
Coniques
Géométrie analytique
Complexes
Homographies
Aide
Précédente
Suivante
Plein écran
Quitter
Solution
Accueil
Géométrie affine
Convexité
Géométrie euclidienne
Pour tout k = 1, . . . ,n, la réflexion d’axe la médiatrice de Ak Ak+1 échange les
points Ak et Ak+1 , ainsi que les demi-droites Ak Ak−1 et Ak+1 Ak+2 puisque les
Û
angles géométriques Ak−1
Ak Ak+1 et AkÛ
Ak+1 Ak+2 sont égaux et les points Ak−1 et
Ak+2 situés sur le cercle C. Il en résulte que les longueurs Ak−1 Ak et Ak+1 Ak+2 sont
égales, d’où A1 A2 = A3 A4 = · · · = An A1 = A2 A3 = · · · = An−1 An puisque n est
impair. On est donc ramené à la question précédente et le polygone est régulier.
Isométries planes
Isométries de l’espace
Coniques
Le résultat subsiste-t-il si n est pair?
Géométrie analytique
Complexes
Homographies
Aide
Précédente
Suivante
Plein écran
Quitter
Solution
Si n est pair, la longueur du côté Ak Ak+1 peut dépendre de la parité de k :
Accueil
Géométrie affine
Convexité
Géométrie euclidienne
Isométries planes
Isométries de l’espace
Coniques
Géométrie analytique
Complexes
Homographies
Aide
Précédente
Suivante
Plein écran
Quitter
Suite
Accueil
Géométrie affine
Soient A1 , A2 , . . . , An n points distincts du plan affine euclidien. On suppose tous
Û
les angles géométriques Ak−1
Ak Ak+1 (k = 1, . . . ,n) égaux et toutes les longueurs
Ak Ak+1 égales. Existe-t-il nécessairement une rotation r du plan telle que A2 =
r(A1 ), A3 = r(A2 ), . . . , An = r(An−1 ), A1 = r(An )?
Convexité
Géométrie euclidienne
Isométries planes
Isométries de l’espace
Coniques
Géométrie analytique
Complexes
Homographies
Aide
Précédente
Suivante
Plein écran
Quitter
Solution
Pas nécessairement (ce serait vrai si on avait des égalités d’angles orientés) :
Accueil
Géométrie affine
Convexité
Géométrie euclidienne
Isométries planes
Isométries de l’espace
Coniques
Géométrie analytique
Complexes
Homographies
Aide
Précédente
Suivante
Plein écran
Quitter
Table
Deux carrés (ou trois. . . )
Accueil
Géométrie affine
Convexité
Géométrie euclidienne
Soient, dans le plan affine euclidien, ABCD et AEF G deux carrés de même orientation ayant un sommet commun et P , Q, R, S les milieux respectifs des segments
BD, DE, EG et GB. Montrer que P QRS est un carré.
Isométries planes
Isométries de l’espace
Coniques
Géométrie analytique
Complexes
Homographies
Aide
Précédente
Suivante
Plein écran
Quitter
Indication
Solution
Indication :
Accueil
Géométrie affine
Convexité
−→ −−→
−→ −−→
Comparez (par exemple) les vecteurs P Q et BE (resp. P S et DG). Ne voyez-vous
−−→
−−→
pas une transformation géométrique simple qui transforme BE en DG?
Géométrie euclidienne
Isométries planes
Isométries de l’espace
Coniques
Géométrie analytique
Complexes
Homographies
Aide
Précédente
Suivante
Plein écran
Quitter
Solution
Solution :
Accueil
Géométrie affine
Convexité
Géométrie euclidienne
−→
−→
1 −−→
π
1 −−→
On a P Q = BE et P S = DG. Or la rotation r de centre A et d’angle +
2
2
−−→
−−→2
transforme B en D et E en G. Sa partie linéaire ~r transforme donc BE en DG.
−→ −→
Ces deux vecteurs sont donc orthogonaux et de même norme. De même P Q et P S.
Isométries planes
Isométries de l’espace
Coniques
Géométrie analytique
Complexes
Homographies
Aide
Précédente
Suivante
Plein écran
Quitter
Autre solution
Retour
Table
Billard polygonal
Accueil
Géométrie affine
Convexité
Géométrie euclidienne
Isométries planes
Soit, dans le plan affine euclidien, ABCD un quadrilatère convexe. On suppose
qu’il existe dans ce quadrilatère une trajectoire de billard (angle de réflexion =
angle d’incidence) fermée P QRS (voir figure). Montrer que le quadrilatère ABCD
est inscriptible.
Isométries de l’espace
Coniques
Géométrie analytique
Complexes
Homographies
Aide
Précédente
Suivante
Plein écran
Quitter
Indication
Solution
Indication :
Accueil
Géométrie affine
Convexité
Géométrie euclidienne
Interprétez l’égalité angle de réflexion = angle d’incidence en termes de réflexions
et composez ces réflexions pour comparer (par exemple) les angles de droites
(AD,AB) et (CD,CB).
Isométries planes
Isométries de l’espace
Coniques
Géométrie analytique
Complexes
Homographies
Aide
Précédente
Suivante
Plein écran
Quitter
Solution
Accueil
Géométrie affine
Convexité
Géométrie euclidienne
Isométries planes
Isométries de l’espace
La réflexion sAD d’axe AD transforme la demi-droite orientée SR en la demi-droite
orientée SR0 opposée à SP . Sa partie linéaire ~sAD transforme donc le vecteur
−→
−→
SR
PS
de cette demi-droite en le vecteur unitaire
. En continuant
unitaire ~i =
SR
PS
ainsi, on voit que la composée ~r = ~sDC ◦~sCB ◦~sBA ◦~sAD laisse fixe le vecteur ~i. Mais
~r est une rotation d’angle 2(AD,AB) + 2(CB,CD). Cet angle orienté de vecteurs
est donc nul, ce qui équivaut à l’égalité d’angles de droites (AD,AB) = (CD,CB).
Cette dernière condition exprime la cocyclicité des points ABCD.
Coniques
Géométrie analytique
Complexes
Homographies
Aide
Précédente
Suivante
Plein écran
Table
Quitter
Plus court chemin entre deux points
Accueil
Géométrie affine
Convexité
Géométrie euclidienne
Soit D une droite du plan affine euclidien et P et Q deux points du plan situés
d’un même côté de cette droite. Déterminer le point I de la droite D qui minimise
la somme P I + IQ.
Isométries planes
Isométries de l’espace
Coniques
Géométrie analytique
Complexes
Homographies
Aide
Précédente
Suivante
Plein écran
Quitter
Solution
Soit P 0 le symétrique de P par rapport à D. L’égalité P I + IQ = P 0 I + IQ, vraie
pour tout point I de D, montre qu’il s’agit de minimiser la distance P 0 Q. Le point
I est donc le point d’intersection de D et P 0 Q.
Accueil
Géométrie affine
Convexité
Q
Géométrie euclidienne
Isométries planes
Isométries de l’espace
Coniques
P
Géométrie analytique
Complexes
Homographies
I
Aide
Précédente
P’
Suivante
Plein écran
Quitter
Suite
Plus court chemin (suite)
Accueil
Géométrie affine
Convexité
Géométrie euclidienne
Isométries planes
Soient D1 et D2 deux demi-droites de même origine O du plan affine euclidien et
P et Q deux points du plan situés dans un même secteur angulaire déterminé par
ces deux demi-droites. Déterminer deux points I et J situés respectivement sur D1
et D2 tels que la somme P I + IJ + JQ soit minimale.
Isométries de l’espace
Coniques
Géométrie analytique
Complexes
Homographies
Aide
Précédente
Suivante
Plein écran
Quitter
Solution
Accueil
Géométrie affine
Convexité
Géométrie euclidienne
Isométries planes
Soit P1 le symétrique de P par rapport à D1 et Q2 le symétrique de Q par rapport
à D2 . L’égalité P I + IJ + JQ = P1 I + IJ + JQ2 , vraie pour tout point I de D1 et
tout point J de D2 , montre qu’il s’agit de minimiser la distance P1 Q2 . Les points
I et J sont donc obtenus en prenant l’intersection de la droite P1 Q2 avec les demidroites D1 et D2 . Ces intersections existent toujours si l’angle géométrique de D1
π
et D2 est inférieur à .
3
Isométries de l’espace
Coniques
Géométrie analytique
Complexes
Homographies
Aide
Précédente
Suivante
Plein écran
Quitter
Table
Le problème de Fagnano
Accueil
Géométrie affine
Convexité
Géométrie euclidienne
Isométries planes
Soit ABC un triangle du plan affine euclidien dont tous les angles sont aigus et P ,
Q, R trois points appartenant respectivement aux côtés BC, CA et AB du triangle
et distincts de ses sommets tels que la somme P Q+QR+RP soit minimale. Montrer
que les côtés du triangle ABC sont les bissectrices extérieures du triangle P QR.
Isométries de l’espace
Coniques
Géométrie analytique
Complexes
Homographies
Aide
Précédente
Suivante
Plein écran
Quitter
Indication
Solution
Fixez Q et R et déterminez le point P qui minimise P Q + P R (voir plus court
chemin)
Accueil
Géométrie affine
Convexité
Géométrie euclidienne
Isométries planes
Isométries de l’espace
Coniques
Géométrie analytique
Complexes
Homographies
Aide
Précédente
Suivante
Plein écran
Quitter
Solution
Accueil
Géométrie affine
Convexité
Géométrie euclidienne
Pour Q et R fixés, le point I de la droite BC qui minimise la somme IQ + IR est
obtenu en prenant l’intersection de la droite QR0 (où R0 est le symétrique de R par
rapport à BC) avec la droite BC (voir plus court chemin). Ce point est intérieur
au segment BC : c’est donc bien le point P . La réflexion d’axe BC échange donc
les droites P Q et P R (mais pas les demi-droites) : c’est la bissectrice extérieure en
P du triangle P QR.
Isométries planes
Isométries de l’espace
Coniques
Géométrie analytique
Complexes
Homographies
Aide
Précédente
Suivante
Plein écran
Quitter
Suite
En déduire que les points P , Q, R sont les pieds des hauteurs du triangle ABC.
Accueil
Géométrie affine
Convexité
Géométrie euclidienne
Isométries planes
Isométries de l’espace
Coniques
Géométrie analytique
Complexes
Homographies
Aide
Précédente
Suivante
Plein écran
Quitter
Indication
Solution
Déterminez l’image de la droite P R par la composée f = sAB ◦ sCA ◦ sBC des
réflexions d’axes BC, CA et AB. Quelle est la nature géométrique de f ? (Voir
composition de réflexions.)
Accueil
Géométrie affine
Convexité
Géométrie euclidienne
Isométries planes
Isométries de l’espace
Coniques
Géométrie analytique
Complexes
Homographies
Aide
Précédente
Suivante
Plein écran
Quitter
Solution
Accueil
Géométrie affine
La droite P R est globalement invariante par la composée f = sAB ◦ sCA ◦ sBC des
réflexions d’axes BC, CA et AB. C’est donc l’axe de cette symétrie glissée ; on en
déduit que P et R sont les pieds des hauteurs issues de A et C dans le triangle
ABC.
Convexité
Géométrie euclidienne
Isométries planes
Isométries de l’espace
Coniques
Géométrie analytique
Complexes
Homographies
Aide
Précédente
Suivante
Plein écran
Quitter
Suite
Accueil
Géométrie affine
Soit P , Q, R les pieds des hauteurs issues de A, B, C dans le triangle ABC, P1 et
P2 les symétriques de P par rapport à AB et AC. Montrer que les points P1 , R, Q
et P2 sont alignés et que le périmètre du triangle P QR est égal à P1 P2 . Exprimer
ce périmètre en fonction de AP et de l’angle en A du triangle ABC.
Convexité
Géométrie euclidienne
Isométries planes
Isométries de l’espace
Coniques
Géométrie analytique
Complexes
Homographies
Aide
Précédente
Suivante
Plein écran
Quitter
Solution
Accueil
Géométrie affine
Convexité
Géométrie euclidienne
Isométries planes
La droite RQ est symétrique de la droite RP (resp. QP ) par rapport à AB (resp.
CA). Il en résulte que P1 et P2 appartiennent à RQ. Les points P1 , R, Q et P2 étant
alignés dans cet ordre, le périmètre du triangle P QR est égal à P1 P2 , puisque RP1 =
c
c
RP et QP2 = QP . Mais P1 P2 = 2 AP cos((AP1 ,AP2 )/2) = 2 AP cos(A)
=, où A
−−→ −→
est l’angle en A du triangle ABC (remarquer que AP1 = AP2 = AP , (AP1 ,AB) =
−→ −→ −→ −→
−→ −−→
(AB,AP ), (AP ,AC) = (AC,AP2 )).
Isométries de l’espace
Coniques
Géométrie analytique
Complexes
Homographies
Aide
Précédente
Suivante
Plein écran
Quitter
Suite
En déduire que le triangle orthique P QR est le triangle de périmètre minimal inscrit dans le triangle ABC.
Accueil
Géométrie affine
Convexité
Géométrie euclidienne
Isométries planes
Isométries de l’espace
Coniques
Géométrie analytique
Complexes
Homographies
Aide
Précédente
Suivante
Plein écran
Quitter
Solution
Accueil
Géométrie affine
Convexité
Géométrie euclidienne
Isométries planes
La fonction (P,Q,R) 7→ P Q+QR+RP de [BC]×[CA]×[AB] dans R est continue,
donc atteint son minimum sur le compact [BC]×[CA]×[AB]. On vient de voir que
si ce minimum est atteint en des points P , Q, R distincts des sommets A, B, C,
ces points sont nécessairement les pieds des hauteurs. Si le minimum était atteint
pour R0 = A (par exemple), on aurait aussi Q0 = A et P 0 serait le pied P de la
hauteur issue de A ; le périmètre de P 0 Q0 R0 serait donc 2 AP , ce qui est supérieur
au périmètre de P QR. Le triangle P QR est donc bien le triangle de périmètre
minimal inscrit dans le triangle ABC.
Isométries de l’espace
Coniques
Géométrie analytique
Complexes
Homographies
Aide
Précédente
Suivante
Plein écran
Remarque
Quitter
Accueil
Géométrie affine
Convexité
Le triangle P QR est donc à la fois le triangle de périmètre minimal inscrit dans
le triangle ABC et le seul triangle de lumière (ou trajectoire de billard) contenu
dans ce triangle. Si l’un des angles du triangle ABC est obtus, il n’existe pas de
trajectoire de lumière dans le triangle ABC et le triangle de périmètre minimal est
aplati : c’est le triangle AP A si l’angle en A du triangle ABC est obtus.
Géométrie euclidienne
Isométries planes
Isométries de l’espace
Coniques
Géométrie analytique
Complexes
Homographies
Aide
Précédente
Suivante
Plein écran
Quitter
Table
Sous-groupes finis d’isométries
Accueil
Géométrie affine
Convexité
Géométrie euclidienne
Soit G un sous-groupe fini du groupe des isométries du plan affine euclidien E.
Montrer qu’il existe un point O fixe par tout élément de G : f (O) = O pour tout
f ∈ G.
Isométries planes
Isométries de l’espace
Coniques
Géométrie analytique
Complexes
Homographies
Aide
Précédente
Suivante
Plein écran
Quitter
Indication
Solution
Soit A un point quelconque de E. Que peut-on dire de l’isobarycentre des points
f (A), f ∈ G?
Accueil
Géométrie affine
Convexité
Géométrie euclidienne
Isométries planes
Isométries de l’espace
Coniques
Géométrie analytique
Complexes
Homographies
Aide
Précédente
Suivante
Plein écran
Quitter
Solution
Accueil
Géométrie affine
Soit A un point quelconque de E. L’isobarycentre O des points f (A), f ∈ G, est
fixe par tout élément g de G, puisque g(O) est l’isobarycentre des points g(f (A)),
pour f ∈ G (une isométrie est affine et conserve donc les barycentres) et que
l’application f 7→ g ◦ f est une bijection de G sur G.
Convexité
Géométrie euclidienne
Isométries planes
Isométries de l’espace
Coniques
Géométrie analytique
Complexes
Homographies
Aide
Précédente
Suivante
Plein écran
Quitter
Suite
En déduire que tout sous-groupe fini du groupe des déplacements du plan affine
euclidien est un groupe cyclique constitué de rotations de même centre.
Accueil
Géométrie affine
Convexité
Géométrie euclidienne
Isométries planes
Isométries de l’espace
Coniques
Géométrie analytique
Complexes
Homographies
Aide
Précédente
Suivante
Plein écran
Quitter
Solution
Accueil
Géométrie affine
Convexité
Géométrie euclidienne
Isométries planes
Isométries de l’espace
Coniques
Géométrie analytique
Complexes
Soit G un sous-groupe fini du groupe des déplacements du plan affine euclidien E.
Il existe un point O de E fixe par tout élément de G. Tout élément de G est donc
une rotation de centre O ou l’identité.
Soit n l’ordre de G. Tout élément g de G vérifie g n = idE . L’angle α de la rotation
2kπ
g vérifie donc nα ≡ 0 (mod 2π) : il existe donc k ∈ {0, . . . , n − 1} tel que α =
.
n
Le groupe G est donc constitué de toutes les rotations de centre O et d’angles
2kπ
, k ∈ {0, . . . , n − 1} : c’est le groupe cyclique engendré par la rotation r de
n
2π
.
centre O et d’angle
n
Homographies
Aide
Précédente
Suivante
Plein écran
Quitter
Suite
Accueil
Géométrie affine
Convexité
Montrer que tout sous-groupe fini G du groupe des isométries du plan affine euclidien E est :
• soit un groupe cyclique constitué de rotations de même centre ;
• soit un groupe diédral.
Géométrie euclidienne
Isométries planes
Isométries de l’espace
Coniques
Géométrie analytique
Complexes
Homographies
Aide
Précédente
Suivante
Plein écran
Quitter
Solution
Accueil
Géométrie affine
Convexité
Géométrie euclidienne
Isométries planes
Isométries de l’espace
Coniques
Géométrie analytique
Complexes
Si G n’est constitué que de déplacements, c’est un groupe cyclique constitué de
rotations de même centre.
Sinon soit O un point de E fixe par tout élément de G. Tout élément de G qui
n’est pas un déplacement est une réflexion d’axe passant par O. Soit s une telle
réflexion. Le groupe G+ des déplacements de G (intersection de G et du groupe
des déplacements du plan) est un groupe cyclique de rotations de centre O et G
est la réunion disjointe G+ ∪ G− , où G− = sG+ est l’ensemble des réflexions s ◦ g,
g ∈ G+ . Si r est un générateur de G+ , G est engendré par r et s qui vérifient les
relations s2 = rn = idE , s ◦ r = r−1 ◦ s = rn−1 ◦ s, où n est l’ordre de G+ . En effet
s ◦ r est une réflexion et vérifie donc s ◦ r = (s ◦ r)−1 = r−1 ◦ s. Le groupe G est le
groupe des isométries d’un polygone régulier à n sommets.
Homographies
Aide
Précédente
Suivante
Plein écran
Quitter
Table
Composée de trois réflexions
Accueil
Géométrie affine
Convexité
Géométrie euclidienne
Donner une condition nécessaire et suffisante sur trois plans P1 , P2 , P3 de l’espace
affine euclidien pour que la composée s3 ◦ s2 ◦ s1 des réflexions de plans P1 , P2 , P3
soit une réflexion.
Isométries planes
Isométries de l’espace
Coniques
Géométrie analytique
Complexes
Homographies
Aide
Précédente
Suivante
Plein écran
Quitter
Indication
Solution
L’égalité s3 ◦ s2 ◦ s1 = s peut encore s’écrire s2 ◦ s1 = s3 ◦ s. Qu’est-ce que la
composée de deux réflexions?
Accueil
Géométrie affine
Convexité
Géométrie euclidienne
Isométries planes
Isométries de l’espace
Coniques
Géométrie analytique
Complexes
Homographies
Aide
Précédente
Suivante
Plein écran
Quitter
Retour
Solution
Accueil
Géométrie affine
Convexité
Géométrie euclidienne
Isométries planes
Isométries de l’espace
Coniques
Géométrie analytique
Complexes
Homographies
Aide
Précédente
La composée de deux réflexions est :
• une translation de vecteur orthogonal aux plans de ces réflexions si ceux-ci
sont parallèles ;
• une rotation d’axe l’intersection des plans de ces réflexions si ceux-ci sont
sécants.
Si s = s3 ◦ s2 ◦ s1 est une réflexion de plan P , l’égalité s2 ◦ s1 = s3 ◦ s montre que :
• si P1 et P2 sont parallèles, s2 ◦s1 est une translation de vecteur ~u orthogonal à P1
et P2 ; P3 et P doivent donc être parallèles et orthogonaux à ~u ; en particulier,
P3 est parallèle à P1 et P2 ;
• si P1 et P2 sont sécants, s2 ◦ s1 est une rotation d’axe ∆ = P1 ∩ P2 ; P3 et P
doivent donc être sécants, d’intersection ∆ ; en particulier, P3 contient ∆, et
les plans P1 , P2 , P3 ont une droite commune.
Réciproquement, si P1 , P2 , P3 sont parallèles, s est une réflexion de plan parallèle
à P1 , P2 , P3 . De même, si P1 , P2 , P3 ont une droite commune ∆, s est une réflexion
de plan passant par ∆.
Suivante
Plein écran
Quitter
Table
Caractérisation de l’axe d’un vissage
Accueil
Géométrie affine
Convexité
Géométrie euclidienne
Soit, dans l’espace affine euclidien E de dimension 3, f un vissage d’axe D. Montrer
−−−−−→
que D est l’ensemble des points M de E tels que le vecteur M f (M ) appartienne à
−
→
D et que c’est aussi l’ensemble des points M de E minimisant la distance M f (M ).
Isométries planes
Isométries de l’espace
Coniques
Géométrie analytique
Complexes
Homographies
Aide
Précédente
Suivante
Plein écran
Quitter
Solution
Accueil
Géométrie affine
Convexité
Géométrie euclidienne
Isométries planes
Isométries de l’espace
Coniques
Géométrie analytique
Soit t = r ◦ t~u = t~u ◦ r la décomposition de f en produit commutatif d’une rotation
→
−
r d’axe D et d’une translation de vecteur ~u ∈ D . Pour tout point M de E, le
−−−−−→
vecteur M f (M ) s’écrit
−−−−−→ −−−−−→ −−−−−−−→ −−−−−→
M f (M ) = M r(M ) + r(M )f (M ) = M r(M ) + ~u
−−−−−→
−−−−−→
→
−
et le vecteur M r(M ) est orthogonal à D . Le vecteur M f (M ) appartient donc à
−−−−−→ −
−
→
→
D si et seulement si M r(M ) = 0 , i.e. si et seulement si M = r(M ), ce qui signifie
que M appartient à D.
−−−−−→
Les vecteurs ~u et M r(M ) étant orthogonaux, on a
Complexes
Homographies
Aide
M f (M )2 = M r(M )2 + k~uk2 ≥ k~uk2
pour tout point M de E et M f (M ) = k~uk si et seulement si M r(M ) = 0, ce qui
signifie encore que M appartient à D.
Précédente
Suivante
Plein écran
Quitter
Table
Isométries des tétraèdres équifaciaux
Accueil
Géométrie affine
Convexité
Géométrie euclidienne
Isométries planes
Soit ABCD un tétraèdre non aplati. On note I,I 0 ,J,J 0 ,K,K 0 les milieux respectifs
des arêtes AB, CD, BC, DA, CA et BD et on appelle bimédianes les droites II 0 ,
JJ 0 et KK 0 joignant les milieux de deux arêtes opposées.
a) Montrer que les trois bimédianes sont concourantes.
Isométries de l’espace
Coniques
Géométrie analytique
Complexes
Homographies
Aide
Précédente
Suivante
Plein écran
Quitter
Solution
Suite
L’isobarycentre G des points A, B, C, D est le milieu commun de II 0 , JJ 0 et KK 0 .
Accueil
Géométrie affine
Convexité
Géométrie euclidienne
Isométries planes
Isométries de l’espace
Coniques
Géométrie analytique
Complexes
Homographies
Aide
Précédente
Suivante
Plein écran
Quitter
Retour
Suite
b) On suppose la bimédiane II 0 orthogonale aux deux arêtes AB et CD dont
elle joint les milieux. Déterminer l’image du tétraèdre ABCD par le retournement
d’axe II 0 . En déduire que les arêtes BC et AD ont même longueur.
Accueil
Géométrie affine
Convexité
Géométrie euclidienne
Isométries planes
Isométries de l’espace
Coniques
Géométrie analytique
Complexes
Homographies
Aide
Précédente
Suivante
Plein écran
Quitter
Solution
Suite
Le retournement d’axe II 0 échange les points A et B, ainsi que les points C et D.
Elle échange donc les segments BC et AD, qui ont donc même longueur.
Accueil
Géométrie affine
Convexité
Géométrie euclidienne
Isométries planes
Isométries de l’espace
Coniques
Géométrie analytique
Complexes
Homographies
Aide
Précédente
Suivante
Plein écran
Quitter
Retour
Suite
On suppose désormais que chacune des trois bimédianes est orthogonale aux deux
arêtes dont elle joint les milieux.
Accueil
Géométrie affine
Convexité
Géométrie euclidienne
Isométries planes
c) Déterminer la composée des retournements d’axes II 0 et JJ 0 (on pourra déterminer
les images des sommets du tétraèdre par cette transformation). En déduire que les
trois bimédianes sont deux à deux orthogonales et que les quatre faces du tétraèdre
ABCD sont des triangles isométriques.
Isométries de l’espace
Coniques
C
Géométrie analytique
K
Complexes
A
Homographies
I’
J
J’
Aide
I
D
Précédente
K’
Suivante
B
Plein écran
Quitter
Solution
Suite
Accueil
Géométrie affine
Convexité
Géométrie euclidienne
Isométries planes
La composée sJJ 0 ◦ sII 0 des retournements d’axes II 0 et JJ 0 échange A et C d’une
part, B et D d’autre part. C’est donc le retournement d’axe KK 0 . Mais la composée de deux retournements n’est un retournement que si les axes de ces deux
retournements sont perpendiculaires. On en déduit que II 0 et JJ 0 sont perpendiculaires ; de même II 0 et KK 0 , JJ 0 et KK 0 .
On avait déjà remarqué que BC = AD ; on a de même AB = CD et AC = BD.
Il en résulte que les quatre faces du tétraèdre sont des triangles isométriques.
Isométries de l’espace
Coniques
C
Géométrie analytique
K
Complexes
A
Homographies
I’
J
J’
Aide
I
D
Précédente
K’
Suivante
B
Plein écran
Quitter
Retour
Suite
Accueil
Géométrie affine
Convexité
d) Soit (x,y,z) les coordonnées du point A dans un repère orthonormé dont les
axes sont portés par les droites II 0 , JJ 0 et KK 0 . Exprimer les coordonnées dans
ce repère des points B, C et D. Montrer que les sommets du tétraèdre ABCD
constituent avec leurs symétriques A0 B 0 C 0 D0 par rapport à l’origine de ce repère
les sommets d’un parallélipipède rectangle.
Géométrie euclidienne
Isométries planes
C
Isométries de l’espace
Coniques
A
Géométrie analytique
Complexes
Homographies
Aide
D
Précédente
B
Suivante
Plein écran
Quitter
Solution
Table
Accueil
Géométrie affine
Convexité
Géométrie euclidienne
B est l’image de A par le retournement d’axe II 0 . Si (x,y,z) sont les coordonnées
du point A dans un repère orthonormé dont les axes sont portés par les droites II 0 ,
JJ 0 et KK 0 , les coordonnées de B dans ce repère sont donc (x, − y, − z). De même
celles de C sont (−x, − y,z) et celles de D (−x,y, − z). Il en résulte que les points
A, B, C et D constituent avec leurs symétriques A0 , B 0 , C 0 et D0 par rapport à
l’origine de ce repère les sommets d’un parallélipipède rectangle.
Isométries planes
Isométries de l’espace
C
Coniques
Géométrie analytique
A
Complexes
Homographies
Aide
D
Précédente
Suivante
B
Plein écran
Quitter
Retour
Remarque
Table
Accueil
Géométrie affine
Convexité
Géométrie euclidienne
Isométries planes
Isométries de l’espace
Coniques
Il en résulte que tout tétraèdre équifacial peut être construit à partir d’un parallélipipède rectangle. Si les longueurs des arêtes de √
ce parallélipipède√sont 2x,
2y et √
2z, celles des arêtes du tétraèdre sont a = 2 y 2 + z 2 , b = 2 z 2 + x2 ,
c = 2 x2 + y 2 . Les faces de ce tétraèdre sont donc des triangles dont tous les
angles sont aigus, puisque a2 < b2 + c2 , b2 < c2 + a2 , c2 < a2 + b2 . Réciproquement,
pour tout triangle acutangle, on peut construire un tétraèdre équifacial√dont les
faces √
sont isométriques
à ce triangle (il suffit de résoudre le système a = 2 y 2 + z 2 ,
√
b = 2 z 2 + x2 , c = 2 x2 + y 2 , dont on montre facilement qu’il admet une solution
positive et une seule).
Géométrie analytique
Complexes
Homographies
Aide
Précédente
Suivante
Plein écran
Quitter
Retour
Table
Isométries du tétraèdre régulier
Accueil
Géométrie affine
Convexité
Géométrie euclidienne
Soit, dans l’espace affine E de dimension 3, ABCD un tétraèdre régulier et G le
groupe des isométries conservant globalement ce tétraèdre :
G = {f ∈ Is(E) | {f (A),f (B),f (C),f (D)} = {A,B,C,D} } .
Isométries planes
Isométries de l’espace
Coniques
Géométrie analytique
Montrer que l’application ϕ de G dans le groupe des permutations de {A,B,C,D}
qui associe à un élément f de G sa restriction à {A,B,C,D} est un homomorphisme
injectif de groupes.
Complexes
Homographies
Aide
Précédente
Suivante
Plein écran
Quitter
Solution
Accueil
Géométrie affine
L’application ϕ est clairement un homomorphisme de groupes. Cet homomorphisme est injectif puisque les points A, B, C, D constituent un repère affine
de E : si deux isométries f et g vérifient f (A) = g(A), f (B) = g(B), f (C) = g(C),
f (D) = g(D), alors f = g (une isométrie est une transformation affine).
Convexité
Géométrie euclidienne
Isométries planes
Isométries de l’espace
Coniques
Géométrie analytique
Complexes
Homographies
Aide
Précédente
Suivante
Plein écran
Quitter
Suite
Montrer que pour tout couple de sommets du tétraèdre ABCD, il existe une
réflexion qui échange ces sommets et laisse les deux autres fixes.
Accueil
Géométrie affine
Convexité
Géométrie euclidienne
Isométries planes
Isométries de l’espace
Coniques
Géométrie analytique
Complexes
Homographies
Aide
Précédente
Suivante
Plein écran
Quitter
Indication
Solution
Quel est le plan médiateur de AB ?
Accueil
Géométrie affine
Convexité
Géométrie euclidienne
Isométries planes
Isométries de l’espace
Coniques
Géométrie analytique
Complexes
Homographies
Aide
Précédente
Suivante
Plein écran
Quitter
Retour
Solution
Accueil
Géométrie affine
Soit I le milieu de AB. Les trois points I, C et D sont équidistants de A et B et
ne sont pas alignés. Le plan CDI est donc le plan médiateur de AB. La réflexion
par rapport à ce plan échange les points A et B et laisse fixes C et D.
On obtient ainsi 6 réflexions conservant le tétraèdre.
Convexité
Géométrie euclidienne
Isométries planes
Isométries de l’espace
Coniques
Géométrie analytique
Complexes
Homographies
Aide
Précédente
Suivante
Plein écran
Quitter
Suite
En déduire que l’homomorphisme ϕ est surjectif.
Accueil
Géométrie affine
Convexité
Géométrie euclidienne
Isométries planes
Isométries de l’espace
Coniques
Géométrie analytique
Complexes
Homographies
Aide
Précédente
Suivante
Plein écran
Quitter
Solution
Accueil
Géométrie affine
Convexité
Géométrie euclidienne
L’image de ϕ est un sous-groupe du groupe des permutations des sommets et
ce sous-groupe contient toutes les transpositions (permutations échangeant deux
éléments et laissant les autres fixes). Comme les transpositions engendrent le groupe
symétrique, l’image de ϕ est le groupe de toutes les permutations des sommets :
ϕ est donc un isomorphisme du groupe G sur le groupe des permutations d’un
ensemble à 4 éléments.
Isométries planes
Isométries de l’espace
Coniques
Géométrie analytique
Complexes
Homographies
Aide
Précédente
Suivante
Plein écran
Quitter
Suite
Montrer que le sous-groupe G+ des déplacements du tétraèdre est isomorphe au
groupe des permutations de signature +1 des sommets (groupe alterné).
Accueil
Géométrie affine
Convexité
Géométrie euclidienne
Isométries planes
Isométries de l’espace
Coniques
Géométrie analytique
Complexes
Homographies
Aide
Précédente
Suivante
Plein écran
Quitter
Solution
Accueil
Géométrie affine
Convexité
Géométrie euclidienne
Isométries planes
Isométries de l’espace
Soit ε l’homomorphisme du groupe des permutations des sommets dans {+1, − 1}
qui à toute permutation associe sa signature, et ψ l’homomorphisme de G dans
{+1, − 1} qui à toute isométrie du tétraèdre associe le déterminant de sa partie
linéaire : f 7→ det(f~). Les homomorphismes ε ◦ ϕ et ψ coı̈ncident sur l’ensemble des
réflexions de plans les plans médiateurs des arêtes ; mais ces réflexions engendrent
G puisque leurs images par ϕ engendrent le groupe symétrique. Il en résulte que
ε ◦ ϕ = ψ. Les déplacements de G correspondent donc par ϕ aux permutations
paires et G+ est isomorphe au groupe alterné.
Coniques
Géométrie analytique
Complexes
Homographies
Aide
Précédente
Suivante
Plein écran
Quitter
Suite
Déterminer tous les éléments de G+ (on montrera que tous ces déplacements sont
des rotations et on précisera leur axe et leur angle).
Accueil
Géométrie affine
Convexité
Géométrie euclidienne
Isométries planes
Isométries de l’espace
Coniques
Géométrie analytique
Complexes
Homographies
Aide
Précédente
Suivante
Plein écran
Quitter
Solution
Accueil
Géométrie affine
Convexité
Géométrie euclidienne
Isométries planes
Tous les éléments de G conservent l’isobarycentre O de ABCD. Il en résulte que
les éléments de G+ autres que l’identité sont des rotations d’axe passant par O.
On trouve 3 retournements d’axes les bimédianes (droites joignant les milieux de
deux arêtes opposées : elles sont perpendiculaires à ces deux arêtes) et 8 rotations
2π
d’angle ±
d’axes les droites joignant un sommet à l’isobarycentre de la face
3
opposée.
Isométries de l’espace
Coniques
Géométrie analytique
Complexes
Homographies
Aide
Précédente
Suivante
Plein écran
Quitter
Suite
Déterminer tous les éléments de l’ensemble G− des antidéplacements conservant le
tétraèdre.
Accueil
Géométrie affine
Convexité
Géométrie euclidienne
Isométries planes
Isométries de l’espace
Coniques
Géométrie analytique
Complexes
Homographies
Aide
Précédente
Suivante
Plein écran
Quitter
Solution
Accueil
Géométrie affine
Convexité
On a déjà trouvé 6 réflexions dans G− (les plans de ces réflexions sont les plans
médiateurs des arêtes). Il reste donc 6 autres éléments dans G− . Ces antidéplacements
correspondent par ϕ aux 6 permutations circulaires des sommets. Ce sont les antirotations d’axes les bimédianes et d’angles droits (remarquer que le plan perpendiculaire en O à une bimédiane est le plan contenant les deux autres bimédianes).
Géométrie euclidienne
Isométries planes
Isométries de l’espace
Coniques
Géométrie analytique
Complexes
Homographies
Aide
Précédente
Suivante
Plein écran
Quitter
Table
Isométries du cube
Accueil
Géométrie affine
Convexité
Géométrie euclidienne
Isométries planes
Soit, dans l’espace affine euclidien E de dimension 3, ABCDA0 B 0 C 0 D0 un cube
de centre O et G le groupe des isométries conservant globalement l’ensemble des
sommets de ce cube. Montrer que l’image par tout élément de G d’une diagonale
du cube (i.e. l’une des droites AA0 , BB 0 , CC 0 , DD0 joignant un sommet au sommet
symétrique par rapport à O) est une diagonale du cube.
Isométries de l’espace
Coniques
Géométrie analytique
Complexes
Homographies
Aide
Précédente
Suivante
Plein écran
Solution
Quitter
Suite
Tout élément de G conserve l’isobarycentre O des sommets du cube et transforme
une droite en une droite. L’image d’une diagonale est donc une diagonale.
Accueil
Géométrie affine
Convexité
Géométrie euclidienne
Isométries planes
Isométries de l’espace
Coniques
Géométrie analytique
Complexes
Homographies
Aide
Précédente
Suivante
Plein écran
Quitter
Suite
Soit ϕ l’application de G dans le groupe des permutations des diagonales du cube
qui à une isométrie f associe la permutation des diagonales induite par f . Montrer
que ϕ est un homomorphisme de groupes. Déterminer son noyau.
Accueil
Géométrie affine
Convexité
Géométrie euclidienne
Isométries planes
Isométries de l’espace
Coniques
Géométrie analytique
Complexes
Homographies
Aide
Précédente
Suivante
Plein écran
Quitter
Indication
Solution
Si un élément f de G conserve globalement chaque diagonale du cube, les vecteurs
−→ −−→ −→ −−→
OA, OB, OC, OD sont vecteurs propres de f~. Que peut-on dire des valeurs propres
correspondantes?
Accueil
Géométrie affine
Convexité
Géométrie euclidienne
Isométries planes
Isométries de l’espace
Coniques
Géométrie analytique
Complexes
Homographies
Aide
Précédente
Suivante
Plein écran
Quitter
Retour
Solution
Accueil
Géométrie affine
Convexité
Géométrie euclidienne
Isométries planes
Isométries de l’espace
Coniques
Géométrie analytique
Complexes
Homographies
L’application ϕ est clairement un homomorphisme de groupes. Un élément f de
G appartient au noyau de cet homomorphisme si et seulement si il conserve glo−→ −−→
balement chaque diagonale du cube, i.e. si et seulement si les vecteurs OA, OB,
−→ −−→
OC, OD sont vecteurs propres de f~. Les valeurs propres associées sont ±1, puisque
f~ est une transformation orthogonale. Si deux de ces vecteurs étaient associés à
la valeur propre +1 et deux à la valeur propre -1, les deux sous-espaces propres
correspondants seraient de dimension supérieure ou égale à 2 et leur intersection
→
−
non réduite à { 0 }, ce qui est impossible. Trois de ces vecteurs sont donc associés
→
−
à la même valeur propre, et le sous-espace propre correspondant est donc E , ce
→ . Le noyau de ϕ est donc constitué de l’identité de E et
qui montre que f~ est ±id−
E
de la symétrie de centre O.
Aide
Précédente
Suivante
Plein écran
Quitter
Suite
Montrer que la restriction de ϕ au sous-groupe G+ des rotations du cube est un
isomorphisme de G+ sur le groupe des permutations des diagonales.
Accueil
Géométrie affine
Convexité
Géométrie euclidienne
Isométries planes
Isométries de l’espace
Coniques
Géométrie analytique
Complexes
Homographies
Aide
Précédente
Suivante
Plein écran
Quitter
Retour
Solution
Accueil
Géométrie affine
Convexité
Géométrie euclidienne
Isométries planes
Isométries de l’espace
Coniques
Géométrie analytique
Complexes
Homographies
Aide
Précédente
Suivante
La restriction de ϕ au sous-groupe G+ des déplacements du cube est un homomorphisme injectif de groupes, puisque son noyau est réduit à idE . Pour montrer
que cet homomorphisme est surjectif, on pourrait montrer que son image contient
les transpositions, puisque celles-ci engendrent le groupe symétrique : il suffit pour
cela de remarquer que les retournements d’axes les droites joignant les milieux de
deux arêtes opposées échangent les diagonales joignant les extrémités de ces arêtes
et laissent les deux autres diagonales globalement invariantes. Une autre méthode
consiste à expliciter 24 rotations laissant le cube invariant. On trouve en effet :
• 4 axes de rotation d’ordre 3 (les diagonales) ;
• 6 axes de rotation d’ordre 2 (les droites joignant les milieux de deux arêtes
opposées) ;
• 3 axes de rotation d’ordre 4 (les droites joignant les centres de deux faces
opposées)
2π
π
soit 8 rotations d’angle ± , 6 rotations d’angle ± , 9 retournements et l’identité.
3
2
Les figures suivantes représentent un axe de chacun de ces types et la projection
orthogonale du cube dans la direction de cet axe.
Plein écran
Quitter
Suite
Accueil
Géométrie affine
Convexité
Géométrie euclidienne
Isométries planes
Isométries de l’espace
Coniques
Géométrie analytique
Complexes
Homographies
Aide
Précédente
Suivante
Plein écran
Quitter
Suite
Montrer que le groupe G des isométries du cube est isomorphe au produit direct
du groupe des permutations de 4 éléments par un groupe à 2 éléments.
Accueil
Géométrie affine
Convexité
Géométrie euclidienne
Isométries planes
Isométries de l’espace
Coniques
Géométrie analytique
Complexes
Homographies
Aide
Précédente
Suivante
Plein écran
Quitter
Solution
Accueil
Géométrie affine
Convexité
Géométrie euclidienne
Isométries planes
Isométries de l’espace
Coniques
Géométrie analytique
Complexes
Le groupe G des isométries du cube est le produit direct de son sous-groupe G+ par
le sous-groupe à deux éléments {idE , sO ), où sO est la symétrie par rapport à O.
En effet l’application f 7→ f ◦ sO est une bijection de G+ sur l’ensemble G− des antidéplacements du cube et sO commute avec tout élément de G (plus généralement
avec toute transformation affine laissant O invariant). On remarque en particulier
que le cube admet 9 plans de symétrie, qui sont les plans perpendiculaires en O
aux axes des retournements de G (f ◦ sO est une réflexion si et seulement si f est
un retournement).
On peut aussi retrouver et identifier tous les éléments de G en écrivant les matrices
de leurs parties linéaires dans un repère orthonormé porté par les perpendiculaires
aux faces issues de O.
Homographies
Aide
Précédente
Suivante
Plein écran
Quitter
Table
Cube, tétraèdres et octaèdre
Accueil
Géométrie affine
Convexité
Géométrie euclidienne
Isométries planes
Soit, dans l’espace affine euclidien E de dimension 3, C = ABCDA0 B 0 C 0 D0 un
cube de centre O. Montrer que les tétraèdres T = ACB 0 D0 et T 0 = A0 C 0 BD sont
réguliers et que toute isométrie laissant le cube globalement invariant conserve
globalement chacun de ces tétraèdres ou les échange.
Isométries de l’espace
Coniques
Géométrie analytique
Complexes
Homographies
Aide
Précédente
Suivante
Plein écran
Quitter
Solution
Accueil
Géométrie affine
Convexité
Les tetraèdres T et T 0 sont évidemment réguliers, puisque leurs arêtes sont toutes
des diagonales des faces du cube et ont donc même longueur. Si une isométrie f
conserve le cube, soit f (A) est un sommet de T et f (T ) = T , f (T 0 ) = T 0 , soit
f (A) est un sommet de T 0 et f (T ) = T 0 , f (T 0 ) = T (une isométrie conserve les
longueurs).
Géométrie euclidienne
Isométries planes
Isométries de l’espace
Coniques
Géométrie analytique
Complexes
Homographies
Aide
Précédente
Suivante
Plein écran
Quitter
Suite
Soit G le groupe des isométries de E laissant le cube C globalement invariant :
G = {f ∈ Is(E) | f (C) = C}
Accueil
Géométrie affine
Convexité
Géométrie euclidienne
et H le sous-groupe de G constitué des isométries laissant T globalement invariant :
H = {f ∈ G | f (T ) = T } .
Isométries planes
Isométries de l’espace
Coniques
Géométrie analytique
Complexes
Homographies
Montrer que H est le groupe de toutes les isométries de E laissant T globalement
invariant :
H = {f ∈ Is(E) | f (T ) = T }
et que G est produit direct de H et du sous-groupe à deux éléments {idE , sO }, où
sO est la symétrie de centre O.
Aide
Précédente
Suivante
Plein écran
Quitter
Solution
Accueil
Géométrie affine
Par définition, tout élément de H est une isométrie laissant le tétraèdre T globalement invariant. Réciproquement, si une isométrie f de E laisse T globalement
invariant, elle laisse fixe l’isobarycentre O de ses sommets et commute donc avec
sO . Il en résulte que
Convexité
Géométrie euclidienne
Isométries planes
Isométries de l’espace
Coniques
Géométrie analytique
Complexes
Homographies
Aide
Précédente
f (T 0 ) = f (sO (T )) = sO (f (T )) = sO (T ) = T 0
d’où f (C) = C : f appartient donc à G.
Le sous-ensemble sO H = {sO ◦ f | f ∈ H} de G est l’ensemble des isométries de C
échangeant T et T 0 : G est donc la réunion disjointe de H et de sO H. On a déjà
remarqué que sO commutait avec tout élément de G.
Remarque : il ne faut pas en conclure que H est le groupe des déplacements du
cube : il est facile de trouver une rotation du cube qui échange T et T 0 . On peut
cependant en déduire que G est un groupe à 48 éléments, isomorphe au produit
direct du groupe des permutations de 4 éléments par un groupe à 2 éléments.
Suivante
Plein écran
Quitter
Suite
Montrer que le groupe G est aussi le groupe des isométries de E laissant globalement invariant l’octaèdre régulier O de sommets les centres des faces de C (ces
centres sont aussi les milieux des arêtes de chacun des deux tétraèdres T et T 0 ).
Accueil
Géométrie affine
Convexité
Géométrie euclidienne
Isométries planes
Isométries de l’espace
Coniques
Géométrie analytique
Complexes
Homographies
Aide
Précédente
Suivante
Plein écran
Quitter
Solution
Accueil
Géométrie affine
Convexité
Géométrie euclidienne
Il est clair que tout élément de G transforme un sommet de O en un sommet de O
(toutes isométrie du cube transforme une face en une face). Réciproquement toute
isométrie conservant globalement O conserve aussi le cube C (on peut par exemple
remarquer que les 6 centres de gravité des faces de O sont les sommets d’un cube C 0
image de C par l’homothétie de centre O et de rapport 1/3 et que toute isométrie
conservant O conserve globalement C 0 , donc aussi C).
Isométries planes
Isométries de l’espace
Coniques
Géométrie analytique
Complexes
Homographies
Aide
Précédente
Suivante
Plein écran
Quitter
Table
Isométries de l’hélice circulaire
Accueil
Géométrie affine
Convexité
Géométrie euclidienne
Isométries planes
Isométries de l’espace
Coniques
Géométrie analytique
→
−
Soit, dans l’espace affine euclidien E de dimension 3, D une droite, ~u ∈ D un
vecteur directeur non nul de cette droite et α un réel non nul. On note, pour
tout réel λ, rλ la rotation d’axe D (orienté par ~u) et d’angle λα, et fλ = rλ ◦ tλ~u
la composée de rλ et de la translation de vecteur λ~u. Montrer que l’application
λ 7→ fλ est un homomorphisme injectif du groupe additif (R,+) dans le groupe
Is+ (E) des déplacements de E. Préciser la nature géométrique de fλ (on discutera
suivant la valeur de λ).
Complexes
Homographies
Aide
Précédente
Suivante
Plein écran
Quitter
Solution
Accueil
Géométrie affine
Convexité
Géométrie euclidienne
Isométries planes
Isométries de l’espace
Coniques
Géométrie analytique
Complexes
Homographies
Pour tout réel λ, la rotation rλ et la translation de vecteur λ~u commutent, puisque
le vecteur λ~u appartient à la direction de l’axe de rλ . La composée fλ = rλ ◦ tλ~u =
tλ~u ◦ rλ de ces deux transformations est donc :
• un vissage si λα 6≡ 0 (mod 2π)
• une translation si λα ≡ 0 (mod 2π).
La relation
fλ ◦ fµ = rλ ◦ tλ~u ◦ rµ ◦ tµ~u = rλ ◦ rµ ◦ tλ~u ◦ tµ~u = rλ+µ ◦ t(λ+µ)~u = fλ+µ
montre que l’application λ 7→ fλ est un homomorphisme de groupes de (R,+) dans
le groupe Is+ (E) des déplacements de E. Le noyau de cet homomorphisme est
clairement réduit à 0 : il est donc injectif.
Aide
Précédente
Suivante
Plein écran
Quitter
Suite
Accueil
Géométrie affine
Soit A un point de D et ∆ une droite perpendiculaire en A à D. On note, pour
tout réel λ, Aλ = fλ (A) l’image de A par fλ , ∆λ = fλ (∆) et sλ le retournement
d’axe ∆λ . Montrer que, pour tout λ ∈ R, on a sλ = fλ ◦ s0 ◦ fλ−1 . Déterminer sλ ◦ s0
(où s0 est le retournement d’axe ∆0 = ∆).
Convexité
Géométrie euclidienne
Isométries planes
Isométries de l’espace
Coniques
Géométrie analytique
Complexes
Homographies
Aide
Précédente
Suivante
Plein écran
Quitter
Solution
Accueil
Géométrie affine
Convexité
Géométrie euclidienne
Isométries planes
Isométries de l’espace
Coniques
Géométrie analytique
Complexes
L’isométrie g = fλ ◦ s0 ◦ fλ−1 est un déplacement involutif (g ◦ g = idE ), c’est donc
un retournement. Son axe est l’ensemble de ses points fixes. Or un point M est
fixe par g si et seulement si g(M ) = fλ ◦ s0 ◦ fλ−1 (M ) = M , i.e. si et seulement si
s0 ◦ fλ−1 (M ) = fλ−1 (M ), ce qui équivaut à fλ−1 (M ) ∈ ∆, ou encore à M ∈ fλ (∆).
−−→
→
→
La partie linéaire −
s ◦−
s0 = −
s−
λ ◦ s0 de sλ ◦ s0 est la rotation vectorielle d’axe D et
→−
−
→ λ
d’angle 2( ∆,∆λ ) = 2λα. L’image de A par sλ ◦ s0 est le point symétrique de A par
−−→
rapport à Aλ , i.e. le point déduit de A par la translation de vecteur 2AAλ = 2λ~u.
On en déduit que sλ ◦ s0 est le vissage d’axe D, d’angle 2λα et de vecteur 2λ~u, i.e.
sλ ◦ s0 = f2λ .
Homographies
Aide
Précédente
Suivante
Plein écran
Quitter
Suite
En déduire que G = {fλ | λ ∈ R} ∪ {sλ | λ ∈ R} est un sous-groupe du groupe des
déplacements de E.
Déterminer sλ ◦ fµ , fµ ◦ sλ , sλ ◦ sµ pour tout couple (λ,µ) de réels.
Accueil
Géométrie affine
Convexité
Géométrie euclidienne
Isométries planes
Isométries de l’espace
Coniques
Géométrie analytique
Complexes
Homographies
Aide
Précédente
Suivante
Plein écran
Quitter
Solution
sλ = f2λ ◦ s0 étant un retournement, il est égal à son propre inverse, ce qui peut
encore s’écrire f2λ ◦ s0 = s0 ◦ f−2λ . Il en résulte :
Accueil
Géométrie affine
Convexité
Géométrie euclidienne
Isométries planes
Isométries de l’espace
Coniques
Géométrie analytique
Complexes
sλ ◦ fµ = f2λ ◦ s0 ◦ fµ = f2λ−µ
fµ ◦ sλ = fµ+2λ ◦ s0
sλ ◦ sµ = f2λ ◦ s0 ◦ f2µ ◦ s0 = f2(λ−µ) .
G = {fλ | λ ∈ R} ∪ {sλ | λ ∈ R} = {fλ | λ ∈ R} ∪ {fλ ◦ s0 | λ ∈ R} est stable par
composition et par passage à l’inverse, c’est donc un sous-groupe du groupe des
déplacements de E, et H = {fλ | λ ∈ R} en est un sous-groupe abélien d’indice 2.
Homographies
Aide
Précédente
Suivante
Plein écran
Quitter
Remarque
Accueil
Géométrie affine
Convexité
Si M est un point de ∆ différent de A, {fλ (M ) | λ ∈ R} est une hélice circulaire H
d’axe D et tout élément de G laisse cette hélice globalement invariante. On peut
montrer que G est exactement le groupe des isométries de E laissant H globalement invariante (en particulier il n’existe aucun antidéplacement de E laissant H
globalement invariante).
Géométrie euclidienne
Isométries planes
Isométries de l’espace
Coniques
Géométrie analytique
Complexes
Homographies
Aide
Précédente
Suivante
Plein écran
Quitter
Table
Tangentes menées d’un point à la parabole
Accueil
Géométrie affine
Convexité
Construire à la règle et au compas les tangentes à la parabole passant par un point
P du plan (on discutera selon la position du point).
Géométrie euclidienne
Isométries planes
Isométries de l’espace
Coniques
Géométrie analytique
Complexes
Homographies
Aide
Précédente
Suivante
Plein écran
Quitter
Indication
Solution
Accueil
Géométrie affine
Soit M un point de la parabole, H le projeté orthogonal de M sur la directrice D
et P un point de la tangente en M . Comparer P F et P H, où F est le foyer de
la parabole (on rappelle que la tangente en M à la parabole est la médiatrice de
HF ).
Convexité
Géométrie euclidienne
Isométries planes
Isométries de l’espace
Coniques
Géométrie analytique
Complexes
Homographies
Aide
Précédente
Suivante
Plein écran
Quitter
Retour
Solution
Accueil
Géométrie affine
Convexité
Géométrie euclidienne
Isométries planes
Soit M un point de la parabole, H le projeté orthogonal de M sur la directrice
D et P un point de la tangente en M . La réflexion d’axe M P échange F et H ;
on a donc P F = P H (une réflexion est une isométrie), ce qui montre que le point
H appartient au cercle de centre P passant par F . Ce cercle coupe la directrice
en 0, 1 ou 2 points selon la position de P par rapport à la parabole. Une fois H
déterminé, le point M s’obtient en prenant l’intersection de la médiatrice de HF
avec la perpendiculaire en H à la directrice.
Isométries de l’espace
Coniques
Géométrie analytique
Complexes
Homographies
Aide
Précédente
Suivante
Plein écran
Quitter
Suite
Déterminer l’ensemble des points du plan d’où l’on voit la parabole sous un angle
droit (i.e. tels que les deux tangentes menées par ces points à la parabole soient
perpendiculaires).
Accueil
Géométrie affine
Convexité
Géométrie euclidienne
Isométries planes
Isométries de l’espace
Coniques
Géométrie analytique
Complexes
Homographies
Aide
Précédente
Suivante
Plein écran
Quitter
Solution
Accueil
Géométrie affine
Convexité
Les deux tangentes menées par P à la parabole sont perpendiculaires si et seulement si les droites F H1 et F H2 , qui leur sont perpendiculaires, sont elles-mêmes
perpendiculaires. Le triangle F H1 H2 est alors rectangle en F et le centre P de son
cercle circonscrit est le milieu de H1 H2 : P appartient donc à la directrice de la
parabole. La réciproque s’établit de même.
Géométrie euclidienne
Isométries planes
Isométries de l’espace
Coniques
Géométrie analytique
Complexes
Homographies
Aide
Précédente
Suivante
Plein écran
Quitter
Table
Un problème de lieu géométrique
Accueil
Géométrie affine
Convexité
Géométrie euclidienne
Soit, dans le plan affine euclidien, C un cercle de centre F et de rayon R, et F 0 un
point intérieur à C. Déterminer l’ensemble des centres des cercles passant par F 0
et tangents à C (on distinguera les cas F 0 = F et F 0 6= F ).
Isométries planes
Isométries de l’espace
Coniques
Géométrie analytique
Complexes
Homographies
Aide
Précédente
Suivante
Plein écran
Quitter
Indication
Solution
Soit M le centre d’un tel cercle et T le point de contact de ce cercle et de C. Que
peut-on dire de M F + M F 0 ?
Accueil
Géométrie affine
Convexité
Géométrie euclidienne
Isométries planes
Isométries de l’espace
Coniques
Géométrie analytique
Complexes
Homographies
Aide
Précédente
Suivante
Plein écran
Quitter
Retour
Solution
La relation M F + M F 0 = M F + M T = F T = R montre que M appartient à
l’ellipse de foyers F et F 0 et de demi-grand axe R/2. Réciproquement, tout point
de cette ellipse est centre d’un tel cercle.
Accueil
Géométrie affine
Convexité
Géométrie euclidienne
Isométries planes
Isométries de l’espace
Coniques
Géométrie analytique
Complexes
Homographies
Aide
Précédente
Suivante
Plein écran
Quitter
Remarque
Accueil
Géométrie affine
On sait qu’une parabole est l’ensemble des centres des cercles passant par le foyer
et tangents à la directrice. On obtient ici une propriété analogue pour l’ellipse, la
directrice étant remplacée par un cercle de centre le second foyer, appelé cercle
directeur.
Convexité
Géométrie euclidienne
Isométries planes
Isométries de l’espace
Coniques
Géométrie analytique
Complexes
Homographies
Aide
Précédente
Suivante
Plein écran
Quitter
Cas particulier
Dans le cas particulier où F = F 0 , l’ellipse est remplacée par le cercle de centre F
et de rayon R/2.
Accueil
Géométrie affine
Convexité
Géométrie euclidienne
Isométries planes
Isométries de l’espace
Coniques
Géométrie analytique
Complexes
Homographies
Aide
Précédente
Suivante
Plein écran
Quitter
Table
Diamètres conjugués de l’ellipse
Accueil
Géométrie affine
Convexité
Déterminer l’ensemble des milieux des cordes d’une ellipse parallèles à une droite
D donnée.
Géométrie euclidienne
Isométries planes
Isométries de l’espace
Coniques
Géométrie analytique
Complexes
Homographies
Aide
Précédente
Suivante
Plein écran
Quitter
Indication
Solution
Examinez d’abord la question dans le cas où l’ellipse est un cercle.
Accueil
Géométrie affine
Convexité
Géométrie euclidienne
Isométries planes
Isométries de l’espace
Coniques
Géométrie analytique
Complexes
Homographies
Aide
Précédente
Suivante
Plein écran
Quitter
Retour
Solution
Accueil
Géométrie affine
Convexité
Géométrie euclidienne
Isométries planes
Isométries de l’espace
Soit E l’ellipse, C son cercle principal et f une affinité orthogonale transformant
→
−
E en C. Les cordes considérées, de direction D , ont pour images par f les cordes
→
−
de C de direction f~( D ), i.e. les cordes de C perpendiculaires à un diamètre A1 A2
de C. L’ensemble des milieux de ces cordes est donc le segment A1 A2 . Toute transformation affine conservant les milieux, l’ensemble des milieux des cordes de E de
→
−
direction D est l’image du segment A1 A2 par f −1 , i.e. un diamètre M1 M2 de E
(plus précisément le diamètre de E joignant les deux points de E où la tangente
est parallèle à D, puisqu’une transformation affine conserve le contact).
Coniques
Géométrie analytique
Complexes
Homographies
Aide
Précédente
Suivante
Plein écran
Suite
Quitter
Soit N1 N2 un diamètre de E (i.e. une corde passant par le centre de E), M1 M2
l’ensemble des milieux des cordes de E parallèles à N1 N2 . On dit que M1 M2 est le
diamètre conjugué de N1 N2 . Quel est le diamètre conjugué de M1 M2 ?
Accueil
Géométrie affine
Convexité
Géométrie euclidienne
Isométries planes
Isométries de l’espace
Coniques
Géométrie analytique
Complexes
Homographies
Aide
Précédente
Suivante
Plein écran
Quitter
Solution
Accueil
Géométrie affine
Le diamètre conjugué de M1 M2 est N1 N2 , de sorte que la relation de conjugaison est
symétrique (on dit que M1 M2 et N1 N2 sont des diamètres conjugués de l’ellipse).
Ici encore il suffit de vérifier la propriété pour le cercle, pour lequel elle est évidente,
et de transformer l’ellipse en cercle par une transformation affine.
Convexité
Géométrie euclidienne
Isométries planes
Isométries de l’espace
Coniques
Géométrie analytique
Complexes
Homographies
Aide
Précédente
Suivante
Plein écran
Quitter
Table
Ellipse de Steiner d’un triangle (1)
Accueil
Géométrie affine
Convexité
Montrer que, pour tout triangle non aplati ABC, il existe une ellipse tangente aux
côtés BC, CA et AB de ce triangle en leurs milieux A0 , B 0 , C 0 .
Géométrie euclidienne
Isométries planes
Isométries de l’espace
Coniques
Géométrie analytique
Complexes
Homographies
Aide
Précédente
Suivante
Plein écran
Quitter
Indication
Solution
Ramenez-vous au cas d’un triangle équilatéral au moyen d’une transformation affine.
Accueil
Géométrie affine
Convexité
Géométrie euclidienne
Isométries planes
Isométries de l’espace
Coniques
Géométrie analytique
Complexes
Homographies
Aide
Précédente
Suivante
Plein écran
Quitter
Retour
Solution
Accueil
Géométrie affine
Convexité
Géométrie euclidienne
Isométries planes
Isométries de l’espace
Soit A1 B1 C1 un triangle équilatéral. Il existe une transformation affine f du plan
et une seule qui transforme le triangle A1 B1 C1 en le triangle ABC. Toute transformation affine conservant les milieux et le contact, f transforme le cercle inscrit
dans le triangle A1 B1 C1 en une ellipse tangente aux côtés du triangle ABC en leurs
milieux. Cette ellipse (on peut montrer qu’elle est unique) est appelée ellipse de
Steiner du triangle ABC. Il résulte de la démonstration précédente qu’elle passe
aussi par les milieux des segments AG, BG et CG, où G est l’isobarycentre de
ABC.
Coniques
Géométrie analytique
Complexes
Homographies
Aide
Précédente
Suivante
Plein écran
Quitter
Application
Application
Accueil
Géométrie affine
Convexité
Montrer que pour tout triangle ABC, il existe une affinité orthogonale transformant
ce triangle en un triangle équilatéral.
Géométrie euclidienne
Isométries planes
Isométries de l’espace
Coniques
Géométrie analytique
Complexes
Homographies
Aide
Précédente
Suivante
Plein écran
Quitter
Indication
Solution
Soit g une affinité orthogonale transformant l’ellipse de Steiner du triangle ABC
en un cercle (pourquoi une telle affinité orthogonale existe-t-elle?). Montrer que g
transforme le triangle ABC en un triangle équilatéral.
Accueil
Géométrie affine
Convexité
Géométrie euclidienne
Isométries planes
Isométries de l’espace
Coniques
Géométrie analytique
Complexes
Homographies
Aide
Précédente
Suivante
Plein écran
Quitter
Retour
Solution
Accueil
Géométrie affine
Soit g l’affinité orthogonale transformant l’ellipse de Steiner du triangle ABC en
son cercle principal. L’image du triangle ABC par g est un triangle A1 B1 C1 dont
le cercle inscrit touche les côtés en leurs milieux A0 , B 0 , C 0 . Un tel triangle est
équilatéral, puisque A1 B 0 = A1 C 0 , B1 C 0 = B1 A0 , C1 A0 = C1 B 0 .
Convexité
Géométrie euclidienne
Isométries planes
Isométries de l’espace
Coniques
Géométrie analytique
Complexes
Homographies
Aide
Précédente
Suivante
Plein écran
Quitter
Table
Construction de l’hyperbole
Accueil
Géométrie affine
Convexité
Construire par points une hyperbole H connaissant ses asymptotes et un de ses
points.
Géométrie euclidienne
Isométries planes
Isométries de l’espace
Coniques
Géométrie analytique
Complexes
Homographies
Aide
Précédente
Suivante
Plein écran
Quitter
Indication
Solution
Comment obtient-on l’intersection de H avec une droite passant par un de ses
points?
Accueil
Géométrie affine
Convexité
Géométrie euclidienne
Isométries planes
Isométries de l’espace
Coniques
Géométrie analytique
Complexes
Homographies
Aide
Précédente
Suivante
Plein écran
Quitter
Retour
Solution
Accueil
Géométrie affine
Soit M le point donné, D une droite non parallèle aux asymptotes passant par
ce point et P , Q les points d’intersection de cette droite avec les asymptotes. Le
symétrique N de M par rapport au milieu I de P Q appartient à H. On obtient
ainsi autant de points de H que l’on veut.
Convexité
Géométrie euclidienne
Isométries planes
Isométries de l’espace
Coniques
Géométrie analytique
Complexes
Homographies
Aide
Précédente
Suivante
Plein écran
Quitter
Table
Projection et affinités
Accueil
Géométrie affine
Convexité
Géométrie euclidienne
Isométries planes
Isométries de l’espace
Soit, dans l’espace affine E de dimension 3 rapporté à un repère cartésien (0,~i,~j,~k),
→
−
P le plan d’équation 2x − 3y + 8z − 4 = 0 et D la droite vectorielle de vecteur
directeur ~u = 3~i − 2~j − ~k. Donner l’expression en coordonnées de la projection sur
→
−
P dans la direction D , puis, pour tout réel λ, de l’affinité de base P , de direction
−
→
D et de rapport λ.
Coniques
Géométrie analytique
Complexes
Homographies
Aide
Précédente
Suivante
Plein écran
Quitter
Solution
Accueil
Géométrie affine
Convexité
Géométrie euclidienne
Isométries planes
Isométries de l’espace
Coniques
Géométrie analytique
Complexes
Homographies
Aide
Précédente
Suivante
Plein écran
Quitter
Soit M0 un point de E de coordonnées (x0 ,y0 ,z0 ). La droite de vecteur directeur
~u passant par M0 admet la représentation paramétrique x = x0 + 3t, y = y0 − 2t,
z = z0 − t. Le point d’intersection de cette droite avec le plan P (i.e. le projeté de
M0 sur le plan P dans la direction D) est le point M00 de paramètre t0 vérifiant
1
2(x0 + 3t0 ) − 3(y0 − 2t0 ) + 8(z0 − t0 ) − 4 = 0, i.e. t0 = − (2x0 − 3y0 + 8z0 − 4). Les
4
coordonnées de ce point sont donc :
−2x0 + 9y0 − 24z0 + 12
x =
4
4x0 − 2y0 + 16z0 − 8
y =
4
2x0 − 3y0 + 12z0 − 4
z =
.
4
−−−−→
−−−→
L’image M000 de M0 par l’affinité de base P et de rapport λ vérifie M00 M000 = λM00 M0 ,
−−−−→ −−−→ −−−−→
−−−→
d’où M0 M000 = M0 M00 + M00 M000 = (1 − λ)M0 M00 = (1 − λ)t0~u. C’est donc le point
de paramètre (1 − λ)t0 , de coordonnées :
(6λ − 2)x0 − 9(λ − 1)y0 + 24(λ − 1)z0 − 12(λ − 1)
4
4(1 − λ)x0 + (6λ − 2)y0 + 16(1 − λ)z0 + 8(λ − 1)
y =
4
2(1 − λ)x0 + 3(λ − 1)y0 + (12 − 8λ)z0 + 4(λ − 1)
z =
.
4
x =
Table
Transformation affine du plan
Accueil
Géométrie affine
Convexité
Géométrie euclidienne
Isométries planes
Isométries de l’espace
Déterminer les points fixes, puis la nature géométrique de la transformation f du
plan définie par :
x0 = 3x − 4y + 8
y 0 = 2x − 3y + 8 .
Coniques
Géométrie analytique
Complexes
Homographies
Aide
Précédente
Suivante
Plein écran
Quitter
Solution
Accueil
Géométrie affine
Convexité
Géométrie euclidienne
L’ensemble des points fixes de f est„la droite
D d’équation x−2y+8 = 0. La matrice
Ž
3 −4
. Le polynôme caractéristique de A est
de la partie linéaire de f est A =
2 −3
X 2 − 1 ; A admet donc les deux valeurs propres +1 et −1. Le sous-espace propre
→
−
associé à la valeur propre −1 est la droite vectorielle D 0 d’équation x − y = 0. On
→
−
en déduit que f est la symétrie par rapport à la droite D dans la direction D 0 .
Isométries planes
Isométries de l’espace
Coniques
Géométrie analytique
Complexes
Homographies
Aide
Précédente
Suivante
Plein écran
Quitter
Table
Position relative de deux cercles
Accueil
Géométrie affine
Convexité
Géométrie euclidienne
Isométries planes
Soit, dans le plan affine euclidien rapporté à un repère orthonormé, C1 et C2 les deux
cercles d’équations respectives x2 +y 2 −8x−8y −32 = 0 et x2 +y 2 −6x−2y +6 = 0.
Déterminer le centre et le rayon de chacun de ces cercles, puis leur position l’un
par rapport à l’autre.
Isométries de l’espace
Coniques
Géométrie analytique
Complexes
Homographies
Aide
Précédente
Suivante
Plein écran
Quitter
Solution
Accueil
Géométrie affine
Convexité
Les équations de C1 et C2 s’écrivent (x−4)2 +(y −4)2 = 64 et (x−3)2 +(y −1)2 = 4.
Le centre O1 de C1 (resp. O2 de C2 ) a donc pour coordonnées (4,4) (resp.√(3,1))
et son rayon est R1 = 8 (resp. R2 = 2). La distance O1 O2 des centres est 10, et
l’inégalité O1 O2 + R2 < R1 montre que C2 est intérieur à C1 , puisque tout point
M de C2 vérifie O1 M ≤ O1 O2 + O2 M < R1 .
Géométrie euclidienne
Isométries planes
Isométries de l’espace
Coniques
Géométrie analytique
Complexes
Homographies
Aide
Précédente
Suivante
Plein écran
Quitter
Table
Perpendiculaire commune
Accueil
Géométrie affine
Convexité
Géométrie euclidienne
Isométries planes
Isométries de l’espace
Coniques
Géométrie analytique
Complexes
L’espace est rapporté à un repère orthonormé. Donner des équations de la perpendiculaire commune aux droites D1 d’équations
8
>
<
x+y−z−1=0
>
:2x + y + z = 0
et D2 déterminée par le point A2 de coordonnées (1,0,1) et le vecteur directeur ~v2
de composantes (1, − 1,0).
Homographies
Aide
Précédente
Suivante
Plein écran
Quitter
Solution
Accueil
Géométrie affine
Convexité
Géométrie euclidienne
Isométries planes
Isométries de l’espace
Coniques
Géométrie analytique
Complexes
Homographies
Aide
Précédente
La perpendiculaire commune ∆ aux deux droites admet comme vecteur directeur
~u le produit vectoriel ~v1 ∧ ~v2 d’un vecteur directeur ~v1 de D1 par ~v2 . Le vecteur
~v1 s’obtient en faisant le produit vectoriel de vecteurs normaux aux deux plans
définissant D1 , i.e. des vecteurs (1,1, − 1) et (2,1,1), d’où ~v1 = (2, − 3, − 1) et
~u = (−1, − 1,1).
La droite ∆ est alors l’intersection du plan P1 contenant D1 et parallèle à ~u et du
plan P2 contenant D2 et parallèle à ~u.
Tout plan contenant D1 a une équation de la forme λ(x + y − z − 1) + µ(2x +
y + z) = 0. Le vecteur ~u appartient à la direction de ce plan si et seulement si
−(λ − µ) − (λ + µ) + (−λ + µ) = 0, i.e. 3λ + 2µ = 0. On peut donc prendre λ = −2,
µ = 3, d’où une équation de P1 : 4x + y + 5z + 2 = 0.
1
−1
x
−
1
Une équation de P2 s’obtient en annulant le déterminant −1 −1
y et s’écrit
0
1 z − 1
donc x + y − 2z + 3 = 0.
La perpendiculaire commune aux deux droites D1 et D2 admet donc les équations :
8
Suivante
>
<
4x + y + 5z + 2 = 0
>
:
x + y − 2z + 3 = 0 .
Plein écran
Quitter
Suite
Distance de deux droites
Accueil
Géométrie affine
Déterminer la distance de ces deux droites.
Convexité
Géométrie euclidienne
Isométries planes
Isométries de l’espace
Coniques
Géométrie analytique
Complexes
Homographies
Aide
Précédente
Suivante
Plein écran
Quitter
Solution
Accueil
Géométrie affine
Convexité
Géométrie euclidienne
Isométries planes
Isométries de l’espace
Coniques
Géométrie analytique
Complexes
Homographies
La distance de ces deux droites s’obtient comme distance du point A2 au plan
passant par D1 et parallèle à D2 . L’équation de ce plan est de la forme λ(x + y −
z − 1) + µ(2x + y + z) = 0. Le vecteur ~v2 appartient à sa direction si et seulement
si (λ − µ) − (λ + µ) = 0, i.e. µ = 0, d’où l’équation x + y − z − 1 = 0. La distance
1
de A2 à ce plan est alors √ .
3
On remarque en particulier qu’on n’a pas besoin de déterminer la perpendiculaire
commune aux deux droites pour calculer leur distance. On peut cependant calculer
les coordonnées des pieds I1 et I2 de la perpendiculaire commune (i.e. des points
d’intersection de cette perpendiculaire avec les droites D1 et D2 ). On obtient les
7 2
1
8 11
points I2 (− , ,1) et I1 (− ,4, ), dont la distance est bien √ .
3 3
3 3
3
Aide
Précédente
Suivante
Plein écran
Quitter
Table
Equation normale d’une droite, bissectrices
Accueil
Géométrie affine
Convexité
Géométrie euclidienne
Le plan affine euclidien est rapporté à un repère orthonormé. Montrer que toute
droite D admet une équation normale de la forme x cos α + y sin α − p = 0. Interpréter les paramètres α et p.
Isométries planes
Isométries de l’espace
Coniques
Géométrie analytique
Complexes
Homographies
Aide
Précédente
Suivante
Plein écran
Quitter
Solution
Accueil
Géométrie affine
Convexité
Géométrie euclidienne
Isométries planes
Soit ~u(α) = (cos α, sin α) un vecteur unitaire normal à D, H le projeté orthogonal
−−→
de l’origine O sur D, et p le réel défini par OH = p~u(α). Un point M appartient à D
−−→
si et seulement si OM ·~u(α) = p, i.e. si et seulement si f (x,y) = x cos α+y sin α−p =
0. On peut remarquer qu’une droite admet deux équations normales correspondant
aux choix des vecteurs normaux ~u(α) et ~u(α + π) (ces équations sont f (x,y) = 0
et −f (x,y) = 0).
Isométries de l’espace
Coniques
Géométrie analytique
Complexes
Homographies
Aide
Précédente
Suivante
Plein écran
Quitter
Suite
Accueil
Géométrie affine
Soit ABC un triangle, fA (x,y) = 0, fB (x,y) = 0, fC (x,y) = 0 les équations des
droites BC, CA et AB normalisées de sorte que fA (xA ,yA ) > 0, fB (xB ,yB ) > 0,
fC (xC ,yC ) > 0. Ecrire les équations des bissectrices du triangle ABC en fonction
de fA , fB , fC .
Convexité
Géométrie euclidienne
Isométries planes
Isométries de l’espace
Coniques
Géométrie analytique
Complexes
Homographies
Aide
Précédente
Suivante
Plein écran
Quitter
Solution
Accueil
Géométrie affine
Convexité
Géométrie euclidienne
Isométries planes
Un point appartient à l’une des deux bissectrices en A si et seulement si il est
équidistant des droites AB et AC, i.e. si et seulement si ses coordonnées (x,y)
vérifient |fB (x,y)| = |fC (x,y)|. Les équations de ces deux bissectrices sont donc
fB (x,y) − fC (x,y) = 0 et fB (x,y) + fC (x,y) = 0. Le centre I du cercle inscrit est du
même côté de AB que C : il vérifie donc fC (xI ,yI ) > 0. De même fB (xI ,yI ) > 0.
L’équation de la bissectrice intérieure en A est donc fB (x,y) − fC (x,y) = 0 et celle
de la bissectrice extérieure fB (x,y) + fC (x,y) = 0.
Isométries de l’espace
Coniques
Géométrie analytique
Complexes
Homographies
Aide
Précédente
Suivante
Plein écran
Quitter
Suite
Accueil
Géométrie affine
On suppose le triangle ABC non isocèle en A. Montrer que la droite d’équation
fA (x,y) − fB (x,y) − fC (x,y) = 0 passe par les pieds des bissectrices intérieures en
B et C, ainsi que par le pied de la bissectrice extérieure en A. Montrer de même
que les pieds des trois bissectrices extérieures du triangle sont alignés.
Convexité
Géométrie euclidienne
Isométries planes
Isométries de l’espace
Coniques
Géométrie analytique
Complexes
Homographies
Aide
Précédente
Suivante
Plein écran
Quitter
Solution
Accueil
Géométrie affine
Convexité
Géométrie euclidienne
Les coordonnées (x,y) du pied de la bissectrice intérieure en B vérifient fA (x,y) −
fC (x,y) = 0 et fB (x,y) = 0, donc fA (x,y) − fC (x,y) − fB (x,y) = 0. Les pieds des
bissectrices intérieure en C et extérieure en A vérifient également cette équation.
Ces trois points sont donc alignés.
De même, les pieds des trois bissectrices extérieures vérifient tous l’équation fA (x,y)+
fB (x,y) + fC (x,y) = 0. Ils sont donc alignés.
Isométries planes
Isométries de l’espace
Coniques
Géométrie analytique
Complexes
Homographies
Aide
Précédente
Suivante
Plein écran
Quitter
Table
Réflexion
Accueil
Géométrie affine
Convexité
L’espace est rapporté à un repère orthonormé. Donner l’expression en coordonnées
de la réflexion par rapport au plan P d’équation x − y + z + 5 = 0.
Géométrie euclidienne
Isométries planes
Isométries de l’espace
Coniques
Géométrie analytique
Complexes
Homographies
Aide
Précédente
Suivante
Plein écran
Quitter
Solution
Accueil
Géométrie affine
Convexité
Géométrie euclidienne
Isométries planes
Isométries de l’espace
Coniques
Géométrie analytique
Complexes
Homographies
Le vecteur de composantes (1, − 1,1) est orthogonal à P .
La droite orthogonale à P passant par le point M de coordonnées (x,y,z) admet
donc la représentation paramétrique X = x + λ, Y = y − λ, Z = z + λ.
x−y+z+5
Son intersection avec P est le point de paramètre λ = −
.
3
Le point symétrique de M par rapport à P est le point de paramètre 2λ. Il a donc
pour coordonnées :
x + 2y − 2z − 10
3
2x
+
y
+
2z + 10
y0 =
3
−2x
+
2y
+ z − 10
z0 =
.
3
x0 =
Aide
Précédente
Suivante
Plein écran
Quitter
Table
Isométrie de l’espace
Accueil
Géométrie affine
Convexité
Géométrie euclidienne
Isométries planes
Isométries de l’espace
Coniques
Géométrie analytique
Complexes
L’espace affine euclidien E de dimension 3 est rapporté à un repère orthonormé.
Etudier la nature géométrique de la transformation de E qui à un point M de
coordonnées (x,y,z) associe le point M 0 de coordonnées (x0 ,y 0 ,z 0 ) définies par :
x0 = z
y0 = y
z0 = x − 2
(on précisera les éléments caractéristiques de cette transformation).
Homographies
Aide
Précédente
Suivante
Plein écran
Quitter
Solution
‡
Accueil
Géométrie affine
Convexité
Géométrie euclidienne
Isométries planes
Isométries de l’espace
Coniques
Géométrie analytique
Complexes
Homographies
‘
0 0 1
La transformation f étudiée est affine et la matrice 0 1 0 de sa partie linéaire
1 0 0
f~ est orthogonale de déterminant −1 ; f est donc un antidéplacement, qui n’admet
pas de point fixe puisque le système x = z, z = x − 2 n’admet pas de solution.
C’est donc une symétrie glissée, i.e. le produit commutatif t~u ◦ sP = sP ◦ t~u d’une
→
−
translation de vecteur ~u et d’une réflexion de plan P , avec ~u ∈ P . La transformation f ◦ f = t2~u est la translation de vecteur (−2,0, − 2) ; ~u est donc le vecteur
(−1,0, − 1). La réflexion sP = t−~u ◦ f est donnée par les formules x0 = z + 1, y 0 = y,
z 0 = x − 1. C’est donc la symétrie orthogonale par rapport au plan P d’équation
x − z − 1 = 0.
Aide
Précédente
Suivante
Plein écran
Quitter
Table
Isométrie de l’espace
Accueil
Géométrie affine
Convexité
Géométrie euclidienne
Isométries planes
Isométries de l’espace
Coniques
Géométrie analytique
Complexes
Homographies
Aide
L’espace affine euclidien E de dimension 3 est rapporté à un repère orthonormé.
Etudier la nature géométrique de la transformation f de E qui à un point M de
coordonnées (x,y,z) associe le point M 0 de coordonnées (x0 ,y 0 ,z 0 ) définies par :
√
√
x
+
y
−
2
z
−
3
+
2
x0 =
√2
√
x
+
y
+
2
z
−
1
−
2
y0 =
√
√ 2
√
2
x
−
2
y
+
2
+
2
z0 =
2
(on précisera les éléments caractéristiques de cette transformation).
Précédente
Suivante
Plein écran
Quitter
Solution
√ ‘
1
1 −√ 2
1
La matrice A =
de la partie linéaire f~ de f est orthogonale
1
1
2
√
2 √
2 − 2
0
de déterminant 1 ; f est donc une rotation ou un vissage.
La trace de A est égale à 1 ; il en résulte que l’angle de f~ est droit.
→
−
L’axe D de f a pour direction D le sous-espace propre de f~ associée à la valeur
propre -1, qui est la droite vectorielle engendrée par le vecteur ~u = (1,1,0).
−−−−−→
L’axe est l’ensemble des points M tels que le vecteur M f (M ) soit proportionnel à
~u. C’est donc la droite dirigée par ~u et passant par le point (0,1,1).
−−−−−→
Le vecteur de ce vissage est le vecteur M f (M ) pour tout point M de D. C’est
donc le vecteur −~u.
‡
Accueil
Géométrie affine
Convexité
Géométrie euclidienne
Isométries planes
Isométries de l’espace
Coniques
Géométrie analytique
Complexes
Homographies
Aide
Précédente
Suivante
Plein écran
Quitter
Table
Isométries du cube et du tétraèdre
Accueil
Géométrie affine
Convexité
Géométrie euclidienne
Isométries planes
Isométries de l’espace
Soit, dans l’espace affine euclidien E de dimension 3 rapporté à un repère orthonormé (O,~i,~j,~k), C le cube de sommets les points de coordonnées (±1, ± 1, ± 1)
et G le groupe des isométries de E laissant C globalement invariant. Montrer que
tout élément f de G laisse O fixe et conserve globalement l’ensemble des points de
coordonnées (±1,0,0), (0, ± 1,0), (0,0, ± 1). Décrire les matrices dans la base (~i,~j,~k)
des parties linéaires de tous les éléments de G. En déduire le cardinal de G.
Coniques
Géométrie analytique
Complexes
Homographies
Aide
Précédente
Suivante
Plein écran
Solution
Quitter
Accueil
Géométrie affine
Convexité
Géométrie euclidienne
Isométries planes
Isométries de l’espace
Coniques
Géométrie analytique
Tout élément de G est une transformation affine qui laisse globalement invariant
l’ensemble des sommets de C, donc laisse fixe l’isobarycentre O de ces sommets.
Toute face du cube est transformée en une face : il en résulte que l’ensemble des
milieux des faces est globalement conservé. La partie linéaire f~ d’un élément f de
G conserve donc globalement l’ensemble des 6 vecteurs {±~i, ± ~j, ± ~k}. Sa matrice
dans la base (~i,~j,~k) comporte donc dans chaque ligne et dans chaque colonne un
élément non nul et un seul, égal à ±1. Réciproquement toute transformation affine
f de E conservant O et dont la partie linéaire f~ a une telle matrice conserve le
cube. On trouve ainsi 48 éléments dans G (6 permutations des vecteurs de base et
pour chacune de ces permutations 8 choix de signes).
Complexes
Homographies
Aide
Précédente
Suivante
Plein écran
Quitter
Suite
Soit T le tétraèdre régulier de sommets (1,1,1), (1,−1,−1), (−1,1,−1), (−1,−1,1).
Montrer que le groupe H des isométries de E conservant globalement T est un
sous-groupe de G. Déterminer le nombre d’éléments de ce groupe.
Accueil
Géométrie affine
Convexité
Géométrie euclidienne
Isométries planes
Isométries de l’espace
Coniques
Géométrie analytique
Complexes
Homographies
Aide
Précédente
Suivante
Plein écran
Quitter
Solution
Accueil
Géométrie affine
Convexité
Géométrie euclidienne
Isométries planes
Isométries de l’espace
Toute isométrie conservant globalement T laisse fixe l’isobarycentre O de ses sommets et conserve globalement l’ensemble des milieux de ses arêtes, i.e. l’ensemble
des points de coordonnées (±1,0,0), (0, ± 1,0), (0,0, ± 1). On en déduit que H est
un sous-groupe de G. L’image du tétraèdre T par un élément de G est soit T , soit
le tétraèdre T 0 de sommets (−1, − 1, − 1), (−1,1,1), (1, − 1,1), (1,1, − 1) symétrique
de T par rapport à O. On en déduit un homomorphisme de groupes de G dans le
groupe des permutations de {T,T 0 } dont H est le noyau. Le sous-groupe H de G
est donc d’indice 2 et a 24 éléments.
Coniques
Géométrie analytique
Complexes
Homographies
Aide
Précédente
Suivante
Plein écran
Quitter
Table
Equation d’une conique
Accueil
Géométrie affine
Convexité
Géométrie euclidienne
Isométries planes
Le plan est rapporté à un repère orthonormé. Ecrire l’équation de l’hyperbole
H
√ de foyer F (3,2), de directrice D d’équation x − y + 1 = 0 et d’excentricité
2. Déterminer le centre de symétrie de H, puis le second couple foyer-directrice
(F 0 ,D0 ).
Isométries de l’espace
Coniques
Géométrie analytique
Complexes
Homographies
Aide
Précédente
Suivante
Plein écran
Quitter
Solution
Accueil
Géométrie affine
Convexité
Géométrie euclidienne
Isométries planes
Le point M de coordonnées (x,y) appartient à H si et seulement si M F 2 =
2d(M,D)2 , i.e. si et seulement si (x − 3)2 + (y − 2)2 = (x − y + 1)2 , ou encore
xy − 4x − y + 6 = 0. Cette équation s’écrit encore (x − 1)(y − 4) = −2, soit, en
posant X = x−1, Y = y −4, XY = −2, ce qui montre que le centre Ω de H a pour
coordonnées (1,4). Le foyer F 0 est le symétrique de F par rapport à Ω et a donc
pour coordonnées (−1,6) et la directrice associée D0 pour équation y − x − 5 = 0
(la symétrie par rapport à Ω s’écrit en effet en coordonnées x0 = 2 − x, y 0 = 8 − y).
Isométries de l’espace
Coniques
Géométrie analytique
Complexes
Homographies
Aide
Précédente
Suivante
Plein écran
Quitter
Table
Construction de l’ellipse par le procédé de la bande
de papier
Accueil
Géométrie affine
Convexité
Géométrie euclidienne
Isométries planes
Sur un segment AB de longueur fixée a + b dont les extrémités A et B se déplacent
sur deux axes orthogonaux Ox et Oy, on place un point M tel que AM = b (et
donc BM = a). Déterminer la courbe décrite par le point M quand A et B varient.
Isométries de l’espace
Coniques
Géométrie analytique
Complexes
Homographies
Aide
Précédente
Suivante
Plein écran
Quitter
Solution
Accueil
Géométrie affine
−→ −−→
Soit t une mesure de l’angle (BA,BO) et (x,y) les coordonnées de M dans le repère
Oxy. Les égalités x = a cos t, y = b sin t montrent que M décrit un quart d’ellipse
quand A décrit la demi-droite Ox et B la demi-droite Oy.
Convexité
Géométrie euclidienne
Isométries planes
Isométries de l’espace
Coniques
Géométrie analytique
Complexes
Homographies
Aide
Précédente
Suivante
Plein écran
Quitter
Table
Paramétrisation du cercle
Accueil
Géométrie affine
Convexité
1 + it
est une bijection de R sur le cercle unité
1 − it
U privé du point −1. Expliciter l’application réciproque de ϕ.
Montrer que l’application ϕ : t 7→
Géométrie euclidienne
Isométries planes
Isométries de l’espace
Coniques
Géométrie analytique
Complexes
Homographies
Aide
Précédente
Suivante
Plein écran
Quitter
Indication
Solution
1 + it
1 + it = 1. En écrivant
= eiα et en posant t = tan θ,
Remarquez que
1 − it
1 − it
exprimez α en fonction de θ.
Accueil
Géométrie affine
Convexité
Géométrie euclidienne
Isométries planes
Isométries de l’espace
Coniques
Géométrie analytique
Complexes
Homographies
Aide
Précédente
Suivante
Plein écran
Quitter
Retour
Solution
Accueil
Géométrie affine
Convexité
Géométrie euclidienne
Isométries planes
Isométries de l’espace
Coniques
Géométrie analytique
Complexes
1 + it
1 + it |1 + it|
=
=
1
montre
qu’il
existe
α
∈]
−
π,
+
π]
tel
que
=
L’égalité
1 − it |1 − it|
1 − it –
™
π
π
eiα . En posant t = tan θ (pour tout réel t, il existe un unique θ ∈ − , +
2
2
1 + it
1 − t2
2it
vérifiant cette égalité), on obtient
=
+
= cos α + i sin α, d’où
1 − it
1 + t2 1 + t2
cos α = cos 2θ et sin α = sin 2θ, i.e. α = 2θ. Cette relation montre que ϕ est une
bijection de R sur U \ {−1}, la bijection réciproque étant donnée par z = eiα 7→
α
sin α
Im (z)
1 − cos α
1 − Re (z)
tan , qui peut aussi s’écrire
=
ou
=
si
2
1 + cos α
1 + Re (z)
sin α
Im (z)
z 6= 0. L’interprétation géométrique de la relation α = 2θ résulte du théorème de
l’angle inscrit.
Homographies
Aide
Précédente
Suivante
Plein écran
Quitter
Table
Orthocentre
Accueil
Géométrie affine
Convexité
Géométrie euclidienne
Soient b et c deux nombres complexes. Donner une condition nécessaire et suffisante
faisant intervenir les modules de b et c pour que les vecteurs d’affixes b + c et c − b
soient orthogonaux. Interpréter géométriquement cette condition.
Isométries planes
Isométries de l’espace
Coniques
Géométrie analytique
Complexes
Homographies
Aide
Précédente
Suivante
Plein écran
Quitter
Solution
Suite
Le produit scalaire de ces deux vecteurs s’écrit :
Accueil
Géométrie affine
Convexité
(b + c)(c − b) + (b + c)(c − b)
= |c|2 − |b|2 .
2
Il est donc nul si et seulement si |b| = |c|.
Géométrie euclidienne
Isométries planes
Isométries de l’espace
Coniques
Géométrie analytique
Complexes
Soient B, C et D les points du plan complexe d’affixes b, c et b + c. Le quadrilatère OBDC est un parallélogramme et les nombres complexes b + c et c − b
−−→
−−→
sont les affixes des vecteurs OD et BC. La condition |b| = |c| signifie que ce parallélogramme est un losange : on retrouve ainsi le fait qu’un parallélogramme est
un losange si et seulement si ses diagonales sont perpendiculaires.
Homographies
Aide
Précédente
Suivante
Plein écran
Quitter
Suite
Accueil
Géométrie affine
Convexité
Géométrie euclidienne
Soit ABC un triangle non aplati du plan affine euclidien, O le centre de son cercle
circonscrit. On rapporte le plan à un repère orthonormé d’origine O et on note a,
b, c les affixes des points A, B, C. Montrer que le point H d’affixe h = a + b + c
appartient aux trois hauteurs du triangle ABC. En déduire que ces trois hauteurs
−−→
−→
sont concourantes et que leur point d’intersection H vérifie OH = 3OG, où G est
le centre de gravité du triangle (en particulier O, G et H sont alignés).
Isométries planes
Isométries de l’espace
Coniques
Géométrie analytique
Complexes
Homographies
Aide
Précédente
Suivante
Plein écran
Quitter
Retour
Solution
Accueil
Géométrie affine
Convexité
Géométrie euclidienne
O étant équidistant des trois points A, B et C, on a |a| = |b| = |c|. Les vecteurs
−−→
−−→
AH et BC, d’affixes b + c et c − b sont donc orthogonaux, ce qui montre que H
appartient à la hauteur issue de A. Il appartient de même aux deux autres hauteurs.
C’est donc l’orthocentre du triangle ABC. Le centre de gravité du triangle G est
−−→
−→
a+b+c
le point d’affixe g =
. On a donc OH = 3OG.
3
Isométries planes
Isométries de l’espace
Coniques
Géométrie analytique
Complexes
Homographies
Aide
Précédente
Suivante
Plein écran
Quitter
Retour
Table
Deux carrés (ou trois. . . )
Accueil
Géométrie affine
Convexité
Géométrie euclidienne
Soient, dans le plan affine euclidien, ABCD et AEF G deux carrés de même orientation ayant un sommet commun et P , Q, R, S les milieux respectifs des segments
BD, DE, EG et GB. Montrer que P QRS est un carré.
Isométries planes
Isométries de l’espace
Coniques
Géométrie analytique
Complexes
Homographies
Aide
Précédente
Suivante
Plein écran
Quitter
Indication
Solution
Indication :
Accueil
Géométrie affine
Convexité
Considérer un repère orthonormé d’origine A, dans lequel B a pour affixe b, E pour
−→ −→
affixe e, et écrire les affixes des vecteurs P R et QS (par exemple).
Géométrie euclidienne
Isométries planes
Isométries de l’espace
Coniques
Géométrie analytique
Complexes
Homographies
Aide
Précédente
Suivante
Plein écran
Quitter
Solution
Accueil
Géométrie affine
Convexité
Géométrie euclidienne
Isométries planes
Isométries de l’espace
Coniques
Géométrie analytique
Complexes
Homographies
Aide
Précédente
Suivante
Plein écran
Quitter
Soit b l’affixe de B et e l’affixe de E dans
un repère orthonormé d’origine A. Les
points P , Q, R, S ont pour affixes respectives :
(1 + i)b
p =
2
e + ib
q =
2
(1 + i)e
r =
2
b + ie
s =
2
−→ −→
et les vecteurs P R et QS pour affixes :
(1 + i)(e − b)
2
(1 − i)(b − e)
s−q =
.
2
Les segments P R et QS sont donc orthogonaux, de même longueur et se coupent
s+q
r+p
en leur milieu, puisque s − q = i(r − p) et
=
. Il en résulte que P QRS
2
2
est un carré.
r−p =
Autre solution
Retour
Table
Deux carrés (ou trois. . . )
Accueil
Géométrie affine
Convexité
Géométrie euclidienne
Soit, dans le plan complexe, ABC un triangle, f une rotation de centre Ω et d’angle
π
, et A0 B 0 C 0 l’image du triangle ABC par f . On note P , Q, R les milieux respectifs
3
des segments AC 0 , BA0 et CB 0 . Montrer que le triangle P QR est équilatéral.
Isométries planes
Isométries de l’espace
Coniques
Géométrie analytique
Complexes
Homographies
Aide
Précédente
Suivante
Plein écran
Indication
Quitter
Solution
Ecrire l’affixe z 0 de l’image M 0 = f (M ) d’un point M d’affixe z par la rotation f
(on pourra prendre un repère orthonormé d’origine Ω), puis les affixes des vecteurs
−→ −→
P Q et P R en fonction des affixes de A, B, C.
Accueil
Géométrie affine
Convexité
Géométrie euclidienne
Isométries planes
Isométries de l’espace
Coniques
Géométrie analytique
Complexes
Homographies
Aide
Précédente
Suivante
Plein écran
Quitter
Solution
2iπ
Accueil
Géométrie affine
Convexité
Géométrie euclidienne
iπ
Posons j = e 3 , d’où e 3 = −j 2 , et 1 + j + j 2 = 0. L’affixe z 0 de l’image M 0 = f (M )
d’un point M d’affixe z par f est z 0 = −j 2 z. Les points A0 , B 0 , C 0 ont donc pour
affixes a0 = −j 2 a, b0 = −j 2 b, c0 = −j 2 c si les affixes de A, B, C sont a, b, c.
a − j 2c
b − j 2a
c − j 2b
Les points P , Q, R ont pour affixes p =
,q=
,r=
.
2
2
2
−→
L’affixe du vecteur P Q est donc :
Isométries planes
b + ja + j 2 c
q−p=
2
Isométries de l’espace
Coniques
Géométrie analytique
Complexes
Homographies
Aide
Précédente
−→
et celle du vecteur P R :
−j 2 b − a − jc
= −j 2 (q − p) .
2
−→
−→
Le vecteur P R est donc l’image du vecteur P Q par la rotation vectorielle d’angle
π
, ce qui montre que le triangle P QR est équilatéral.
3
r−p=
Suivante
Plein écran
Quitter
Retour
Table
Configuration de Vecten
Accueil
Géométrie affine
Convexité
Géométrie euclidienne
Isométries planes
À l’extérieur d’un triangle ABC, on construit trois carrés de bases les côtés et de
centres P , Q, R. Montrer que les segments AP et QR (resp. BQ et RP , CR et
P Q) sont orthogonaux et de même longueur. En déduire que les droites AP , BQ
et CR sont concourantes.
Isométries de l’espace
Coniques
Géométrie analytique
Complexes
Homographies
Aide
Précédente
Suivante
Plein écran
Quitter
Solution
Soient a, b, c les affixes des points A, B, C dans un repère orthonormé. Des relations b−p = i(c−p),
c − q = i(a − q), a − r = i(b − r), on déduit que les points P , Q, R ont pour affixes respectives :
Accueil
p=
Géométrie affine
Convexité
−→ −→
et les vecteurs AP et QR pour affixes :
Géométrie euclidienne
Isométries planes
Isométries de l’espace
Coniques
Géométrie analytique
b − ic
c − ia
a − ib
, q=
, r=
1−i
1−i
1−i
p−a=
a − ib − c + ia
b − ic − (1 − i)a
, r−q =
.
1−i
1−i
La relation r − q = −i(p − a) montre que les segments AP et QR sont orthogonaux et de
même longueur. Il en va de même pour BQ et RP (resp. CR et P Q) par permutation circulaire. Les droites AP , BQ et CR sont les hauteurs du triangle P QR : elles sont donc concourantes.
Complexes
Homographies
Aide
Précédente
Suivante
Plein écran
Quitter
Table
Triangles de Napoléon
Accueil
Géométrie affine
Convexité
Géométrie euclidienne
On construit à l’extérieur d’un triangle ABC trois triangles équilatéraux BCA0 ,
CAB 0 et ABC 0 , de centres respectifs P , Q, R. Montrer que le triangle P QR est
équilatéral.
Isométries planes
Isométries de l’espace
Coniques
Géométrie analytique
Complexes
Homographies
Aide
Précédente
Suivante
Plein écran
Quitter
Solution
Accueil
Géométrie affine
Convexité
Géométrie euclidienne
Isométries planes
Isométries de l’espace
Soient a, b, c, p, q, r les affixes des points A, B, C, P , Q, R et j = e2iπ/3 . Les angles
−→ −−→ −→ −→ −→ −→
(P C,P B), (QA,QC), (RB,RA) étant tous égaux à 2π/3, on a b − p = j(c − p),
b − jc
c − ja
a − jb
c − q = j(a − q), a − r = j(b − r). On en déduit p =
,q=
,r=
.
1−j
1−j
1−j
−→
−→
En utilisant la relation 1 + j + j 2 = 0, on voit que les vecteurs P Q et P R ont
−ja − b − j 2 c
a + j 2 b + jc
pour affixes q − p =
et r − p =
. La relation r − p =
1−j
1−j
−j 2 (q − p) = eiπ/3 (q − p) montre que le triangle P QR est équilatéral.
Coniques
Géométrie analytique
Complexes
Homographies
Aide
Précédente
Suivante
Plein écran
Table
Quitter
Le théorème de Ptolémée
Accueil
Géométrie affine
Convexité
Soient A, B, C, D quatre points du plan affine euclidien. Démontrer l’inégalité :
(∗)
AC.BD ≤ AB.CD + AD.BC .
Géométrie euclidienne
Isométries planes
Isométries de l’espace
Coniques
Géométrie analytique
Complexes
Montrer qu’on a égalité dans (∗) si et seulement si les quatre points A, B, C, D
sont :
• soit alignés dans cet ordre
• soit les sommets consécutifs d’un quadrilatère convexe et cyclique.
Homographies
Aide
Précédente
Suivante
Plein écran
Quitter
Indication
Solution
Soient a, b, c, d les affixes respectives des points A, B, C, D. Comparer (c−a)(d−b)
et (b − a)(d − c) + (d − a)(c − b). À quelle condition a-t-on égalité dans l’inégalité
triangulaire?
Accueil
Géométrie affine
Convexité
Géométrie euclidienne
Isométries planes
Isométries de l’espace
Coniques
Géométrie analytique
Complexes
Homographies
Aide
Précédente
Suivante
Plein écran
Quitter
Solution
De l’égalité (c − a)(d − b) = (b − a)(d − c) + (d − a)(c − b), on tire :
Accueil
Géométrie affine
AC.BD = |(c − a)(d − b)| = |(b − a)(d − c) + (d − a)(c − b)|
≤ |(b − a)(d − c)| + |(d − a)(c − b)| = AB.CD + AD.BC
Convexité
Géométrie euclidienne
Isométries planes
Isométries de l’espace
Coniques
Géométrie analytique
Complexes
Homographies
Aide
Précédente
Suivante
Plein écran
Quitter
(le produit des longueurs des diagonales d’un quadrilatère est inférieur à la somme
des produits des longueurs des côtés opposés).
On a égalité dans l’inégalité triangulaire
|(b − a)(d − c) + (d − a)(c − b)| ≤ |(b − a)(d − c)| + |(d − a)(c − b)|
si et seulement si
(b − a)(d − c)
est un réel positif, i.e. si et seulement si
(d − a)(c − b)
(b − a)(d − c)
= 0 (mod 2π) .
(d − a)(c − b)
−−→ −→
−−→ −−→
Mais cet argument est l’angle (AD,AB)−(CD,BC). La condition peut donc encore
s’écrire :
−−→ −→
−−→ −−→
(AD,AB) = (CD,CB) + π
Arg
et signifie que les points A, B, C, D sont soit alignés dans cet ordre, soit les
sommets consécutifs d’un quadrilatère convexe et cyclique.
Table
Projection orthogonale d’un cube
Accueil
Géométrie affine
Convexité
Géométrie euclidienne
Isométries planes
Isométries de l’espace
Coniques
L’espace affine euclidien E de dimension 3 est rapporté à un repère orthonormé
(O,~i,~j,~k). On identifie le plan P d’équation z = 0 au plan complexe. Montrer
qu’une condition nécessaire et suffisante pour que trois points A0 , B 0 , C 0 du plan
P , d’affixes a, b, c, soient les projections orthogonales sur P des sommets A, B, C
d’un cube de sommet O adjacents à O est :
a2 + b 2 + c 2 = 0 .
Géométrie analytique
Complexes
Homographies
Aide
Précédente
Suivante
Plein écran
Quitter
Indication
Solution
‡
Accueil
Géométrie affine
Convexité
Géométrie euclidienne
Isométries planes
‘
xA xB xC
Soit yA yB yC la matrice des coordonnées des points A, B, C dans le repère
zA zB zC
orthonormé (O,~i,~j,~k). Montrez que les vecteurs colonnes de cette matrice sont deux
à deux orthogonaux et de même norme si et seulement si les vecteurs lignes de cette
même matrice vérifient la même propriété.
Isométries de l’espace
Coniques
Géométrie analytique
Complexes
Homographies
Aide
Précédente
Suivante
Plein écran
Quitter
Retour
Solution
Les points A, B, C sont les
−→
OA
seulement si les vecteurs
,
OA
Accueil
Géométrie affine
Convexité
Géométrie euclidienne
Isométries planes
Isométries de l’espace
Coniques
Géométrie analytique
Complexes
Homographies
Aide
Précédente
Suivante
Plein écran
Quitter
sommets adjacents au sommet O d’un cube si et
−−→ −→
→
−
OB OC
,
forment une base orthonormée de E , i.e.
OB OC
‘
‡
xA xB xC
des
si et seulement si les vecteurs colonnes de la matrice M = yA yB yC
zA zB zC
coordonnées des points A, B, C dans le repère orthonormé (O,~i,~j,~k) sont deux
à deux orthogonaux et de même norme r, i.e. si et seulement si il existe un réel
1
r tel que la matrice M soit orthogonale. Une matrice étant orthogonale si et
r
seulement si sa transposée l’est, cette condition signifie que les vecteurs lignes de
M sont orthogonaux et de même norme. Mais la relation
a2 + b2 + c2 = (xA + iyA )2 + (xB + iyB )2 + (xC + iyC )2
= (x2A + x2B + x2C ) − (yA2 + yB2 + yC2 ) + 2i(xA yA + xB yB + xC yC )
montre que cette condition implique a2 + b2 + c2 = 0.
Réciproquement, si cette dernière condition est vérifiée, les deux premières lignes
de M sont orthogonales et de même norme r. Il existe alors un vecteur (zA ,zB ,zC )
1
(et un seul au signe près) tel que la matrice M soit orthogonale.
r
Retour
Suite
Exemple numérique :
Accueil
Géométrie affine
Convexité
Géométrie euclidienne
Soit a = 3+2i, b = 6−3i. Montrer qu’il existe exactement deux nombres complexes
c, que l’on déterminera, vérifiant a2 + b2 + c2 = 0. Représenter les projections
orthogonales sur P des cubes correspondants.
Isométries planes
Isométries de l’espace
Coniques
Géométrie analytique
Complexes
Homographies
Aide
Précédente
Suivante
Plein écran
Quitter
Solution
Suite
Accueil
Géométrie affine
Convexité
Si a = 3 + 2i et b = 6 − 3i, on a a2 + b2 = 32 − 24i, d’où c2 = −32 + 24i et c = 2 + 6i
ou c = −2 − 6i. On peut donc construire les projections des sommets O, A, B,
C du cube. On complète ensuite les projections des faces en remarquant que ces
projections sont des parallélogrammes. On obtient ainsi quatre cubes de longueur
d’arête 7, correspondant aux quatre matrices M :
Géométrie euclidienne
‡
Isométries planes
Isométries de l’espace
Coniques
‘
3 6 2
2 −3 6
6 −2 −3
‡
‘
3 6 2
2 −3 6
−6 2 3
‡
‘
3 6 −2
2 −3 −6
−6 2 −3
Géométrie analytique
‡
‘
3 6 −2
2 −3 −6
6 −2 3
a
Complexes
c
a
o
c
o
Homographies
b
a
b
a
o
o
c
Aide
b
c
b
Précédente
Suivante
Plein écran
Quitter
Retour
Suite
Projection orthogonale d’un tétraèdre régulier
Accueil
Géométrie affine
Convexité
Géométrie euclidienne
Isométries planes
En déduire que trois nombres complexes a, b, c sont les affixes des projections
orthogonales sur P des sommets A, B, C d’un tétraèdre régulier OABC si et
seulement si ils vérifient la relation :
3(a2 + b2 + c2 ) − 2(ab + bc + ca) = 0 .
Isométries de l’espace
Coniques
Géométrie analytique
Complexes
Homographies
Aide
Précédente
Suivante
Plein écran
Quitter
Indication
Solution
OABC est un tétraèdre régulier si et seulement si il existe trois points R, S, T
−→ −→ −→
de l’espace, sommets adjacents au sommet O d’un cube tels que OA = OS + OT ,
−−→ −→ −→ −→ −→ −→
OB = OT + OR, OC = OR + OS.
Accueil
Géométrie affine
Convexité
Géométrie euclidienne
Isométries planes
Isométries de l’espace
Coniques
Géométrie analytique
Complexes
Homographies
Aide
Précédente
Suivante
Plein écran
Quitter
Retour
Solution
Accueil
Géométrie affine
Pour tout triplet (A,B,C) de points de E, il existe un et un seul triplet (R,S,T )
−→ −→ −→ −−→ −→ −→ −→ −→ −→
de points de E vérifiant OA = OS + OT , OB = OT + OR, OC = OR + OS : ce
−→ −−→ −→
−→ −−→ −→ −→
−→
système équivaut en effet à OA − OB − OC = −2OR, OB − OC − OA = −2OS,
−→ −→ −−→
−→
OC − OA − OB = −2OT .
Convexité
Géométrie euclidienne
Isométries planes
Isométries de l’espace
Coniques
Géométrie analytique
Complexes
Homographies
Aide
Précédente
Suivante
Un calcul élémentaire montre que le tétraèdre OABC est régulier si et seulement
si les points R, S, T sont les sommets ajacents à O d’un cube de sommet O.
Soient alors a, b, c, r, s, t les affixes des projections orthogonales des points A, B,
C, R, S, T sur le plan P . La relation
i
1h
(b + c − a)2 + (c + a − b)2 + (a + b − c)2
4
i
1h 2
= 3(a + b2 + c2 ) − 2(ab + bc + ca)
4
r2 + s2 + t2 =
et le résultat de la première partie montrent que r, s, t sont les affixes des projections orthogonales sur P des sommets adjacents à O d’un cube de sommet O si et
seulement si r2 +s2 +t2 = 0, i.e. si et seulement si 3(a2 +b2 +c2 )−2(ab+bc+ca) = 0.
Plein écran
Quitter
Table
Accueil
Géométrie affine
z−a
Lignes de niveau de
z−b
Convexité
Géométrie euclidienne
Soient a et b deux nombres complexes distincts. Déterminer les lignes de niveau
Isométries planes
(
Géométrie analytique
Complexes
z − a = k
Lk = z ∈ C |
z − b
Isométries de l’espace
Coniques
)
(k > 0)
z − a .
de la fonction z →
7
z − b
Homographies
Aide
Précédente
Suivante
Plein écran
Quitter
Solution
z − a 2
2
2
= k s’écrit |z − a| − k |z − b| = 0 ou encore :
La relation
z−b
Accueil
Géométrie affine
(1 − k 2 )z z̄ − (a − k 2 b)z̄ − (ā − k 2 b̄)z + aā − k 2 bb̄ = 0
Convexité
On reconnaı̂t là l’équation :
Géométrie euclidienne
a − k2b
k|b − a|
et
de
rayon
si k 6= 1 ;
1 − k2
1 − k2
• de la droite d’équation (b − a)z̄ + (b̄ − ā)z + |a|2 − |b|2 = 0 si k = 1.
a+b
a+b
) + (b − a)(z −
)=0
Cette dernière équation s’écrit encore (b − a)(z −
2
2
a+b
et exprime l’orthogonalité des vecteurs d’affixes b − a et z −
.
2
On retrouve ainsi des résultats bien connus. L’ensemble Lk est l’ensemble des points
M du plan dont le rapport des distances aux points A et B d’affixes respectives a
et b est égal à k. C’est un cercle dont le centre est le barycentre du système pondéré
kAB
((A,1),(B, − k 2 )) et le rayon
si k 6= 1, la médiatrice de AB si k = 1.
1 − k2
Isométries planes
Isométries de l’espace
Coniques
Géométrie analytique
Complexes
Homographies
Aide
Précédente
• du cercle de centre le point d’affixe
Suivante
Plein écran
Quitter
Table
Matrices et homographies
Accueil
Géométrie affine
Convexité
Géométrie euclidienne
Isométries planes
Isométries de l’espace
Coniques
Géométrie analytique
Montrer que l’application du groupe GL2 (C) des matrices carrées d’ordre 2 inverc
sibles à coefficients
complexes dans le groupe des homographies de C
qui à une
„
Ž
az
+
b
a b
est un
matrice A =
associe l’homographie fA définie par fA (z) =
c d
cz + d
homomorphisme de groupes. Déterminer son noyau. En déduire que le groupe des
c
homographies de C
est isomorphe au groupe quotient P GL2 (C) de GL2 (C) par le
sous-groupe des matrices λI2 , λ complexe non nul.
Complexes
Homographies
Aide
Précédente
Suivante
Plein écran
Quitter
Solution
„
Ž
a b
Soient A =
c d
plexes. L’égalité :
Accueil
Géométrie affine
Convexité
Géométrie euclidienne
Isométries planes
Isométries de l’espace
Coniques
Géométrie analytique
Complexes
„
et A0 =
a0 b 0
c0 d0
Ž
deux matrices inversibles à coefficients com„
Ž
a0 z + b
a 0
+b
cz+d
Ž
fA ◦ fA0 (z) = „ 0
az+b
c 0
+d
cz+d
(aa0 + bc0 )z + ab0 + bd0
=
(ca0 + dc0 )z + cb0 + dd0
= fAA0 (z) .
Homographies
Aide
Précédente
montre que l’application A 7→ fA est un homomorphisme de groupes. Son noyau est
le sous-groupe K des matrices A telles que fA = idCb , i.e. des matrices scalaires λI2 ,
λ complexe non nul. On en déduit que le groupe des homographies est isomorphe
au groupe quotient P GL2 (C) de GL2 (C) par K.
Suivante
Plein écran
Quitter
Table
Points fixes d’une homographie (1)
Accueil
Géométrie affine
Convexité
c
Montrer que toute homographie f de C
différente de l’identité admet exactement
c
un ou deux points fixes dans C.
Géométrie euclidienne
Isométries planes
Isométries de l’espace
Coniques
Géométrie analytique
Complexes
Homographies
Aide
Précédente
Suivante
Plein écran
Quitter
Solution
Suite
Soit f (z) =
Accueil
Géométrie affine
Convexité
Géométrie euclidienne
Isométries planes
Isométries de l’espace
az + b
c
une homographie de C.
cz + d
b
.
d−a
• Si c = 0 et a = d, le seul point fixe de f est ∞.
• Si c 6= 0, les points fixes de f sont les solutions dans C de l’équation du
second degré cz 2 + (d − a)z − b = 0. Cette équation admet 2 solutions si
(d − a)2 + 4bc 6= 0, une sinon.
• Si c = 0 et a 6= d, f laisse fixe ∞ et le point
Coniques
Géométrie analytique
Complexes
Homographies
Aide
Précédente
Suivante
Plein écran
Quitter
Suite
Points fixes d’une homographie (2)
Accueil
Géométrie affine
Convexité
Géométrie euclidienne
c
Montrer que toute homographie f de C
admettant exactement un point fixe dans
c
c
C est conjuguée dans le groupe des homographies de C
à une translation z 7→ z + b,
∗
b∈C .
Isométries planes
Isométries de l’espace
Coniques
Géométrie analytique
Complexes
Homographies
Aide
Précédente
Suivante
Plein écran
Quitter
Solution
Suite
Accueil
Géométrie affine
Convexité
Géométrie euclidienne
c
Soit f une homographie admettant le seul point fixe z0 dans C
et g une homographie
−1
vérifiant g(z0 ) = ∞. L’homographie h = g◦f ◦g a pour seul point fixe ∞. En effet
h(z) = z équivaut à f (g −1 (z)) = g −1 (z) et donc à g −1 (z) = z0 , i.e. z = g(z0 ) = ∞.
L’homographie h est donc de la forme h(z) = az + b avec a = 1 (sinon h aurait un
point fixe dans C en plus du point fixe ∞).
Isométries planes
Isométries de l’espace
Coniques
Géométrie analytique
Complexes
Homographies
Aide
Précédente
Suivante
Plein écran
Quitter
Suite
Points fixes d’une homographie (3)
Accueil
Géométrie affine
Convexité
Géométrie euclidienne
c
Montrer que toute homographie f de C
admettant exactement deux points fixes
c
c
dans C est conjuguée dans le groupe des homographies de C
à une similitude de
∗
centre 0 : z 7→ az, a ∈ C .
Isométries planes
Isométries de l’espace
Coniques
Géométrie analytique
Complexes
Homographies
Aide
Précédente
Suivante
Plein écran
Quitter
Solution
Suite
Accueil
Géométrie affine
Convexité
Géométrie euclidienne
Isométries planes
Soit f une homographie admettant les deux points fixes z1 et z2 et g une homographie vérifiant g(z1 ) = ∞, g(z2 ) = 0 (on vérifiera qu’il existe bien toujours une telle
homographie, en distinguant le cas où l’un des points fixes de f est ∞). L’homographie h = g ◦ f ◦ g −1 admet les deux points fixes ∞ et 0. En effet h(z) = z équivaut
à f (g −1 (z)) = g −1 (z) et donc à g −1 (z) = z1 , i.e. z = g(z1 ) = ∞, ou g −1 (z) = z2 , i.e.
z = g(z2 ) = 0. L’homographie h est donc de la forme h(z) = az + b, avec a ∈ C∗ ,
puisque h(∞) = ∞ et b = 0, puisque h(0) = 0.
Isométries de l’espace
Coniques
Géométrie analytique
Complexes
Homographies
Aide
Précédente
Suivante
Plein écran
Quitter
Suite
Points fixes d’une homographie (4)
Accueil
Géométrie affine
Convexité
Géométrie euclidienne
Montrer que toute homographie f involutive (i.e. différente de l’identité et telle que
f ◦ f soit l’identité) est conjuguée dans le groupe des homographies à la symétrie
de centre 0 : z 7→ −z.
Isométries planes
Isométries de l’espace
Coniques
Géométrie analytique
Complexes
Homographies
Aide
Précédente
Suivante
Plein écran
Quitter
Solution
Table
Accueil
Géométrie affine
Convexité
Une telle homographie admet nécessairement deux points fixes, sinon elle serait
conjuguée à une translation différente de l’identité et ne pourrait donc être involutive. Elle est donc conjuguée à une similitude de centre 0 : z 7→ az, a ∈ C∗ . Mais
une telle similitude n’est involutive que si a2 = 1 et a 6= 1, i.e. a = −1, auquel cas
elle est égale à la symétrie de centre 0.
Géométrie euclidienne
Isométries planes
Isométries de l’espace
Coniques
Géométrie analytique
Complexes
Homographies
Aide
Précédente
Suivante
Plein écran
Quitter
Table
Invariant anallagmatique de deux cercles (1)
Accueil
Géométrie affine
Convexité
Géométrie euclidienne
Soient Γ1 et Γ2 deux cercles sécants du plan complexe, O1 et O2 leurs centres, R1
Ù
et R2 leurs rayons, A et B leurs deux points d’intersection. Exprimer cos O
1 AO2
Ú
et cos O1 BO2 en fonction de R1 , R2 et O1 O2 .
Isométries planes
Isométries de l’espace
Coniques
Géométrie analytique
Complexes
Homographies
Aide
Précédente
Suivante
Plein écran
Quitter
Solution
Suite
On a évidemment :
Accueil
R12 + R22 − O1 O22
cos O1 AO2 = cos O1 BO2 =
2R1 R2
Ù
Géométrie affine
Convexité
Ú
(relation d’Al-Kashi dans les triangles O1 AO2 et O1 BO2 ).
Géométrie euclidienne
Isométries planes
Isométries de l’espace
Coniques
Géométrie analytique
Complexes
Homographies
Aide
Précédente
Suivante
Plein écran
Quitter
Suite
Invariant anallagmatique de deux cercles (2)
Accueil
Géométrie affine
Convexité
Géométrie euclidienne
Isométries planes
Soient Γ1 et Γ2 deux cercles quelconques du plan complexe, O1 et O2 leurs centres,
R1 et R2 leurs rayons, |z|2 − a1 z̄ − ā1 z + c1 = 0 et |z|2 − a2 z̄ − ā2 z + c2 = 0 leurs
R2 + R22 − O1 O22
en fonction de a1 , a2 , c1 , c2 .
équations complexes. Exprimer 1
2R1 R2
Isométries de l’espace
Coniques
La valeur absolue de ce nombre s’appelle invariant anallagmatique des deux cercles.
Géométrie analytique
Complexes
Homographies
Aide
Précédente
Suivante
Plein écran
Quitter
Retour
Solution
Suite
L’équation complexe du cercle de centre O, d’affixe a, et de rayon R s’écrit :
|z − a|2 = R2
Accueil
Géométrie affine
Convexité
Géométrie euclidienne
Isométries planes
Isométries de l’espace
Coniques
Géométrie analytique
Complexes
soit encore :
|z|2 − az̄ − āz + |a|2 − R2 = 0 .
Il en résulte qu’on a c1 = |a1 |2 − R12 , c2 = |a2 |2 − R22 et O1 O22 = |a1 − a2 |2 =
|a1 |2 + |a2 |2 − a1 ā2 − a2 ā1 , d’où :
R12 + R22 − O1 O22
a1 ā2 + a2 ā1 − c1 − c2
.
= q
2R1 R2
2 (|a1 |2 − c1 )(|a2 |2 − c2 )
Homographies
Aide
Précédente
Suivante
Plein écran
Quitter
Suite
Invariant anallagmatique de deux cercles (3)
Accueil
Géométrie affine
Convexité
Géométrie euclidienne
Isométries planes
Soient Γ1 et Γ2 deux cercles quelconques du plan complexe d’équations respectives
|z|2 − a1 z̄ − ā1 z + c1 = 0 et |z|2 − a2 z̄ − ā2 z + c2 = 0 et f une homographie du
plan complexe telle que f (Γ1 ) et f (Γ2 ) soient des cercles. Montrer que l’invariant
anallagmatique de f (Γ1 ) et f (Γ2 ) est égal à celui de Γ1 et Γ2 .
Isométries de l’espace
Coniques
Géométrie analytique
Complexes
Homographies
Aide
Précédente
Suivante
Plein écran
Quitter
Indication
Solution
Décomposez f en produit d’homographies simples et démontrez la propriété pour
chacune de ces homographies.
Accueil
Géométrie affine
Convexité
Géométrie euclidienne
Isométries planes
Isométries de l’espace
Coniques
Géométrie analytique
Complexes
Homographies
Aide
Précédente
Suivante
Plein écran
Quitter
Retour
Solution
Soit f (z) =
Accueil
relation
Géométrie affine
Convexité
Géométrie euclidienne
Isométries planes
Isométries de l’espace
Coniques
Géométrie analytique
Complexes
Homographies
Aide
az + b
une homographie qui n’est pas une similitude (c 6= 0). La
cz + d
a
bc − ad
+
c c(cz + d)
montre que f s’écrit comme un produit f = s2 ◦g◦s1 , où s1 et s2 sont des similitudes
1
directes et g est définie par g(z) = . Les similitudes conservent évidemment
z
l’invariant anallagmatique et f (Γ1 ) et f (Γ2 ) sont des cercles si et seulement si g
transforme les cercles s1 (Γ1 ) et s1 (Γ2 ) en des cercles, i.e. si ces cercles ne passent
pas par l’origine. Il suffit donc de démontrer la propriété dans le cas f = g.
Mais g transforme le cercle Γ d’équation |z|2 − az̄ − āz + c = 0 (c 6= 0) en le cercle
a
ā
1
ā
1
Γ0 d’équation |z|2 − z − z̄ + = 0, i.e. de paramètres a0 = , c0 = . Un calcul
c
c
c
c
c
élémentaire montre alors que les invariants anallagmatiques de deux cercles et de
leurs images par g sont égaux.
f (z) =
Précédente
Suivante
Plein écran
Quitter
Remarque
Remarque :
Accueil
Géométrie affine
Convexité
Géométrie euclidienne
Isométries planes
Isométries de l’espace
Coniques
Cet exercice montre en particulier qu’une homographie conserve l’angle sous lequel se coupent deux cercles (à condition que les images de ces cercles soient des
cercles); en particulier, elle transforme des cercles orthogonaux en des cercles orthogonaux. Cette propriété est en fait un cas particulier d’une propriété beaucoup plus
générale : toute homographie est une transformation conforme, c’est-à-dire qu’elle
conserve l’angle de deux courbes.
Géométrie analytique
Complexes
Homographies
Aide
Précédente
Suivante
Plein écran
Quitter
Retour
Table
Aide à la navigation
Accueil
Géométrie affine
Convexité
Géométrie euclidienne
Isométries planes
Isométries de l’espace
Coniques
Géométrie analytique
Complexes
Homographies
Aide
Précédente
Vous pouvez à partir de n’importe quelle page accéder à la liste des exercices de
chaque chapitre en cliquant à gauche sur le titre de ce chapitre. En cliquant sur un
élément de cette liste, vous accédez à l’énoncé correspondant.
Aide
Le bouton
vous ramène à cette page. Les boutons Précédente et Suivante
vous permettent de vous déplacer dans l’historique de votre navigation (pas dans
l’ordre physique des pages). Le bouton Plein écran permet de basculer du mode
Quitter
ferme Acrobat
plein écran au mode fenêtre et vice-versa. Le bouton
Reader.
La roulette et les boutons de votre souris vous permettent également de vous
déplacer dans le texte (le fonctionnement est un peu différent en mode plein écran
et en mode fenêtre). En mode fenêtre, un clic droit permet d’accéder à un menu
contextuel et de retrouver les outils habituels d’Acrobat Reader (utile par exemple
si vous voulez zoomer sur une figure ou imprimer une page).
Les liens cliquables en cours de texte sont indiqués en rouge.
Suivante
Plein écran
Quitter
Un conseil : gardez toujours une feuille de papier et un crayon à portée de main :
une figure même grossière que vous tracerez vous-même vous en apprendra souvent
plus que la contemplation béate de l’écran.
C’est bien : revenez. . .
Accueil
Géométrie affine
Convexité
Géométrie euclidienne
Isométries planes
Isométries de l’espace
Coniques
Géométrie analytique
Complexes
Homographies
Aide
Précédente
Suivante
Plein écran
Quitter
Téléchargement